Test Bank for Porths Pathophysiology Concepts of Altered Health States 2nd Canadian Edition by Ruth

Page 1

TEST BANK Porths Pathophysiology Concepts of Altered Health Porths Pathophysiology Concepts of Altered Health States States 2nd Canadian Edition by Ruth Hannon


Question 1: (see full question)

You selected:

The nurse is caring for a client with renal failure experiencing shortness of breath and increased respiratory rate. The arterial blood gas reflects a pH of 7.10 and a HCO3 level of 18 mEq/L. How does the nurse interpret these findings? Metabolic acidosis

Correct Explanation:

Metabolic acidosis would be diagnosed based on the findings related to a low pH level and a low bicarbonate level. The other conditions would not result from the findings provided ..... (more)

Reference:

Grossman S, Porth CM (2014). Porth’s Pathophysiology: Concepts of Altered Health States. 9th ed. Philadelphia: Lippincott Williams & Wilkins, Chapter 40, Disorders of Acid-Base Balance, p. 1071.

Question 2:

When interpreting the arterial blood gas, the nurse recognizes which of these reflects the value for bicarbonate?

(see full question)

You selected:

HCO3-

Correct Explanation:

Bicarbonate is abbreviated as HCO3. H2CO3 is the abbreviation for carbonic acid, HCl is the abbreviation for hydrochloric acid, and H+ is the abbreviation for hydrogen ..... (more)

Reference:

Grossman S, Porth CM (2014). Porth’s Pathophysiology: Concepts of Altered Health States. 9th ed. Philadelphia: Lippincott Williams & Wilkins, Chapter 40, Disorders of Acid-Base Balance, p. 1072.

Question 3:

A client's most recent laboratory results suggest the presence of metabolic alkalosis. What action by the nurse best addresses a potential cause of this acid-base imbalance?

(see full question)


You selected:

Administering an antiemetic to treat the client's frequent vomiting

Correct Explanation:

Vomiting results in the loss of hydrogen ions, potentially resulting in metabolic alkalosis. Constipation and skin breakdown are not among the most common causes of metabolic alka ... (more)

Reference:

Grossman S, Porth CM (2014). Porth’s Pathophysiology: Concepts of Altered Health States. 9th ed. Philadelphia: Lippincott Williams & Wilkins, Chapter 40, Disorders of Acid-Base Balance, p. 1076.

Question 4:

The nurse recognizes the role of the lungs in acid-base balance is regulation of which of the following?

(see full question)

You selected:

CO2

Correct

(see full question)

A client has these arterial blood gas values: anion gap 20 mEq/L, pH 7.29, pCO2 37mm Hg, HCO3- 11 mEq/L, base excess -6 mEq/L. With what condition do these values correspond?

You selected:

Lactic acidosis

Question 5:

Correct Explanation:

Anion gap (AG) is the difference between the plasma concentration of sodium ions and the sum of the measured anions (chloride and bicarbonate). Normally, the AG value should be bet ... (more)

Reference:

Grossman S, Porth CM (2014). Porth’s Pathophysiology: Concepts of Altered Health States. 9th ed. Philadelphia: Lippincott Williams & Wilkins, Chapter 40, Disorders of Acid-Base Balance, pp. 1069-1070, 1072.

Question 6:

The nurse is reviewing the following lab results of a client diagnosed with renal failure:

(see full question)


pH: 7.24 PCO2: 38 mm Hg HCO3:18 mEq/L The nurse would interpret this as: You selected:

Metabolic alkalosis

Incorrect Correct response:

Metabolic acidosis

Explanation:

Metabolic acidosis would be diagnosed based on the findings related to a low pH level ( ... (more)

Reference:

Grossman S, Porth CM (2014). Porth’s Pathophysiology: Concepts of Altered Health States. 9th ed. Philadelphia: Lippincott Williams & Wilkins, Chapter 40, Disorders of Acid-Base Balance, p. 1071.

Question 7:

What is the nurse’s expectation about a client’s ability to compensate for a metabolic blood gas disorder?

(see full question)

You selected:

The client will compensate with the respiratory system.

Correct Explanation:

The one thing that a nurse can expect is that a client with a metabolic disorder will compensate with the respiratory system. The client cannot compensate with the same system. The ... (more)

Reference:

Grossman S, Porth CM (2014). Porth’s Pathophysiology: Concepts of Altered Health States. 9th ed. Philadelphia: Lippincott Williams & Wilkins, Chapter 40, Disorders of Acid Base Balance, p. 1071.

Question 8:

A client with ethylene glycol toxicity is restless, and stating he has flank pain. What intervention should the nurse perform to minimize complications?

(see full question)

You selected:

Increase IV fluids


Correct

Question 9: (see full question)

You selected:

A 77-year-old woman has been brought to the emergency department by her daughter because of a sudden and unprecedented onset of confusion. The client admits to ingesting large amounts of baking soda this morning to treat some “indigestion.” How will the woman's body attempt to resolve this disruption in acid–base balance? Increased renal HCO3- reabsorption

Incorrect Correct response:

Hypoventilation

Explanation:

When neurologic manifestations occur with metabolic alkalosis, they include mental confusion, hyperactive reflexes, tetany, and carpopedal spasm. Respiratory compensation will take ... (more)

Reference:

Grossman S, Porth CM (2014). Porth’s Pathophysiology: Concepts of Altered Health States. 9th ed. Philadelphia: Lippincott Williams & Wilkins, Chapter 40, Disorders of Acid-Base Balance, p. 1076.

Question 10:

The nurse is caring for the following group of clients. Select the client most likely to be diagnosed with respiratory alkalosis.

(see full question)

You selected:

A 26-year-old female with anxiety who has been hyperventilating

Correct Explanation:

Respiratory alkalosis can occur with hyperventilating and the loss of CO2.The other three clients are more at risk for respiratory acidosis as a result of retaining CO2........... (more)

Question 1: (see full question)

The nurse is caring for a client with acute primary respiratory acidosis. When determining the cause of the acidosis the nurse is aware that which of these is most common?

You selected:

Impaired alveolar ventilation

Correct Explanation:

Acute respiratory acidosis is frequently caused by impaired alveolar ventilation


with CO2 retention. Increased metabolic acids, such as lactic acid, are characteristic of metabolic ... (more)

Reference:

Grossman S, Porth CM (2014). Porth’s Pathophysiology: Concepts of Altered Health States. 9th ed. Philadelphia: Lippincott Williams & Wilkins, Chapter 40, Disorders of Acid-Base Balance, p. 1071.

Question 2:

As other mechanisms prepare to respond to a pH imbalance, immediate buffering is a result of increased:

(see full question)

You selected:

Bicarbonate/carbonic acid regulation

Correct Explanation:

The bicarbonate buffering system, which is the principal ECF buffer, uses H2CO3 as its weak acid and bicarbonate salt such as sodium bicarbonate (NaHCO3) as its weak base. It subst ... (more)

Reference:

Grossman S, Porth CM (2014). Porth’s Pathophysiology: Concepts of Altered Health States. 9th ed. Philadelphia: Lippincott Williams & Wilkins, Chapter 40, Disorders of Acid-Base Balance, p. 1063.

Question 3:

A client has been admitted to the hospital after losing 20 kg (44 lb) over the past 3 months, largely due to frequent induction of vomiting. What intervention should the nurse anticipate in the treatment of the client's resulting acid-base imbalance?

(see full question)

You selected:

Fluid replacement with an intravenous solution containing KCl as prescribed

Correct Explanation:

Vomiting ultimately results in the increase in pH that constitutes metabolic alkalosis; intravenous KCl solutions are a mainstay of treatment for this acidbase imbalance. Respirat ... (more)

Question 4:

The nurse is caring for a client with metabolic alkalosis. Which of these arterial blood gas results supports this diagnosis?

(see full question)

You selected:

pH of 7.50 and HCO3 of 45 mEq/L


Correct Explanation:

Metabolic alkalosis is diagnosed based on increased pH and HCO3 levels, which is represented in the first option. pH of 7.25 and HCO3 of 18 mEq/L represents metabolic acidosis, whil ... (more)

Reference:

Grossman S, Porth CM (2014). Porth’s Pathophysiology: Concepts of Altered Health States. 9th ed. Philadelphia: Lippincott Williams & Wilkins, Chapter 40, Disorders of Acid-Base Balance, p. 1071

Question 5: (see full question)

The nurse caring for a client with respiratory acidosis examines arterial blood gas (ABG) results. Which change from the initial value indicates the client’s respiratory acidosis is improving?

You selected:

CO2 has decreased

Correct Explanation:

In respiratory acidosis, the client has a net gain of carbon dioxide (CO2) which creates a drop in pH. The pH needs to move away from acidosis, which is the low end of the pH scale ... (more)

Reference:

Grossman S, Porth CM (2014). Porth’s Pathophysiology: Concepts of Altered Health States. 9th ed. Philadelphia: Lippincott Williams & Wilkins, Chapter 40, Disorders of Acid-Base Balance, pp. 1078-1079.

Question 6: (see full question)

To calculate the H2CO3 content of the blood, the nurse needs to measure the Pco2 (partial pressure of CO2) by its solubility coefficient. What is the solubility coefficient of CO2?

You selected:

0.3

Incorrect Correct response:

0.03

Question 7:

A client has these arterial blood gas values: anion gap 20 mEq/L, pH 7.29, pCO2 37mm Hg, HCO3- 11 mEq/L, base excess -6 mEq/L. With what condition do these values correspond?

(see full question)


You selected:

Hyperkalemia

Incorrect Correct response:

Lactic acidosis

Explanation:

Anion gap (AG) is the difference between the plasma concentration of sodium ions and the sum of the measured anions (chloride and bicarbonate). Normally, the AG value should be bet ... (more)

Reference:

Grossman S, Porth CM (2014). Porth’s Pathophysiology: Concepts of Altered Health States. 9th ed. Philadelphia: Lippincott Williams & Wilkins, Chapter 40, Disorders of Acid-Base Balance, pp. 1069-1070, 1072.

Question 8:

A client with ethylene glycol toxicity is restless, and stating he has flank pain. What intervention should the nurse perform to minimize complications?

(see full question)

You selected:

Encourage ambulation

Incorrect Correct response:

Increase IV fluids

Explanation:

Symptoms of ethylene glycol toxicity appear in stages. Within the first 12 hours, the client may appear drunk or comatose. In the second stage the client may develop tachycardia or ... (more)

Reference:

Grossman S, Porth CM (2014). Porth’s Pathophysiology: Concepts of Altered Health States. 9th ed. Philadelphia: Lippincott Williams & Wilkins, Chapter 40, Disorders of Acid-Base Balance, p. 1074.

Question 9:

In which client would the nurse be most likely to assess the signs and symptoms of an acid-base imbalance?

(see full question)

You selected:

A client with chronic obstructive pulmonary disease whose most recent arterial blood gasses reveal a PCO2 of 51 mm Hg

Correct Explanation:

An elevated PCO2 is commonly associated with respiratory acidosis because excess CO2 ultimately increases the concentration of H+ ions. Increased intracranial pressure, oxytocin in ... (more)


Reference:

Grossman S, Porth CM (2014). Porth’s Pathophysiology: Concepts of Altered Health States. 9th ed. Philadelphia: Lippincott Williams & Wilkins, Chapter 40, Disorders of Acid-Base Balance, p. 1069.

Question 10: (see full question)

The nurse is assessing blood gas results for a patient with diabetes and ketoacidosis and notes a pH level of 7.15. Which of the following dysrhythmias should the nurse closely monitor while treating this patient?

You selected:

First-degree heart block

Incorrect Correct response:

Ventricular tachycardia

Question 1: (see full question)

A client with a long history of alcohol abuse has been admitted to the emergency department after several of days of heavy drinking. The nurse can best promote the restoration of the client's acid-base balance by:

You selected:

encouraging pursed lip breathing and deep breathing and coughing exercises.

Incorrect Correct response:

administering intravenous sodium bicarbonate as prescribed.

Explanation:

Sodium bicarbonate is among the more common treatments for the metabolic acidosis that results for high alcohol intake. Breathing exercises do not have appreciable effect, though r ... (more)

Reference:

Grossman S, Porth CM (2014). Porth’s Pathophysiology: Concepts of Altered Health States. 9th ed. Philadelphia: Lippincott Williams & Wilkins, Chapter 40, Disorders of Acid-Base Balance, p. 1076.

Question 2: (see full question)

The nurse is caring for a client who has developed hypoxemia and tissue hypoxia. Which of these interventions does the nurse set as a priority intervention?

You selected:

Administration of supplemental oxygen


Correct Explanation:

Hypoxia and hypoxemia may be corrected by administration of supplemental oxygen. People with hyperventilation may benefit from reassurance, rebreathing from a paper bag during sym ... (more)

Reference:

Grossman S, Porth CM (2014). Porth’s Pathophysiology: Concepts of Altered Health States. 9th ed. Philadelphia: Lippincott Williams & Wilkins, Chapter 40, Disorders of Acid-Base Balance, p. 1080.

Question 3:

The renal control mechanism of restoring the acid-base balance is accomplished through which process?

(see full question)

You selected:

Reabsorption of HCO3 and excretion of H+ restores acid-base balance through the renal control mechanisms.

Correct Explanation:

Reabsorption of HCO3 and excretion of H+ restores acid-base balance through the renal control mechanisms. Respiratory control mechanisms of restoring acid-base balance are done via ... (more)

Question 4:

The nurse is evaluating a client’s blood gases. The client has a pH of 7.35. How does the nurse interpret this value?

(see full question)

You selected:

It indicates acidosis.

Incorrect Correct response:

It is within a normal range.

Explanation:

The normal body range of pH is 7.35 to 7.45. The other answers are incorrect. An acidotic value would be indicated by a pH less than 7.35 and an alkalotic level would be indicated ... (more)

Reference:

Grossman S, Porth CM (2014). Porth’s Pathophysiology: Concepts of Altered Health States. 9th ed. Philadelphia: Lippincott Williams & Wilkins, Chapter 40, Disorders of Acid-Base Balance, p. 1070.


Question 5: (see full question)

You selected:

A client's most recent laboratory results suggest the presence of metabolic alkalosis. What action by the nurse best addresses a potential cause of this acid-base imbalance? Assessing the client's bowel sounds and administering scheduled stool softeners

Incorrect Correct response:

Administering an antiemetic to treat the client's frequent vomiting

Explanation:

Vomiting results in the loss of hydrogen ions, potentially resulting in metabolic alkalosis. Constipation and skin breakdown are not among the most common causes of metabolic alka ... (more)

Reference:

Grossman S, Porth CM (2014). Porth’s Pathophysiology: Concepts of Altered Health States. 9th ed. Philadelphia: Lippincott Williams & Wilkins, Chapter 40, Disorders of Acid-Base Balance, p. 1076.

Question 6: (see full question)

The body regulates the pH of its fluids by what mechanism? (Select all that apply.)

You selected:

• Chemical buffer systems of the body fluids • The kidneys • The liver

Incorrect Correct response:

• Chemical buffer systems of the body fluids • The lungs • The kidneys

Explanation:

The pH of body fluids is regulated by three major mechanisms: (1) chemical buffer systems of the body fluids, which immediately combine with excess acids or bases to prevent ... (more)

Question 7:

A client has been diagnosed with metabolic acidosis. What assessment finding does the nurse expect?

(see full question)

You selected:

Increased PC02 above 45 mm/hg

Incorrect Correct response:

Decreased pH below 7.35


Explanation:

In metabolic acidosis, the client's pH will decrease below 7.35 or normal range. In addition , the client's HC03- will decrease to below 22 mEq/L.

Reference:

Grossman S, Porth CM (2014). Porth’s Pathophysiology: Concepts of Altered Health States. 9th ed. Philadelphia: Lippincott Williams & Wilkins, Chapter 40, Disorders of Acid-Base Balance, p. 1070.

Question 8:

A client with chronic obstructive pulmonary disease (COPD) receives oxygen in the emergency department at a rate of 4 L/min for acute respiratory distress. Later, the nurse finds the client unresponsive with a respiratory rate of 8/minute. What ABG values would the nurse expect to obtain?

(see full question)

You selected:

pH 7.41, pCO2 39mm Hg, HCO3- 23 mEq/L, pO2 91mm Hg

Incorrect Correct response:

pH 7.29, pCO2 67mm Hg, HCO3- 26 mEq/L, pO2 64mm Hg

Explanation:

The respiratory center in clients with COPD may have lost sensitivity to carbon dioxide (CO2) as a result of their disease. Thus, hypoxia becomes the stimulus to breathe. When give ... (more)

Reference:

Grossman S, Porth CM (2014). Porth’s Pathophysiology: Concepts of Altered Health States. 9th ed. Philadelphia: Lippincott Williams & Wilkins, Chapter 40, Disorders of Acid-Base Balance, pp. 1078-1079.

Question 9: (see full question)

To calculate the H2CO3 content of the blood, the nurse needs to measure the Pco2 (partial pressure of CO2) by its solubility coefficient. What is the solubility coefficient of CO2?

You selected:

0.03

Correct Explanation:

The H2CO3 content of the blood can be calculated by multiplying the partial pressure of CO2 (Pco2) by its solubility coefficient, which is 0.03.

Reference:

Grossman S, Porth CM (2014). Porth's Pathophysiology: Concepts of Altered


Health States. 9th ed. Philadelphia: Lippincott Williams & Wilkins, Chapter 40: Disorders of Acid-Base Balance, p. 1064.

Question 10: (see full question)

You selected:

As other mechanisms prepare to respond to a pH imbalance, immediate buffering is a result of increased: Hydrogen/potassium binding

Incorre ct Correct Bicarbonate/carbonic acid regulation respons e: Explana tion:

The bicarbonate buffering system, which is the principal ECF buffer, uses H2CO3 as its weak acid and bicarbonate salt such as sodium bicarbonate (NaHCO3) as its weak base. It subst ... (more)

Question 2: (see full question)

There are both metabolic and respiratory effects on the acid–base balance in the body. How do metabolic disorders change the pH of the body?

You selected:

Alter the plasma Hco3–

Correct Explanation:

Metabolic disorders produce an alteration in the plasma HCO3– concentration and result from the addition or loss of nonvolatile acid or alkali to or from the extracell ... (more)

Reference:

Grossman S, Porth CM (2014). Porth's Pathophysiology: Concepts of Altered Health States. 9th ed. Philadelphia: Lippincott Williams & Wilkins, Chapter 40: Disorders of Acid-Base Balance, p. 1071.

Question 3:

The renal control mechanism of restoring the acid-base balance is accomplished through which process?

(see full question)

You selected:

Reabsorption of HCO3 and excretion of H+ restores acid-base balance


through the renal control mechanisms. Correct Explanation:

Reabsorption of HCO3 and excretion of H+ restores acid-base balance through the renal control mechanisms. Respiratory control mechanisms of restoring acid-base balance are done via ... (more)

Reference:

Grossman S, Porth CM (2014). Porth’s Pathophysiology: Concepts of Altered Health States. 9th ed. Philadelphia: Lippincott Williams & Wilkins, Chapter 40, Disorders of Acid-Base Balance, p. 1068.

Question 4:

Which conditions could cause the anion gap to fall below normal? Select all that apply.

(see full question)

You selected:

• Lithium intoxication • Ketoacidosis • Hyperkalemia

Incorrect Correct response:

• Hyperkalemia • Lithium intoxication • Multiple myeloma

Explanation:

Anion gap (AG) is the difference between the plasma concentration of sodium ions and the sum of the measured anions (chloride and bicarbonate). Normally, the AG value should be bet ... (more)

Question 5: (see full question)

The nurse caring for a client with respiratory alkalosis examines arterial blood gas (ABG) results. Which change from the initial value indicates the client’s respiratory alkalosis is improving?

You selected:

pH has decreased

Correct Explanation:

In respiratory alkalosis, the client has a net loss of carbon dioxide (CO2) which creates a rise in pH. The pH needs to move away from alkalosis, which is the high end of the pH sc ... (more)

Reference:

Grossman S, Porth CM (2014). Porth’s Pathophysiology: Concepts of Altered


Health States. 9th ed. Philadelphia: Lippincott Williams & Wilkins, Chapter 40, Disorders of Acid-Base Balance, pp. 1079-1080.

Question 6: (see full question)

You selected:

The physician is treating a client in lactic acidosis due to cardiac arrest. Which of the following treatments will be effective in correcting this acidosis? Administration of sodium bicarbonate

Incorrect Correct response:

Treatments to improve tissue perfusion and oxygenation

Explanation:

In most people with circulatory shock, cardiac arrest, or sepsis, impaired oxygen delivery is the primary cause of lactic acidosis. In these situations, the administration of large ... (more)

Reference:

Grossman S, Porth CM (2014). Porth’s Pathophysiology: Concepts of Altered Health States. 9th ed. Philadelphia: Lippincott Williams & Wilkins, Chapter 40, Disorders of Acid-Base Balance, p. 1076.

Question 7:

The nurse is providing care for several clients on a medical unit. Which client likely has the highest risk for developing an acid-base imbalance?

(see full question)

You selected:

A client who is being treated for acute kidney injury and who requires dialysis

Correct Explanation:

Because of the key role that the kidneys play in the maintenance of acid-base balance, individuals with kidney disease are vulnerable to acid-base disorders. Anaphylaxis, syphilis ... (more)

Reference:

Grossman S, Porth CM (2014). Porth’s Pathophysiology: Concepts of Altered Health States. 9th ed. Philadelphia: Lippincott Williams & Wilkins, Chapter 40, Disorders of Acid-Base Balance, p. 1067.

Question 8:

A client with ethylene glycol toxicity is restless, and stating he has flank pain. What intervention should the nurse perform to minimize complications?

(see full question)


You selected:

Increase IV fluids

Correct Explanation:

Symptoms of ethylene glycol toxicity appear in stages. Within the first 12 hours, the client may appear drunk or

Question 9: (see full question)

A client's most recent laboratory results suggest the presence of metabolic alkalosis. What action by the nurse best addresses a potential cause of this acid-base imbalance?

You selected:

Administering an antiemetic to treat the client's frequent vomiting

Correct Explanation:

Vomiting results in the loss of hydrogen ions, potentially resulting in metabolic alkalosis. Constipation and skin breakdown are not among the most common causes of metabolic alka ... (more)

Reference:

Grossman S, Porth CM (2014). Porth’s Pathophysiology: Concepts of Altered Health States. 9th ed. Philadelphia: Lippincott Williams & Wilkins, Chapter 40, Disorders of Acid-Base Balance, p. 1076.

Question 10: (see full question)

The nurse caring for a client with metabolic acidosis examines arterial blood gas (ABG) results. Which change from the initial value indicates the client’s metabolic acidosis is improving?

You selected:

pH has increased

Correct Explanation:

In metabolic acidosis, the client has a net loss of bicarbonate (HCO3-) which creates a drop in pH. The pH needs to move away from acidosis, which is the low end of the pH scale, f ... (more)

Question 1:

For which acid-base imbalance will the nurse monitor a client taking large doses of loop diuretics?

(see full question)

You selected:

Metabolic alkalosis

Correct Explanation:

Loop and thiazide diuretics commonly cause metabolic alkalosis as a result of hydrogen and potassium ion excretion in the urine. This leads to increased reabsorption of bicarbonate ... (more)


Reference:

Grossman S, Porth CM (2014). Porth’s Pathophysiology: Concepts of Altered Health States. 9th ed. Philadelphia: Lippincott Williams & Wilkins, Chapter 40, Disorders of Acid-Base Balance, pp. 1068, 1077.

Question 2: (see full question)

A newly admitted client's diagnostic testing indicates metabolic acidosis. What assessment should the nurse perform when working with the interdisciplinary team to determine the cause?

You selected:

Assess the client's blood glucose levels

Correct Explanation:

Hyperglycemia can result in diabetic ketoacidosis, a form of metabolic acidosis. Changes in respiratory status would result in acid-base disorders of the respiratory, not metabolic ... (more)

Reference:

Grossman S, Porth CM (2014). Porth’s Pathophysiology: Concepts of Altered Health States. 9th ed. Philadelphia: Lippincott Williams & Wilkins, Chapter 40, Disorders of Acid-Base Balance, p. 1073.

Question 3: (see full question)

A client with a long history of alcohol abuse has been admitted to the emergency department after several of days of heavy drinking. The nurse can best promote the restoration of the client's acid-base balance by:

You selected:

administering intravenous sodium bicarbonate as prescribed.

Correct Explanation:

Sodium bicarbonate is among the more common treatments for the metabolic acidosis that results for high alcohol intake. Breathing exercises do not have appreciable effect, though r ... (more)

Reference:

Grossman S, Porth CM (2014). Porth’s Pathophysiology: Concepts of Altered Health States. 9th ed. Philadelphia: Lippincott Williams & Wilkins, Chapter 40, Disorders of Acid-Base Balance, p. 1076.


Question 4: (see full question)

You selected:

The nurse is caring for a client who complains of headache and blurred vision. The nurse recognizes these symptoms, accompanied by increased plasma partial pressure carbon dioxide (PCO2) level and decreased pH level are consistent with which of these diagnoses? Respiratory acidosis

Correct Explanation:

Respiratory acidosis is reflected in the ABG as an increased PCO2 and decreased pH level as well as headache, blurred vision, irritability, muscle twitching, and psychological dist ... (more)

Question 5:

Arterial blood gases of a client with a diagnosis of acute renal failure reveal a pH of 7.25, HCO3- level of 21 mEq/L, and decreased PCO2 level accompanied by a respiratory rate of 32. This client is most likely experiencing which disorder of acid–base balance?

(see full question)

You selected:

Metabolic alkalosis

Incorrect Correct response:

Metabolic acidosis

Explanation:

Metabolic acidosis involves a decreased serum HCO3- concentration along with a decrease in pH. In metabolic acidosis, the body compensates for the decrease in pH by increasing the ... (more)

Reference:

Grossman S, Porth CM (2014). Porth’s Pathophysiology: Concepts of Altered Health States. 9th ed. Philadelphia: Lippincott Williams & Wilkins, Chapter 40, Disorders of Acid-Base Balance, p. 1075.

Question 6:

What is the nurse’s expectation about a client’s ability to compensate for a metabolic blood gas disorder?

(see full question)

You selected:

The client will compensate with the same system.

Incorrect Correct response:

The client will compensate with the respiratory system.

Explanation:

The one thing that a nurse can expect is that a client with a metabolic disorder will compensate with the respiratory system. The client cannot compensate with the same system. The ... (more)


Reference:

Grossman S, Porth CM (2014). Porth’s Pathophysiology: Concepts of Altered Health States. 9th ed. Philadelphia: Lippincott Williams & Wilkins, Chapter 40, Disorders of Acid Base Balance, p. 1071.

Question 7:

The nurse is caring for a client with renal failure experiencing shortness of breath and increased respiratory rate. The arterial blood gas reflects a pH of 7.10 and a HCO3 level of 18 mEq/L. How does the nurse interpret these findings?

(see full question)

You selected:

Metabolic acidosis

Correct Explanation:

Metabolic acidosis would be diagnosed based on the findings related to a low pH level and a low bicarbonate level. The other conditions would not result from the findings provided ..... (more)

Question 8:

A client with chronic obstructive pulmonary disease (COPD) receives oxygen in the emergency department at a rate of 4 L/min for acute respiratory distress. Later, the nurse finds the client unresponsive with a respiratory rate of 8/minute. What ABG values would the nurse expect to obtain?

(see full question)

You selected:

pH 7.29, pCO2 67mm Hg, HCO3- 26 mEq/L, pO2 64mm Hg

Correct Explanation:

The respiratory center in clients with COPD may have lost sensitivity to carbon dioxide (CO2) as a result of their disease. Thus, hypoxia becomes the stimulus to breathe. When give ... (more)

Reference:

Grossman S, Porth CM (2014). Porth’s Pathophysiology: Concepts of Altered Health States. 9th ed. Philadelphia: Lippincott Williams & Wilkins, Chapter 40, Disorders of Acid-Base Balance, pp. 1078-1079.

Question 9:

The nurse is reviewing lab results of a client diagnosed with metabolic acidosis. The most important electrolyte for the nurse to assess would be:

(see full question)


You selected:

Potassium (K+)

Correct Explanation:

When excess H+ is present in the extracellular fluid (ECF), it moves into the intracellular fluid (ICF) in exchange for K+, and when excess K+ is present in the ECF, it moves into ... (more)

Reference:

Grossman S, Porth CM (2014). Porth’s Pathophysiology: Concepts of Altered Health States. 9th ed. Philadelphia: Lippincott Williams & Wilkins, Chapter 40, Disorders of Acid-Base Balance, p. 1067.

Question 10: (see full question)

The nurse is reviewing laboratory data for the client with an anion gap of 17. The nurse recognizes which of these conditions are associated with an increased anion gap mEq/L?

You selected:

Lactic acidosis

Correct Explanation:

The anion gap describes the difference between the serum concentration of the major measured cation (Na) and the sum of the measured anions (Cl and HCO3 ). This difference represen ... (more)

Question 1:

A patient staggers into the emergency department with his son who states that his father has not been well for a couple of weeks. The patient's heart rate is 120 and lung sounds reveal crackles and a respiratory rate of 30. The patient also complains of right flank pain. The son says that his mother has been acting strangely and thinks she might be poisoning her husband. Which of the following clinical manifestations indicate the type of poisoning involved?

(see full question)

You selected:

Ethylene glycol (antifreeze)

Correct Explanation:

Ethylene glycol is a solvent found in products ranging from antifreeze and deicing solutions to carpet and fabric cleaners. It tastes sweet and is intoxicating, factors that contri ... (more)

Reference:

Grossman S, Porth CM (2014). Porth’s Pathophysiology: Concepts of Altered Health States. 9th ed. Philadelphia: Lippincott Williams & Wilkins, Chapter 40, Disorders of Acid-Base Balance, p. 1074.


Question 2: (see full question)

You selected:

The nurse is caring for a client who complains of headache and blurred vision. The nurse recognizes these symptoms, accompanied by increased plasma partial pressure carbon dioxide (PCO2) level and decreased pH level are consistent with which of these diagnoses? Respiratory alkalosis

Incorrect Correct response:

Respiratory acidosis

Explanation:

Respiratory acidosis is reflected in the ABG as an increased PCO2 and decreased pH level as well as headache, blurred vision, irritability, muscle twitching, and psychological dist ... (more)

Reference:

Grossman S, Porth CM (2014). Porth’s Pathophysiology: Concepts of Altered Health States. 9th ed. Philadelphia: Lippincott Williams & Wilkins, Chapter 40, Disorders of Acid-Base Balance, p. 1079

Question 3: (see full question)

The client has arterial blood gases (ABG) drawn. The results are a pH of 7.50, pCO2 30, and HCO3- 24. The nurse understands that which of the following could cause these ABG results?

You selected:

Mechanical ventilation

Correct Explanation:

The pH of 7.50 indicates alkalosis which is due to a low pCO2 (30). One cause of respiratory alkalosis is mechanical ventilation if the rate and tidal volume are set so that CO2 el ... (more)

Question 4: (see full question)

The nurse caring for a client with respiratory alkalosis examines arterial blood gas (ABG) results. Which change from the initial value indicates the client’s respiratory alkalosis is improving?

You selected:

O2 has increased

Incorrect Correct response:

pH has decreased


Explanation:

In respiratory alkalosis, the client has a net loss of carbon dioxide (CO2) which creates a rise in pH. The pH needs to move away from alkalosis, which is the high end of the pH sc ... (more)

Reference:

Grossman S, Porth CM (2014). Porth’s Pathophysiology: Concepts of Altered Health States. 9th ed. Philadelphia: Lippincott Williams & Wilkins, Chapter 40, Disorders of Acid-Base Balance, pp. 1079-1080.

Question 5: (see full question)

The nurse caring for a client with metabolic alkalosis examines arterial blood gas (ABG) results. Which change from the initial value indicates the client’s metabolic alkalosis is improving?

You selected:

pH has increased

Incorrect Correct response:

HCO3- has decreased

Explanation:

In metabolic alkalosis, the client has a net gain of bicarbonate (HCO3-) which creates a rise in pH. The pH needs to move away from alkalosis, which is the high end of the pH scale ... (more)

Reference:

Grossman S, Porth CM (2014). Porth’s Pathophysiology: Concepts of Altered Health States. 9th ed. Philadelphia: Lippincott Williams & Wilkins, Chapter 40, Disorders of Acid-Base Balance, pp. 1076-1078.

Question 6: (see full question)

The body has built-in compensatory mechanisms that take over when correction of pH is not possible or cannot be immediately achieved. What are these compensatory mechanisms considered?

You selected:

Short-term measures that depend on first-line correction mechanisms

Incorrect Correct response:

Interim measures that permit survival

Explanation:

Often, compensatory mechanisms are interim measures that permit survival while the body attempts to correct the primary disorder. All of the other answers are wrong.


Question 7: (see full question)

You selected:

Which arterial blood gas (ABG) values tell the nurse a client is in respiratory acidosis? Select all that apply. • HCO3- of 29 mEq/L • Anion gap of 8 mEq/L

Incorrect Correct response:

• pCO2 of 57mm Hg • pH 7.32

Explanation:

ABGs measure pH, carbon dioxide (pCO2), bicarbonate ion (HCO3-), oxygen pO2, base excess, and the anion gap. A pH that is below 7.35 is considered acidic while above 7.45 is consid ... (more)

Reference:

Grossman S, Porth CM (2014). Porth’s Pathophysiology: Concepts of Altered Health States. 9th ed. Philadelphia: Lippincott Williams & Wilkins, Chapter 40, Disorders of Acid-Base Balance, pp. 1069-1070, 1078.

Question 8:

Which intravenous solution would be appropriate to treat metabolic acidosis?

(see full question)

You selected:

NaHCO3

Correct Explanation:

Sodium bicarbonate will treat metabolic acidosis in clients with adequate perfusion and gas exchange. Potassium chloride will treat metabolic alkalosis as the potassium allows the ... (more)

Reference:

Grossman S, Porth CM (2014). Porth’s Pathophysiology: Concepts of Altered Health States. 9th ed. Philadelphia: Lippincott Williams & Wilkins, Chapter 40, Disorders of Acid-Base Balance, pp. 1076-1079.

Question 9:

What assessment would a nurse expect when caring for a client with respiratory acidosis? Select all that apply.

(see full question)

You selected:

• Decreased level of consciousness • Cool and clammy skin


Incorrect Correct response:

• Decreased level of consciousness • Decreased cardiac output • Anorexia, nausea, vomiting

Explanation:

Conditions that cause acidosis suppress neural excitability, so as the condition progresses, clients become lethargic, stuporous, or comatose. Skin may be warm and flushed because ... (more)

Question 10:

The nurse teaches a client with renal insufficiency to limit protein intake. What is an appropriate explanation for this restriction?

(see full question)

You selected:

Protein metabolism increases the need for renal excretion of acids.

Correct Explanation:

Fixed, or nonvolatile acids, such as sulfuric, hydrochloric, and phosphoric acid are the products of protein metabolism which are eliminated by the kidneys. In renal insufficiency ... (more)

Question 1:

A nurse is providing care for several clients on an acute medicine unit. Which client should the nurse recognize as being at the highest risk for metabolic alkalosis?

(see full question)

You selected:

A client on continuous nasogastric suction and whose hypertension is being treated with diuretics

Correct Explanation:

Nasogastric suction creates a significant risk for metabolic alkalosis due to the loss of gastric acids. This risk is compounded by the concurrent use of diuretics. Alcohol use is ... (more)

Reference:

Grossman S, Porth CM (2014). Porth’s Pathophysiology: Concepts of Altered Health States. 9th ed. Philadelphia: Lippincott Williams & Wilkins, Chapter 40, Disorders of Acid-Base Balance, p. 1076.

Question 2:

A patient arrives in the emergency department by ambulance with a family member stating, "He took an overdose of sleeping pills and I found him breathing very shallowly." For which of the following types of acid-base

(see full question)


disturbance will the nurse anticipate this patient will be treated? You selected:

Respiratory acidosis

Correct Explanation:

Respiratory acidosis occurs in conditions that impair alveolar ventilation and cause an increase in plasma PCO2, also know as hypercapnia, along with a decrease in pH. Respiratory ... (more)

Reference:

Grossman S, Porth CM (2014). Porth’s Pathophysiology: Concepts of Altered Health States. 9th ed. Philadelphia: Lippincott Williams & Wilkins, Chapter 40, Disorders of Acid-Base Balance, p. 1078.

Question 3:

The nurse is caring for a patient in septic shock. The nurse knows that which of the following is a compensatory mechanism for this acid-base imbalance?

(see full question)

You selected:

Kidney excretion of H+ decreases

Incorrect Correct response:

Respiratory rate of 32

Explanation:

Septic shock is a cause of metabolic acidosis. Acidosis typically produces a compensatory increase in respiratory rate (32) with a decrease in pCO2. Metabolic alkalosis leads to a ... (more)

Reference:

Grossman S, Porth CM (2014). Porth’s Pathophysiology: Concepts of Altered Health States. 9th ed. Philadelphia: Lippincott Williams & Wilkins, Chapter 40, Disorders of Acid-Base Balance, p. 1075.

Question 4: (see full question)

The nurse caring for the client with respiratory alkalosis and renal compensation determines which of these diagnostic findings is consistent with this disorder?

You selected:

Increased PCO2 and HCO3, and decreased pH

Incorrect Correct response:

Decreased PCO2 and HCO3, and increased pH


Question 5: (see full question)

You selected:

A client has an increase in her anion gap (AG). What does the nurse determine is the significance of this finding? It indicates the client has metabolic acidosis.

Correct Explanation:

Increased anion gap is found in conditions such as lactic acidosis or ketoacidosis or other conditions leading to metabolic acidosis. The other answers are incorrect as they do not ... (more)

Reference:

Grossman S, Porth CM (2014). Porth’s Pathophysiology: Concepts of Altered Health States. 9th ed. Philadelphia: Lippincott Williams & Wilkins, Chapter 40, Disorders of Acid Base Balance, p. 1070.

Question 6: (see full question)

The nurse is reviewing laboratory data for the client with an anion gap of 17. The nurse recognizes which of these conditions are associated with an increased anion gap mEq/L?

You selected:

Lactic acidosis

Correct Explanation:

The anion gap describes the difference between the serum concentration of the major measured cation (Na) and the sum of the measured anions (Cl and HCO3 ). This difference represen ... (more)

Reference:

Grossman S, Porth CM (2014). Porth’s Pathophysiology: Concepts of Altered Health States. 9th ed. Philadelphia: Lippincott Williams & Wilkins, Chapter 40, Disorders of Acid-Base Balance, p. 1069.

Question 7: (see full question)

The nurse is caring for a client who has consistently been vomiting throughout several days. What will the nurse expect to find when evaluating the client’s blood gas values?

You selected:

Decreased bicarbonate values

Incorrect Correct response:

Increased pH value

Explanation:

The client who is vomiting will be expected to lose gastric acid, resulting in a


metabolic alkalosis. The clients’ PC02 would not be expected to increase.

Reference:

Grossman S, Porth CM (2014). Porth’s Pathophysiology: Concepts of Altered Health States. 9th ed. Philadelphia: Lippincott Williams & ilkins, Chapter 40, Disorders of Acid-Base Balance, p. 1075.

Question 8: (see full question)

A child has been admitted to the emergency department after consuming ethylene glycol in the form of antifreeze. The nurse should prioritize which goal when planning the client's care?

You selected:

The client will remain free from the effects of metabolic acidosis.

Correct Explanation:

Ethylene glycol ingestion causes metabolic acidosis. Normal blood pH is 7.35 to 7.45. Bradypnea would not benefit the client, and has the potential to inhibit respiratory compensat ... (more)

Question 9:

For which acid-base imbalance will the nurse monitor a client taking large doses of loop diuretics?

(see full question)

You selected:

Metabolic alkalosis

Correct Explanation:

Loop and thiazide diuretics commonly cause metabolic alkalosis as a result of hydrogen and potassium ion excretion in the urine. This leads to increased reabsorption of bicarbonate ... (more)

Reference:

Grossman S, Porth CM (2014). Porth’s Pathophysiology: Concepts of Altered Health States. 9th ed. Philadelphia: Lippincott Williams & Wilkins, Chapter 40, Disorders of Acid-Base Balance, pp. 1068, 1077.

Question 10:

The nurse is caring for a client with renal failure experiencing shortness of breath and increased respiratory rate. The arterial blood gas reflects a pH of 7.10 and a HCO3 level of 18 mEq/L. How does the nurse interpret these findings?

(see full question)


You selected:

Respiratory alkalosis

Incorrect Correct response:

Metabolic acidosis

Explanation:

Metabolic acidosis would be diagnosed based on the findings related to a low pH level and a low bicarbonate level. The other conditions would not result from the findings provided ..... (more)

Question 1:

Which laboratory findings should you expect to see in a patient diagnosed with nephritic syndrome?

(see full question)

You selected:

Elevated urine protein and hypoalbuminemia

Correct Explanation:

The nephritic syndrome is due to glomerular disease that is usually of acute onset and is accompanied by grossly visible hematuria, mild to moderate proteinuria, and hypertension...... (more)

Reference:

Grossman S, Porth CM (2014). Porth’s Pathophysiology: Concepts of Altered Health States. 9th ed. Philadelphia: Lippincott Williams & Wilkins, Chapter 41, Disorders of Renal Function, p. 1101.

Question 2: (see full question)

A nurse is caring for a patient who has a recent history of passing calcium urinary stones. Which of the following is a priority nursing consideration for this patient?

You selected:

Hydration

Correct Explanation:

A major goal of treatment in people who have passed kidney stones or have had them removed is to prevent their recurrence. Adequate fluid intake reduces the concentration of stone- ... (more)

Question 3:

The physician suspects that a client with kidney stones has developed magnesium ammonium phosphate (struvite) stones based on which of the

(see full question)


following urinalysis results? Select all that apply. You selected:

• Alkaline urine pH • Elevated uric acid levels • Presence of cystine particles

Incorrect Correct response:

• Alkaline urine pH • High urine phosphate level • High bacterial count

Explanation:

Magnesium ammonium phosphate stones, also called struvite stones, form only in alkaline urine and in the presence of bacteria that possess an enzyme called urease, which splits the ... (more)

Reference:

Grossman, S. & Porth, C.M. Porth's Pathophysiology: Concepts of Altered Health States, 9th ed., Philadelphia: Lippincott Williams & Wilkins, 2014, Chapter 41, Disorders of Renal Function, p. 1091.

Question 4:

A nurse is caring for a patient with systemic lupus erythematosis (SLE). The patient asks why a urinalysis is necessary. The best answer is that a urinalysis would determine whether which of the following factors was present in the urine?

(see full question)

You selected:

Blood or protein

Correct Explanation:

Because of the high risk of kidney disease, all people with SLE should undergo routine urinalysis to monitor for the presence of hematuria or proteinuria.

Question 5:

Glomerulonephritis is usually caused by:

(see full question)

You selected:

Antigen-antibody complexes

Correct Explanation:

Two types of immune mechanisms have been implicated in the development of glomerular disease: injury resulting from antibodies reacting with fixed glomerular antigens and injury re ... (more)


Reference:

Grossman S, Porth CM (2014). Porth’s Pathophysiology: Concepts of Altered Health States. 9th ed. Philadelphia: Lippincott Williams & Wilkins, Chapter 41, Disorders of Renal Function, p. 1098.

Question 6: (see full question)

A patient has passed a kidney stone composed of uric acid (urate). Which of the following pathological conditions is a contributing factor for the development of this type of kidney stone?

You selected:

Gout

Correct Explanation:

Uric acid stones develop in conditions of gout and high concentrations of uric acid in the urine.

Reference:

Grossman S, Porth CM (2014). Porth’s Pathophysiology: Concepts of Altered Health States. 9th ed. Philadelphia: Lippincott Williams & Wilkins, Chapter 41, Disorders of Renal Function, p. 1091.

Question 7:

The nurse is assessing a patient who has a unilateral obstruction of the urinary tract. The nurse anticipates the patient may develop:

(see full question)

You selected:

Inability to control urination

Incorrect Correct response:

Increase in blood pressure

Explanation:

Hypertension is an occasional complication of urinary tract obstruction. It is more common in cases of unilateral obstruction in which renin secretion is enhanced, probably seconda ... (more)

Reference:

Grossman S, Porth CM (2014). Porth’s Pathophysiology: Concepts of Altered Health States. 9th ed. Philadelphia: Lippincott Williams & Wilkins, Chapter 41, Disorders of Renal Function, p. 1089

Question 8:

A patient who has had an intestinal bypass has developed a kidney stone.

(see full question)


Which of the following types of kidney stone does the nurse recognize that this patient will most likely be treated for? You selected:

Uric acid

Incorrect Correct response:

Calcium

Explanation:

Most kidney stones are calcium stones-calcium oxalate, calcium phosphate, or a combination of the two materials. Patients who have had intestinal bypass surgery are at a higher ris ... (more)

Question 9:

A patient is being evaluated for kidney stones. The nurse anticipates the patient will manifest:

(see full question)

You selected:

Acute onset of colicky or dull and achy intermittent flank pain

Correct Explanation:

One of the major manifestations of kidney stones is pain. Depending on the location, there are two types of pain associated with kidney stones—colicky or non colicky. The pain ma ... (more)

Reference:

Grossman S, Porth CM (2014). Porth’s Pathophysiology: Concepts of Altered Health States. 9th ed. Philadelphia: Lippincott Williams & Wilkins, Chapter 41, Disorders of Renal Function, p. 1091.

Question 10:

Which of the following is the most important factor in reducing nosocomial urinary tract infections?

(see full question)

You selected:

Irrigating urinary catheters with an antibiotic solution

Incorrect Correct response:

Using urinary catheters only when necessary and their prompt removal when no longer needed

Explanation:

Catheter-associated bacteriuria remains the most frequent cause of Gramnegative septicemia in hospitalized patients. Bacteriuria are a source of urethral irritation and provide a ... (more)

Reference:

Grossman S, Porth CM (2014). Porth’s Pathophysiology: Concepts of Altered Health States. 9th ed. Philadelphia: Lippincott Williams & Wilkins, Chapter 41,


Disorders of Renal Function, p. 1095.

Question 1: (see full question)

You selected:

Most common uncomplicated urinary tract infections are caused by enter through the urethra.

that

Escherichia coli

Correct Explanation:

Most commonly urinary tract infections (UTIs) are caused by Escherichia coli bacteria that enter through the urethra. Other uropathic pathogens include Staphyl ... (more)

Reference:

Grossman S, Porth CM (2014). Porth’s Pathophysiology: Concepts of Altered Health States. 9th ed. Philadelphia: Lippincott Williams & Wilkins, Chapter 41, Disorders of Renal Function, p. 1094.

Question 2:

Which additional physical finding would you anticipate seeing in a child suspected of having a Wilms tumor?

(see full question)

You selected:

Presence of a palpable flank mass

Incorrect Correct response:

Anomalies of the genitourinary system

Explanation:

An important feature of Wilms tumor is its association with other congenital anomalies, aniridia (absence of the iris), hemihypertrophy (enlargement of one side of the face or body ... (more)

Reference:

Grossman S, Porth CM (2014). Porth’s Pathophysiology: Concepts of Altered Health States. 9th ed. Philadelphia: Lippincott Williams & Wilkins, Chapter 41, Disorders of Renal Function, p. 1108

Question 3:

The nurse suspects that a newborn infant who presents with bilateral flank masses, impaired lung development, and oliguria may be suffering from which of the following disorders?

(see full question)


You selected:

Autosomal recessive polycystic kidney disease (ARPD)

Correct Explanation:

The typical infant with ARPD presents with bilateral flank masses, accompanied by severe renal failure, signs of impaired lung development, and variable degrees of liver fibrosis a ... (more)

Reference:

Grossman S, Porth CM (2014). Porth’s Pathophysiology: Concepts of Altered Health States. 9th ed. Philadelphia: Lippincott Williams & Wilkins, Chapter 41, Disorders of Renal Function, p. 1087.

Question 4:

The nurse is reviewing the lab results of a patient with suspected nephrotic syndrome. The nurse anticipates that the results to include:

(see full question)

You selected:

Protein in the urine

Correct Explanation:

In a person with nephrotic syndrome there is massive proteinuria (protein in the urine), serum hypoalbuminemia, generalized edema and hyperlipidemia.

Reference:

Grossman S, Porth CM (2014). Porth’s Pathophysiology: Concepts of Altered Health States. 9th ed. Philadelphia: Lippincott Williams & Wilkins, Chapter 41, Disorders of Renal Function, p. 1101.

Question 5:

Unilateral obstruction of the urinary tract may result in renin secretion, thereby leading to which of the following?

(see full question)

You selected:

Increased blood pressure

Correct Explanation:

In cases of unilateral obstruction of the kidney, renin secretion is enhanced, probably secondary to impaired renal blood flow. This would result in increased sodium retention, and ... (more)

Reference:

Grossman S, Porth CM (2014). Porth’s Pathophysiology: Concepts of Altered Health States. 9th ed. Philadelphia: Lippincott Williams & Wilkins, Chapter 41,


Disorders of Renal Function, p. 1090.

Question 6: (see full question)

You selected:

Which of the following patients is at greatest risk for developing a urinary tract infection (UTI)? Elderly female patient admitted with an indwelling Foley catheter that has been in place for 1 month

Correct Explanation:

Urinary catheters are a source of urethral irritation and provide a means for entry of microorganisms into the urinary tract. Catheter-associated bacteriuria remains the most freq ... (more)

Reference:

Grossman S, Porth CM (2014). Porth’s Pathophysiology: Concepts of Altered Health States. 9th ed. Philadelphia: Lippincott Williams & Wilkins, Chapter 41, Disorders of Renal Function, p. 1095.

Question 7:

An obese, male client with a history of gout and a sedentary lifestyle has been advised by his primary care provider to avoid organ meats, certain fish, and other foods that are high in purines. The care provider is demonstrating an awareness of the client's susceptibility to which of the following types of kidney stones?

(see full question)

You selected:

Cystine stones

Incorrect Correct response:

Uric acid stones

Explanation:

Uric acid stones develop in conditions of gout and when high concentrations of uric acid in the urine. Unlike radiopaque calcium stones, uric acid stones are not visible on x-ray f ... (more)

Reference:

Grossman S, Porth CM (2014). Porth’s Pathophysiology: Concepts of Altered Health States. 9th ed. Philadelphia: Lippincott Williams & Wilkins, Chapter 41, Disorders of Renal Function, p. 1192.


Question 8: (see full question)

What is the most common cause of a lower urinary tract infection?

You selected:

Escherichia coli

Correct Explanation:

Most uncomplicated lower UTIs are caused by Escherichia coli. The other organisms can cause UTIs, but are not the most common cause of infection.

Reference:

Grossman S, Porth CM (2014). Porth's Pathophysiology: Concepts of Altered Health States. 9th ed. Philadelphia: Lippincott Williams & Wilkins, Chapter 41: Disorders of Renal Function, p. 1094.

Question 9:

A patient diagnosed with Goodpasture’s syndrome would require which of the therapies to remove proteins and autoantibodies from the system?

(see full question)

You selected:

Plasmapheresis

Correct Explanation:

Plasmapheresis is used to filter the blood for removal of proteins and the circulating anti-GBM antibodies. Renal transplantation would return the kidneys to normal function, but a ... (more)

Reference:

Grossman S, Porth CM (2014). Porth’s Pathophysiology: Concepts of Altered Health States. 9th ed. Philadelphia: Lippincott Williams & Wilkins, Chapter 41, Disorders of Renal Function, p. 1101.

Question 10:

The nurse is caring for a female patient with recurring urinary tract infections (UTIs). The patient asks why yogurt is recommended in her diet. Which of the following is the nurse's best response?

(see full question)

You selected:

Yogurt with probiotics helps to defend against the colonization of organisms that can cause UTIs.

Correct Explanation:

In women, the normal flora of the periurethral area, which consists of organisms such as Lactobacillus, provides a defense against the colonization of uropathic bacteria. Evidence ... (more)


Question 1: (see full question)

You selected:

The family asks the nurse what the usual treatment of focal segmental glomerulosclerosis entails. The nurse responds: corticosteroids.

Correct Explanation:

The disorder usually is treated with corticosteroids. Although kidney transplantation is the preferred treatment for end-stage kidney disease, focal segmental glomerulonephritis oc ... (more)

Reference:

Grossman S, Porth CM (2014). Porth’s Pathophysiology: Concepts of Altered Health States. 9th ed. Philadelphia: Lippincott Williams & Wilkins, Chapter 41, Disorders of Renal Function, p. 1102.

Question 2:

The nurse reviews the lab results for a patient who has advanced autosomal dominant polycystic kidney disease (ADPKD). The patient 's hemoglobin is 8.8 g/dL. The nurse suspects this lab value is related to which of the following causes?

(see full question)

You selected:

Hemorrhage

Incorrect Correct response:

Reduced production of erythropoietin

Explanation:

As ADPKD progresses, the nephrons reduce the production of erythropoietin (EPO). EPO is necessary for red blood cell production by bone marrow, so EPO deficiency causes anemia.

Reference:

Grossman S, Porth CM (2014). Porth’s Pathophysiology: Concepts of Altered Health States. 9th ed. Philadelphia: Lippincott Williams & Wilkins, Chapter 41, Disorders of Renal Function, pp. 1086-1087.

Question 3:

The nurse discusses which of the following measures to prevent urinary tract infection (UTI) with a sexually active, 21-year-old woman? Select all that apply.

(see full question)


You selected:

• Drinking cranberry juice daily • Voiding after sexual intercourse • Voiding before sexual intercourse

Incorrect Correct response:

• Voiding after sexual intercourse • Drinking cranberry juice daily • Maintaining adequate fluid intake

Explanation:

Cranberry juice has been suggested as a preventative measure for women with recurrent UTIs. Evidence suggests that cranberry juice reduces bacterial adherence to the epithelial li ... (more)

Reference:

Grossman S, Porth CM (2014). Porth’s Pathophysiology: Concepts of Altered Health States. 9th ed. Philadelphia: Lippincott Williams & Wilkins, Chapter 41, Disorders of Renal Function, p. 1094.

Question 4:

One of the complications of asymptomatic urinary tract infection (UTI) during pregnancy is the risk of preterm birth.

(see full question)

You selected:

True

Correct Explanation:

Complications of asymptomatic UTI during pregnancy include preterm delivery of low birth weight infants, acute and chronic pyelonephritis, and persistent bacteriuria.

Reference:

Grossman S, Porth CM (2014). Porth’s Pathophysiology: Concepts of Altered Health States. 9th ed. Philadelphia: Lippincott Williams & Wilkins, Chapter 41, Disorders of Renal Function, p. 1096.

Question 5: (see full question)

A client has been diagnosed with systemic lupus erythematosus. The client now presents with sudden hematuria, variable proteinuria, and a decreased glomerular filtration rate. Which of the following is a probable diagnosis?

You selected:

Acute nephritic syndrome

Correct


Explanation:

Acute nephritic syndrome may occur as a secondary complicating disorder in systemic diseases, such as SLE. In its most dramatic form, acute nephritic syndrome is characterize ... (more)

Reference:

Grossman S, Porth CM (2014). Porth's Pathophysiology: Concepts of Altered Health States. 9th ed. Philadelphia: Lippincott Williams & Wilkins, Chapter 41: Disorders of Renal Function, p. 1100.

Question 6: (see full question)

Prior to undergoing diagnostic testing with contrast, it is recommended that older adult clients have their creatinine level checked. The rationale for this is to ensure the client:

You selected:

Will not undergo an acute kidney injury by decreasing renal blood flow

Correct Explanation:

Some drugs, such as diuretics, high molecular weight radiocontrast media, the immunosuppressive drugs cyclosporine and tacrolimus, and the nonsteroidal anti-inflammatory drugs (NSA ... (more)

Reference:

Grossman, S. & Porth, C.M. Porth's Pathophysiology: Concepts of Altered Health States, 9th ed., Philadelphia: Lippincott Williams & Wilkins, 2014, Chapter 41, Disorders of Renal Function, p. 1107.

Question 7:

A client with a history of chronic pyelonephritis has been admitted several times with recurrent bacterial infection of the urinary tract. The nurse should anticipate educating this client with regard to which common treatment regimen?

(see full question)

You selected:

Increase intake of cranberry juice to 2 L/day.

Incorrect Correct response:

Continue taking antibiotics for full 10 to 14 days even if symptoms of infection disappear.

Explanation:

Chronic pyelonephritis involves a recurrent or persistent bacterial infection superimposed on urinary tract obstruction, urine reflux, or both. Chronic obstructive pyelonephritis c ... (more)


Reference:

Grossman, S. & Porth, C.M. Porth's Pathophysiology: Concepts of Altered Health States, 9th ed., Philadelphia: Lippincott Williams & Wilkins, 2014, Chapter 41, Disorders of Renal Function, p. 1107.

Question 8:

Which of the following client clinical manifestations most clearly suggests a need for diagnostic testing to rule out renal cell carcinoma?

(see full question)

You selected:

Hematuria

Correct Explanation:

Presenting features of renal cancer include hematuria, flank pain, and presence of a palpable flank mass. Gross or microscopic hematuria, which occurs in more than 50% of cases, is ... (more)

Reference:

Grossman S, Porth CM (2014). Porth’s Pathophysiology: Concepts of Altered Health States. 9th ed. Philadelphia: Lippincott Williams & Wilkins, Chapter 41, Disorders of Renal Function, p. 1109.

Question 9:

Which of the following is a nonsurgical method of treatment for renal calculi (kidney stones)?

(see full question)

You selected:

Extracorporeal shock wave lithotripsy (ESWL)

Correct Explanation:

ESWL is a nonsurgical treatment that uses sound waves, laser, or dry shock wave energy to break apart the stones. All of the other procedures are surgical in nature.

Reference:

Grossman S, Porth CM (2014). Porth’s Pathophysiology: Concepts of Altered Health States. 9th ed. Philadelphia: Lippincott Williams & Wilkins, Chapter 41, Disorders of Renal Function, p. 1193.

Question 10:

Which of the following assessment findings would lead the nurse to suspect


(see full question)

the client has nephrotic syndrome?

You selected:

Proteinuria and generalized edema

Correct Explanation:

The nephrotic syndrome is characterized by massive proteinuria and lipiduria, along with an associated hypoalbuminemia, generalized edema, and hyperlipidemia.

Question 1:

The nurse is planning care for a patient with a urinary tract obstruction. The nurse includes assessment for which of the following possible complications?

(see full question)

You selected:

Increased blood pressure

Correct Explanation:

Urinary tract obstruction can lead to hypertension related to increased renin secretion. The urine output would be decreased and not diluted.

Reference:

Grossman S, Porth CM (2014). Porth’s Pathophysiology: Concepts of Altered Health States. 9th ed. Philadelphia: Lippincott Williams & Wilkins, Chapter 41, Disorders of Renal Function, p. 1089.

Question 2:

Which one of the following is the usual cause of acute pyelonephritis?

(see full question)

You selected:

Infection

Correct Explanation:

Gram-negative bacteria, including Escherichia coli and Proteus, Klebsiella, Enterobacter, and Pseudomonas species, are the most common causative agents for acute pyelonephritis.

Reference:

Grossman S, Porth CM (2014). Porth’s Pathophysiology: Concepts of Altered Health States. 9th ed. Philadelphia: Lippincott Williams & Wilkins, Chapter 41, Disorders of Renal Function, p. 1106.


(see full question)

A nurse is assessing a patient's risk for developing a hospital acquired urniary tract infection (UTI) and determines that which of following places the patient at greatest risk?

You selected:

indwelling urinary catheter

Question 3:

Correct Explanation:

Intrumentation and urinary catheterization are the most common predisposing factors for hospital acquired (nosocomial) urinary tract infections. The other items increase risk fact ... (more)

Reference:

Grossman S, Porth CM (2014). Porth’s Pathophysiology: Concepts of Altered Health States. 9th ed. Philadelphia: Lippincott Williams & Wilkins, Chapter 41, Disorders of Renal Function, p. 1095.

Question 4:

Acute pyelonephritis is a result of which of the following?

(see full question)

You selected:

Bacterial infection

Correct Explanation:

Acute pyelonephritis represents a bacterial infection of the upper urinary tract, specifically the kidney parenchyma and renal pelvis. Gram-negative bacteria, including Escherichia ... (more)

Reference:

Grossman S, Porth CM (2014). Porth’s Pathophysiology: Concepts of Altered Health States. 9th ed. Philadelphia: Lippincott Williams & Wilkins, Chapter 41, Disorders of Renal Function, p. 1094.

Question 5: (see full question)

While taking a history from an adult client newly diagnosed with renal cell cancer, the nurse can associate which of the following high-risk factors with the development of this cancer?

You selected:

Heavy smoking

Correct Explanation:

Epidemiologic evidence suggests a correlation between heavy smoking and kidney cancer. Obesity also is a risk factor; particularly in women. The risk of renal cell carcinoma also i ... (more)


Reference:

Grossman S, Porth CM (2014). Porth’s Pathophysiology: Concepts of Altered Health States. 9th ed. Philadelphia: Lippincott Williams & Wilkins, Chapter 41, Disorders of Renal Function, p. 1109.

Question 6: (see full question)

An elderly female client has been hospitalized for the treatment of acute pyelonephritis. Which of the following characteristics of the client is most likely implicated in the etiology of her current health problem? The client:

You selected:

Recently had a urinary tract infection

Correct Explanation:

There are two routes by which bacteria can gain access to the kidney: ascending infection from the lower urinary tract and through the bloodstream. Ascending infection from the low ... (more)

Reference:

Grossman S, Porth CM (2014). Porth’s Pathophysiology: Concepts of Altered Health States. 9th ed. Philadelphia: Lippincott Williams & Wilkins, Chapter 41, Disorders of Renal Function, p. 1106.

Question 7:

Which of the following statements about the use of angiotensin-converting enzyme inhibitor medications and autosomal recessive polycystic kidney disease (ARPKD) is accurate?

(see full question)

You selected:

ACE inhibitors may interrupt the renin-angiotensin-aldosterone system to reduce renal vasoconstriction.

Correct Explanation:

In addition to increasing water intake to decrease vasopressin levels, the angiotensin-converting enzyme (ACE) inhibitors or angiotensin II receptor blockers (ARBs) may be used to ... (more)

Reference:

Grossman, S. & Porth, C.M. Porth's Pathophysiology: Concepts of Altered Health States, 9th ed., Philadelphia: Lippincott Williams & Wilkins, 2014, Chapter 39, Disorders of Fluid and Electrolyte Balance, p. 1031.


(see full question)

A patient who has had an intestinal bypass has developed a kidney stone. Which of the following types of kidney stone does the nurse recognize that this patient will most likely be treated for?

You selected:

Uric acid

Question 8:

Incorrect Correct response:

Calcium

Explanation:

Most kidney stones are calcium stones-calcium oxalate, calcium phosphate, or a combination of the two materials. Patients who have had intestinal bypass surgery are at a higher ris ... (more)

Reference:

Grossman S, Porth CM (2014). Porth’s Pathophysiology: Concepts of Altered Health States. 9th ed. Philadelphia: Lippincott Williams & Wilkins, Chapter 41, Disorders of Renal Function, p. 1091.

Question 9: (see full question)

The initiating event in the development of nephrotic syndrome is a derangement in the glomerular membrane that causes increased permeability to:

You selected:

plasma proteins.

Correct Explanation:

The initiating event in the development of nephrotic syndrome is a derangement in the glomerular membrane that causes increased permeability to plasma proteins. Some of nephrotic g ... (more)

Reference:

Grossman S, Porth CM (2014). Porth’s Pathophysiology: Concepts of Altered Health States. 9th ed. Philadelphia: Lippincott Williams & Wilkins, Chapter 41, Disorders of Renal Function, p. 1101.

Question 10:

A child has been brought to an urgent care clinic. The parents state that the child is "not making water." When taking a history, the nurse learns the child had a sore throat about 1 week ago but seems to have gotten over it. "We [parents] only had to give antibiotics for 3 days for the throat to be better." The nurse should suspect the child has developed:

(see full question)


You selected:

acute postinfectious glomerulonephritis.

Correct Explanation:

The classic case of poststreptococcal glomerulonephritis follows a streptococcal infection by approximately 7 to 12 days: the time needed for the development of antibodies. The pri ... (more)

Question 1:

A client has been given the diagnosis of diffuse glomerulonephritis. They ask the nurse what diffuse means. The nurse responds:

(see full question)

You selected:

All glomeruli and all parts of the glomeruli are involved.

Correct Explanation:

Glomerular changes can be diffuse, involving all glomeruli and all parts of the glomeruli; focal, meaning only some of the glomeruli are affected; segmental, involving only a certa ... (more)

Reference:

Grossman, S. & Porth, C.M. Porth's Pathophysiology: Concepts of Altered Health States, 9th ed., Philadelphia: Lippincott Williams & Wilkins, 2014, Chapter 41, Disorders of Renal Function, p. 1100.

Question 2:

Which of the following substances would not be found in glomerular filtrate?

(see full question)

You selected:

Protein

Correct Explanation:

The glomerular filtrate has a chemical composition similar to plasma which contains sodium, potassium and water, but it contains no proteins because large molecules do not readily ... (more)

Reference:

Grossman S, Porth CM (2014). Porth’s Pathophysiology: Concepts of Altered Health States. 9th ed. Philadelphia: Lippincott Williams & Wilkins, Chapter 38, Disorders of Renal Function Fluid and Electrolytes, p. 1003.


(see full question)

A patient is to receive a radiocontrast media as part of a diagnostic scan. Which of the following is intended to reduce the nephrotoxic effects of the radiocontrast media?

You selected:

Increasing the normal saline intravenous infusion rate to 125 mL/hour

Question 3:

Correct Explanation:

Some drugs, such as high-molecular-weight radiocontrast media, the immunosuppressive drugs cyclosporine and tacrolimus and nonsteroidal antiinflammatory drugs can cause acute prer ... (more)

Reference:

Grossman S, Porth CM (2014). Porth’s Pathophysiology: Concepts of Altered Health States. 9th ed. Philadelphia: Lippincott Williams & Wilkins, Chapter 41, Disorders of Renal Function, p. 1108.

Question 4: (see full question)

Wilms tumor is a tumor of childhood. It is usually an encapsulated mass occurring in any part of the kidney. What are the common presenting signs of a Wilms tumor?

You selected:

Hypotension and a large abdominal mass

Incorrect Correct response:

Large asymptomatic abdominal mass and hypertension

Explanation:

The common presenting signs of a Wilms tumor are a large asymptomatic abdominal mass and hypertension. The tumor is often discovered inadvertently, and it is not uncommon for ... (more)

Reference:

Grossman S, Porth CM (2014). Porth's Pathophysiology: Concepts of Altered Health States. 9th ed. Philadelphia: Lippincott Williams & Wilkins, Chapter 41: Disorders of Renal Function, p. 1109.

Question 5:

An newborn has been diagnosed with renal agenesis. The nurse caring for the mother mentions some of the classic signs/symptoms of this disease. Which characteristic should the nurse mention to the parents as it relates to this disease? Select all that apply.

(see full question)


You selected:

• Epicanthic folds (skin of the upper eyelid) are present • Low-set ears

Incorrect Correct response:

• Epicanthic folds (skin of the upper eyelid) are present • Low-set ears • Receding chin

Explanation:

Newborns with renal agenesis often have characteristic facial features, called Potter syndrome, that are caused by fetal compression due to a marked reduction in amniotic fluid lev ... (more)

Reference:

Grossman S, Porth CM (2014). Porth’s Pathophysiology: Concepts of Altered Health States. 9th ed. Philadelphia: Lippincott Williams & Wilkins, Chapter 41, Disorders of Renal Function, p. 1084.

Question 6:

What commonly causes chronic pyelonephritis, a significant cause of renal failure?

(see full question)

You selected:

Renal hyperplasia

Incorrect Correct response:

Intrarenal reflux

Explanation:

Reflux, the most common cause of chronic pyelonephritis, results from superimposition of infection on congenital vesicoureteral reflux or intrarenal reflux. Reflux may be unilatera ... (more)

Reference:

Grossman S, Porth CM (2014). Porth’s Pathophysiology: Concepts of Altered Health States. 9th ed. Philadelphia: Lippincott Williams & Wilkins, Chapter 41, Disorders of Renal Function, p. 1107.

Question 7: (see full question)

A nurse is caring for a patient who has been diagnosed with kidney colic and has not yet passed the stone. Which of the following interventions should the nurse be sure to include when planning the care for this patient?

You selected:

Strain the urine.

Correct


Explanation:

All urine should be strained during an attack in the hope of retrieving the stone for chemical analysis and determination of type. This information, along with a careful history an ... (more)

Reference:

Grossman S, Porth CM (2014). Porth’s Pathophysiology: Concepts of Altered Health States. 9th ed. Philadelphia: Lippincott Williams & Wilkins, Chapter 41, Disorders of Renal Function, p. 1092.

Question 8:

Which clinical manifestations would you expect to see in an infant diagnosed with autosomal recessive polycystic kidney disease (ARPKD)?

(see full question)

You selected:

Bilateral flank masses and impaired lung development

Correct Explanation:

Clinical manifestations of the typical infant with ARPKD include bilateral flank masses, accompanied by severe renal failure, signs of impaired lung development, and variable degre ... (more)

Reference:

Grossman S, Porth CM (2014). Porth’s Pathophysiology: Concepts of Altered Health States. 9th ed. Philadelphia: Lippincott Williams & Wilkins, Chapter 41, Disorders of Renal Function, p. 1087.

Question 9:

The nurse reviews the lab results for a patient who has advanced autosomal dominant polycystic kidney disease (ADPKD). The patient 's hemoglobin is 8.8 g/dL. The nurse suspects this lab value is related to which of the following causes?

(see full question)

You selected:

Reduced production of erythropoietin

Correct Explanation:

As ADPKD progresses, the nephrons reduce the production of erythropoietin (EPO). EPO is necessary for red blood cell production by bone marrow, so EPO deficiency causes anemia.

Reference:

Grossman S, Porth CM (2014). Porth’s Pathophysiology: Concepts of Altered Health States. 9th ed. Philadelphia: Lippincott Williams & Wilkins, Chapter 41,


Disorders of Renal Function, pp. 1086-1087.

Question 10: (see full question)

You selected:

A patient is being treated with colchicine (Colcrys) for pain in the big right toe. The patient begins to complain of severe right flank pain and is diagnosed with kidney stones. Which of the following types of kidney stones does the nurse recognize this patient is most likely affected by? Uric acid

Correct Explanation:

Uric acid stones develop in conditions of gout and high concentrations of uric acid in the urine; it accounts for about 7% of all stones.

Question 1:

Unilateral obstruction of the urinary tract may result in renin secretion, thereby leading to which of the following?

(see full question)

You selected:

Increased blood pressure

Correct Explanation:

In cases of unilateral obstruction of the kidney, renin secretion is enhanced, probably secondary to impaired renal blood flow. This would result in increased sodium retention, and ... (more)

Reference:

Grossman S, Porth CM (2014). Porth’s Pathophysiology: Concepts of Altered Health States. 9th ed. Philadelphia: Lippincott Williams & Wilkins, Chapter 41, Disorders of Renal Function, p. 1090.

Question 2: (see full question)

A client has a dilated renal pelvis due to obstruction of urine outflow from the kidney. The pressure of filtrate formation is damaging the renal structures. Which condition is most likely?

You selected:

Prostatic hyperplasia

Incorrect Correct response:

Hydronephrosis


Explanation:

Hydronephrosis refers to urine-filled dilation of the renal pelvis and calyces associated with progressive atrophy of the kidney due to obstruction of urine outflow.

Reference:

Grossman S, Porth CM (2014). Porth's Pathophysiology: Concepts of Altered Health States. 9th ed. Philadelphia: Lippincott Williams & Wilkins, Chapter 41: Disorders of Renal Function, p. 1089.

Question 3:

The nurse recognizes the most common cause of acute postinfectious glomerulonephritis as:

(see full question)

You selected:

A streptococcal infection 7 to 12 days prior to onset

Correct Explanation:

Acute postinfectious glomerulonephritis usually occurs after infection with certain strains of group A β-hemolytic streptococci and is caused by deposition of immune complexes ... (more)

Reference:

Grossman S, Porth CM (2014). Porth’s Pathophysiology: Concepts of Altered Health States. 9th ed. Philadelphia: Lippincott Williams & Wilkins, Chapter 41, Disorders of Renal Function, p. 1100

Question 4:

Manifestations of polycystic kidney disease include which of the following?

(see full question)

You selected:

Increase in kidney size bilaterally

Correct Explanation:

The kidneys are usually enlarged in people with autosomal dominant polycystic kidney disease and may achieve enormous sizes. The external contours of the kidneys are distorted by n ... (more)

Reference:

Grossman S, Porth CM (2014). Porth’s Pathophysiology: Concepts of Altered Health States. 9th ed. Philadelphia: Lippincott Williams & Wilkins, Chapter 41, Disorders of Renal Function, pp. 1086 and 1087.


Question 5: (see full question)

You selected:

Hospitalized patients are at the greatest risk of developing septicemia related to which procedure? Acute viral cystitis

Incorrect Correct response:

Catheter-associated bacteriuria

Explanation:

Catheter-associated bacteriuria remains the most frequent cause of gramnegative septicemia in hospitalized patients.

Reference:

Grossman S, Porth CM (2014). Porth’s Pathophysiology: Concepts of Altered Health States. 9th ed. Philadelphia: Lippincott Williams & Wilkins, Chapter 41, Disorders of Renal Function, p. 1095.

Question 6:

A school nurse is teaching a group of fourth-grade girls about personal hygiene. Important teaching points aimed at reducing the incidence of urinary tract infection (UTI) include which of the following? Select all that apply.

(see full question)

You selected:

• Careful hand washing • Wiping from front to back after a bowel movement • Avoiding bubble baths

Correct Explanation:

Most uncomplicated lower UTIs are caused by Escherichia coli. It is important to wipe from front to back to prevent introducing this pathogen to the urinary tract from the rectal ... (more)

Reference:

Grossman S, Porth CM (2014). Porth’s Pathophysiology: Concepts of Altered Health States. 9th ed. Philadelphia: Lippincott Williams & Wilkins, Chapter 41, Disorders of Renal Function, pp. 1094-1095, 1097.

Question 7:

A nurse observes that a patient's urine is cola colored and considers which of the following as a possible reason?

(see full question)

You selected:

The patient's urine contains material from the degradation of red blood cells.


Correct Explanation:

When red blood cells degrade in the urine, urine may appear cola colored.

Reference:

Grossman S, Porth CM (2014). Porth’s Pathophysiology: Concepts of Altered Health States. 9th ed. Philadelphia: Lippincott Williams & Wilkins, Chapter 41, Disorders of Renal Function, p. 1100.

Question 8:

A nurse is caring for a child with Wilms tumor, stage I. Which of the following can be said regarding this diagnosis? Select all that apply.

(see full question)

You selected:

• Prognosis is good with treatment. • The tumor is limited to the kidney and can be excised with the capsular surface intact.

Correct Explanation:

Long-term survival rates have increased to 90% for Wilms tumor stages I through III. The tumors usually are staged using the National Wilms' Tumor Study Group classification: stag ... (more)

Reference:

Grossman S, Porth CM (2014). Porth’s Pathophysiology: Concepts of Altered Health States. 9th ed. Philadelphia: Lippincott Williams & Wilkins, Chapter 41, Disorders of Renal Function, pp. 1108-1109.

Question 9:

A patient had been diagnosed with a simple cyst of the kidney and is presenting with symptoms. Which does the nurse anticipates the patient will display? Select all that apply.

(see full question)

You selected:

• Compromised renal function • Hypertension • Infection

Incorrect Correct response:

• Hematuria • Hypertension • Infection

Explanation:

Most simple cysts do not produce signs or symptoms or compromise renal function. When symptomatic, they may cause flank pain, hematuria, infection, and hypertension related to isch ... (more)


Reference:

Grossman, S. & Porth, C.M. Porth's Pathophysiology: Concepts of Altered Health States, 9th ed., Philadelphia: Lippincott Williams & Wilkins, 2014, Chapter 41, Disorders of Renal Function, p. 1087.

Question 10:

The nurse is evaluating her patient's risk for an urinary tract infection. Which of the following patients has the highest risk?

(see full question)

You selected:

Patient with obstructed urinary outflow from the kidney

Correct Explanation:

Factors that contribute to the development of ascending infections of the urinary tract are outflow obstruction, catheterization and urinary instrumentation, vesicoureteral reflux, ... (more)

Question 1:

The nurse should assess which of the following when assessing for the initial presence of edema in a patient with nephrotic syndrome?

(see full question)

You selected:

Eyelids

Incorrect Correct response:

Lower extremities

Explanation:

Initially the edema caused by nephritic syndrome presents in the dependent parts of the body, such as the lower extremities, but becomes more generalized as the disease progresses.

Reference:

Grossman S, Porth CM (2014). Porth’s Pathophysiology: Concepts of Altered Health States. 9th ed. Philadelphia: Lippincott Williams & Wilkins, Chapter 41, Disorders of Renal Function, p. 1101.

Question 2: (see full question)

One form of renal tubular acidosis (RTA) results from aldosterone deficiency or resistance to its action, which leads to impaired reabsorption of which of the following electrolytes?

You selected:

Sodium


Correct Explanation:

One type of RTA results from aldosterone deficiency or resistance to its action that leads to impaired reabsorption of sodium ions with decreased elimination of hydrogen and potass ... (more)

Reference:

Grossman S, Porth CM (2014). Porth’s Pathophysiology: Concepts of Altered Health States. 9th ed. Philadelphia: Lippincott Williams & Wilkins, Chapter 41, Disorders of Renal Function, p. 1105.

Question 3:

What commonly causes chronic pyelonephritis, a significant cause of renal failure?

(see full question)

You selected:

Intrarenal reflux

Correct Explanation:

Reflux, the most common cause of chronic pyelonephritis, results from superimposition of infection on congenital vesicoureteral reflux or intrarenal reflux. Reflux may be unilatera ... (more)

Reference:

Grossman S, Porth CM (2014). Porth’s Pathophysiology: Concepts of Altered Health States. 9th ed. Philadelphia: Lippincott Williams & Wilkins, Chapter 41, Disorders of Renal Function, p. 1107.

Question 4:

Which of the following are appropriate interventions in the care of a patient diagnosed with renal calculi? Select all that apply.

(see full question)

You selected:

• Straining the client's urine • Addressing the client's pain

Incorrect Correct response:

• Straining the client's urine • Addressing the client's pain • Keeping track of intake and output

Explanation:

One of the major manifestations of kidney stones is pain. Treatment includes relief of pain. All urine should be strained during an attack in the hope of retrieving the stone for ... (more)


Reference:

Grossman S, Porth CM (2014). Porth’s Pathophysiology: Concepts of Altered Health States. 9th ed. Philadelphia: Lippincott Williams & Wilkins, Chapter 41, Disorders of Renal Function, pp. 1091-1092.

Question 5:

A teenager has been admitted with the diagnosis of Alport Syndrome. The client has been passing large amounts of blood in the urine. Knowing that this disorder is inherited and associated with other abnormalities, the nurse should focus assessment on which area?

(see full question)

You selected:

Visual disturbances and hearing loss

Correct Explanation:

Children may initially present with heavy microscopic hematuria (large amount of blood on dipstick), followed by the development of proteinuria. Many, but not all, persons with Alp ... (more)

Reference:

Grossman S, Porth CM (2014). Porth’s Pathophysiology: Concepts of Altered Health States. 9th ed. Philadelphia: Lippincott Williams & Wilkins, Chapter 41, Disorders of Renal Function, p. 1103-1104.

Question 6:

Which are contributing factors of acute postinfectious glomerulonephritis? Select all that apply.

(see full question)

You selected:

• Occurs after infection of group A ?-hemolytic streptococci. • Antibodies develop in 2 to 5 days after infection.

Incorrect Correct response:

• Occurs after infection of group A ?-hemolytic streptococci. • May result from impetigo. • Primarily infects the pharynx.

Explanation:

The classic case of poststreptococcal glomerulonephritis follows a streptococcal infection by approximately 7 to 12 days—the time needed for the development of antibodies. T ... (more)

Reference:

Grossman, S. & Porth, C.M. Porth's Pathophysiology: Concepts of Altered


Health States, 9th ed., Philadelphia: Lippincott Williams & Wilkins, 2014, Chapter 41, Disorders of Renal Function, p. 1100.

Question 7: (see full question)

You selected:

The family asks the nurse what the usual treatment of focal segmental glomerulosclerosis entails. The nurse responds: corticosteroids.

Correct Explanation:

The disorder usually is treated with corticosteroids. Although kidney transplantation is the preferred treatment for end-stage kidney disease, focal segmental glomerulonephritis oc ... (more)

Reference:

Grossman S, Porth CM (2014). Porth’s Pathophysiology: Concepts of Altered Health States. 9th ed. Philadelphia: Lippincott Williams & Wilkins, Chapter 41, Disorders of Renal Function, p. 1102.

Question 8:

The edema that develops in persons with glomerulonephritis and nephrotic syndrome reflects which of the following?

(see full question)

You selected:

Salt and water retention

Correct Explanation:

Generalized edema, a hallmark of nephrosis, results from salt and water retention and a decrease in plasma colloid osmotic pressure due to loss of albumin in the urine. Glomerulone ... (more)

Reference:

Grossman S, Porth CM (2014). Porth’s Pathophysiology: Concepts of Altered Health States. 9th ed. Philadelphia: Lippincott Williams & Wilkins, Chapter 41, Disorders of Renal Function, pp. 1100-1101.

Question 9:

The nurse is caring for a patient with cystitis. Which of the following is most characteristic of the urine associated with cystitis?

(see full question)

You selected:

Oliguria


Incorrect Correct response:

Foul smell

Explanation:

The manifestations of cystitis include urine that is occasionally cloudy and foul smelling.

Reference:

Grossman S, Porth CM (2014). Porth’s Pathophysiology: Concepts of Altered Health States. 9th ed. Philadelphia: Lippincott Williams & Wilkins, Chapter 41, Disorders of Renal Function, p. 1095.

Question 10:

The form of polycystic kidney disease (PKD) that first manifests in the early infant period is most commonly characterized as which of the following?

(see full question)

You selected:

Autosomal recessive

Correct Explanation:

Autosomal recessive familial PKD is characterized by cystic dilatation of the cortical and medullary collecting tubules. It is rare compared with autosomal dominant polycystic dise ... (more)

Question 1: (see full question)

While taking a history from an adult client newly diagnosed with renal cell cancer, the nurse can associate which of the following high-risk factors with the development of this cancer?

You selected:

Heavy smoking

Correct Explanation:

Epidemiologic evidence suggests a correlation between heavy smoking and kidney cancer. Obesity also is a risk factor; particularly in women. The risk of renal cell carcinoma also i ... (more)

Reference:

Grossman S, Porth CM (2014). Porth’s Pathophysiology: Concepts of Altered Health States. 9th ed. Philadelphia: Lippincott Williams & Wilkins, Chapter 41, Disorders of Renal Function, p. 1109.

Question 2:

Which factor contributes to the development of polycystic kidney disease?

(see full question)


You selected:

Hereditary mutations in polycystin I and II

Correct Explanation:

Polycystic kidney disease is typically related to dominant or recessive hereditary mutations in polycystin. The other options are related to urinary tract infections. Some drugs, s ... (more)

Reference:

Grossman S, Porth CM (2014). Porth’s Pathophysiology: Concepts of Altered Health States. 9th ed. Philadelphia: Lippincott Williams & Wilkins, Chapter 41, Disorders of Renal Function, p. 1085-1086.

Question 3: (see full question)

The nurse is presenting an educational program about renal carcinoma. The nurse determines that the participants understand early signs of the disease when they identify which of the following as an early sign or symptom?

You selected:

hematuria

Incorrect Correct response:

None of the above

Explanation:

Kidney cancer is largely a silent disorder during its early stages, and symptoms usually denote advanced disease. Presenting features of kidney cancer include hematuria, flank pain ... (more)

Reference:

Grossman S, Porth CM (2014). Porth’s Pathophysiology: Concepts of Altered Health States. 9th ed. Philadelphia: Lippincott Williams & Wilkins, Chapter 41, Disorders of Renal Function, p. 1110

Question 4:

A nurse is caring for a patient with diabetic glomerulosclerosis. The analysis is reviewed for the presence of which of the following manifestations?

(see full question)

You selected:

Albumin

Correct Explanation:

The clinical manifestations of diabetic glomerulosclerosis are closely linked to those of diabetes. The increased glomerular filtration rate that occurs in people with early altera ... (more)


Reference:

Grossman S, Porth CM (2014). Porth’s Pathophysiology: Concepts of Altered Health States. 9th ed. Philadelphia: Lippincott Williams & Wilkins, Chapter 41, Disorders of Renal Function, p. 1104.

Question 5: (see full question)

A client has a dilated renal pelvis due to obstruction of urine outflow from the kidney. The pressure of filtrate formation is damaging the renal structures. Which condition is most likely?

You selected:

Hydronephrosis

Correct Explanation:

Hydronephrosis refers to urine-filled dilation of the renal pelvis and calyces associated with progressive atrophy of the kidney due to obstruction of urine outflow.

Reference:

Grossman S, Porth CM (2014). Porth's Pathophysiology: Concepts of Altered Health States. 9th ed. Philadelphia: Lippincott Williams & Wilkins, Chapter 41: Disorders of Renal Function, p. 1089.

Question 6:

Select the manifestations of renal cell carcinoma. Select all that apply.

(see full question)

You selected:

• Hematuria • Palpable flank mass • Often silent in the early stages

Correct Explanation:

Kidney cancer is largely a silent disorder during its early stages, and symptoms usually denote advanced disease. Presenting features include hematuria, flank pain, and presence of ... (more)

Reference:

Grossman, S. & Porth, C.M. Porth's Pathophysiology: Concepts of Altered Health States, 9th ed., Philadelphia: Lippincott Williams & Wilkins, 2014, Chapter 41, Disorders of Renal Function, p. 1109.


(see full question)

The most recent assessment of a client with a diagnosis of type 1 diabetes indicates a heightened risk of diabetic nephropathy. Which of the following assessment findings is most suggestive of this increased risk?

You selected:

Microalbuminuria

Question 7:

Correct Explanation:

The increased glomerular filtration rate (GFR) that occurs in persons with early alterations in renal function is associated with microalbuminuria, which is an important predictor ... (more)

Reference:

Grossman S, Porth CM (2014). Porth’s Pathophysiology: Concepts of Altered Health States. 9th ed. Philadelphia: Lippincott Williams & Wilkins, Chapter 41, Disorders of Renal Function, p. 1104.

Question 8:

Hospitalized patients are at the greatest risk of developing septicemia related to which procedure?

(see full question)

You selected:

Catheter-associated bacteriuria

Correct Explanation:

Catheter-associated bacteriuria remains the most frequent cause of gramnegative septicemia in hospitalized patients.

Reference:

Grossman S, Porth CM (2014). Porth’s Pathophysiology: Concepts of Altered Health States. 9th ed. Philadelphia: Lippincott Williams & Wilkins, Chapter 41, Disorders of Renal Function, p. 1095.

Question 9:

Which one of the following would the nurse see as being liable to cause the most serious long-term problems?

(see full question)

You selected:

Unilateral renal agenesis

Incorrect Correct response:

Polycystic kidney disease


Explanation:

Agenesis refers to failure of an organ to develop at all. The other kidney usually undergoes compensatory hypertrophy and performs the function of the missing kidney. Most simple c ... (more)

Reference:

Grossman S, Porth CM (2014). Porth’s Pathophysiology: Concepts of Altered Health States. 9th ed. Philadelphia: Lippincott Williams & Wilkins, Chapter 25, Disorders of Renal Function, pp. 1084, 1086, 1087.

Question 10:

A child is recovering from a bout with group A β-hemolytic Streptococcus infection. They return to the clinic a week later complaining of decrease in urine output with puffiness and edema noted in the face and hands. The health care provider suspects the child has developed:

(see full question)

You selected:

Acute postinfectious glomerulonephritis

Correct Explanation:

The classic case of poststreptococcal glomerulonephritis follows a streptococcal infection by approximately 7 to 12 days—the time needed for the development of antibodies. O ... (more)

Question 1: (see full question)

The GFR is considered to be the best measure of renal function. What is used to estimate the GFR?

You selected:

Serum creatinine

Correct Explanation:

In clinical practice, GFR is usually estimated using the serum creatinine concentration. The other answers are not used to estimate the GFR.

Reference:

Grossman S, Porth CM (2014). Porth's Pathophysiology: Concepts of Altered Health States. 9th ed. Philadelphia: Lippincott Williams & Wilkins, Chapter 42: Acute Renal Failure and Chronic Kidney Disease, p. 1118.

Question 2:

Bruising of the skin is a manifestation of bleeding disorders in patients with


(see full question)

CKD.

You selected:

True

Correct Explanation:

Rationale: Bleeding disorders manifested by persons with CKD include epistaxis, menorrhagia, gastrointestinal bleeding, and bruising of skin and subcutaneous tissues.

Reference:

Grossman S, Porth CM (2014). Porth’s Pathophysiology: Concepts of Altered Health States. 9th ed. Philadelphia: Lippincott Williams & Wilkins, Chapter 42, Acute Renal Injury and Chronic Kidney Disease, p. 1122.

Question 3:

Several urine tests can be useful in establishing a diagnosis of acute renal failure (ARF). The nurse must consider that fractional excretion of sodium can be particularly affected by administration of which of the following types of drugs?

(see full question)

You selected:

Diuretics

Correct Explanation:

Diuretics, which directly affect renal excretion of sodium, can alter the fractional excretion of sodium. The other drug types listed do not affect this parameter of renal function ... (more)

Reference:

Grossman S, Porth CM (2014). Porth’s Pathophysiology: Concepts of Altered Health States. 9th ed. Philadelphia: Lippincott Williams & Wilkins, Chapter 42, Acute Renal Injury and Chronic Kidney Disease, p. 1116.

Question 4: (see full question)

The nurse is caring for a patient who has had acute blood loss from ruptured esophageal varices. Which of the following does the nurse recognize is an early sign of prerenal failure?

You selected:

Baseline urine output of 50 mL/hr that is now 10 mL/hr

Correct Explanation:

The kidney normally responds to a decrease in the glomerular filtration rate


with a decrease in urine output. Thus, an early sign of prerenal failure is a sharp decrease in urine o ... (more)

Reference:

Grossman S, Porth CM (2014). Porth’s Pathophysiology: Concepts of Altered Health States. 9th ed. Philadelphia: Lippincott Williams & Wilkins, Chapter 42, Acute Renal Injury and Chronic Kidney Disease, p. 1114.

Question 5:

Chronic kidney disease (CKD) can provoke many cardiovascular changes. The nurse will monitor for which of the following in a client with CKD? Select all that apply.

(see full question)

You selected:

• Hypertension • Congestive heart failure

Incorrect Correct response:

• Congestive heart failure • Hypertension • Impaired platelet function • Pericarditis

Explanation:

Hypertension often occurs early in CKD, due to several factors including increased blood volume. Congestive heart failure and pericarditis can occur with more advanced CKD. Impaire ... (more)

Reference:

Grossman S, Porth CM (2014). Porth’s Pathophysiology: Concepts of Altered Health States. 9th ed. Philadelphia: Lippincott Williams & Wilkins, Chapter 42, Acute Renal Injury and Chronic Kidney Disease, p. 1122.

Question 6: (see full question)

Clients with renal transplants must be protected from transplant rejection. To this end, the nurse might monitor for a change in levels of which of the following?

You selected:

Urinary glucose concentration

Incorrect Correct response:

Neutrophil-gelatin-associated lipocalin (NGAL)

Explanation:

NGAL is found in many organs including kidneys. Elevated levels in blood and urine can predict transplant impairment. The other answer selections are less


specific for transplant i ... (more)

Reference:

Grossman S, Porth CM (2014). Porth’s Pathophysiology: Concepts of Altered Health States. 9th ed. Philadelphia: Lippincott Williams & Wilkins, Chapter 42, Acute Renal Injury and Chronic Kidney Disease, p. 1116.

Question 7: (see full question)

A drug abuser was found unconscious after shooting up heroin 2 days prior. Because of the pressure placed on the hip and arm, the client has developed rhabdomyolysis. The nurse knows this can:

You selected:

Obstruct the renal tubules with myoglobin and damage tubular cells

Correct Explanation:

Myoglobin normally is not found in the serum or urine. It has a low molecular weight; if it escapes into the circulation, it is rapidly filtered in the glomerulus. A life-threateni ... (more)

Reference:

Grossman, S. & Porth, C.M. Porth's Pathophysiology: Concepts of Altered Health States, 9th ed., Philadelphia: Lippincott Williams & Wilkins, 2014, Chapter 42, Acute Renal Injury and Chronic Kidney Disease, p. 1115.

Question 8:

The primary care provider for a newly admitted hospital client has added the glomerular filtration rate (GFR) to the blood work scheduled for this morning. The client's GFR results return as 50 mL/minute/1.73 m2. The nurse explains to the client that this result represents:

(see full question)

You selected:

A loss of over half the client's normal kidney function

Correct Explanation:

In clinical practice, GFR is usually estimated using the serum creatinine concentration. A GFR below 60 mL/minute/1.73 m2 represents a loss of one half or more of the level of normal ... (more)

Reference:

Grossman, S. & Porth, C.M. Porth's Pathophysiology: Concepts of Altered Health States, 9th ed., Philadelphia: Lippincott Williams & Wilkins, 2014, Chapter 42, Acute Renal Injury and Chronic Kidney Disease, p. 1118.


Question 9: (see full question)

You selected:

A 35-year-old client is diagnosed with acute kidney injury (AKI) and is started on hemodialysis. The client is concerned with the diagnosis and wants to know what to expect in the progression of this disorder. Which statement best addresses the client’s concern? "Acute kidney injury is abrupt in onset and often reversible if recognized early and treated appropriately."

Correct Explanation:

AKI is the result of a rapid decline in kidney function that increases blood levels of nitrogenous wastes. Once the underlying cause is treated, AKI is potentially reversible if th ... (more)

Reference:

Grossman S, Porth CM (2014). Porth’s Pathophysiology: Concepts of Altered Health States. 9th ed. Philadelphia: Lippincott Williams & Wilkins, Chapter 42, Acute Renal Injury and Chronic Kidney Disease, p. 1116.

Question 10:

A client with a long-standing diagnosis of chronic kidney disease has been experiencing increasing fatigue, lethargy, and activity intolerance in recent weeks. His care team has established that his GFR remains at a low, but stable, level. Which of the following assessments is most likely to inform a differential diagnosis?

(see full question)

You selected:

Blood work for hemoglobin, red blood cells, and hematocrit

Correct Explanation:

Anemia is a frequent, and debilitating, consequence of CKD. The anemia may be due to chronic blood loss, hemolysis, bone marrow suppression due to retained uremic factors, and decr ... (more)

Question 1:

A patient with chronic kidney disease (CKD) will be managed with peritoneal dialysis. The nurse will give the patient which of the following descriptions of this type of dialysis?

(see full question)

You selected:

Treatment involves the introduction into the peritoneum of a sterile dialyzing solution, which is drained after a specified time.


Correct Explanation:

In peritoneal dialysis, a sterile dialyzing solution is instilled into the peritoneum through a catheter over approximately 10 minutes. The solution remains ("dwells") in the perit ... (more)

Reference:

Grossman S, Porth CM (2014). Porth’s Pathophysiology: Concepts of Altered Health States. 9th ed. Philadelphia: Lippincott Williams & Wilkins, Chapter 42, Acute Renal Injury and Chronic Kidney Disease, p. 1126.

Question 2:

A patient with chronic kidney disease (CKD) will begin hemodialysis. The nurse will recommend which of the following diets?

(see full question)

You selected:

Diet low in proteins but including eggs and lean meat

Correct Explanation:

A low protein diet with at least 50% of the proteins that are rich in essential amino acids, such as eggs, lean meat and milk, is recommended for CKD patients. This will prevent pr ... (more)

Reference:

Grossman S, Porth CM (2014). Porth’s Pathophysiology: Concepts of Altered Health States. 9th ed. Philadelphia: Lippincott Williams & Wilkins, Chapter 42, Acute Renal Injury and Chronic Kidney Disease, p. 1127.

Question 3:

A patient in renal failure has marked decrease in renal blood flow caused by hypovolemia, caused by gastrointestinal bleeding. The nurse is aware that this form of renal failure can be reversed if the bleeding is under control. Which of the following forms of acute renal injury does this patient have?

(see full question)

You selected:

Prerenal failure

Correct Explanation:

Prerenal failure, the most common form of acute renal failure, is characterized by a marked decrease in renal blood flow. It is reversible if the cause of the decreased renal blood ... (more)

Reference:

Grossman S, Porth CM (2014). Porth’s Pathophysiology: Concepts of Altered


Health States. 9th ed. Philadelphia: Lippincott Williams & Wilkins, Chapter 42, Acute Renal Injury and Chronic Kidney Disease, p. 1113.

Question 4: (see full question)

Acute renal failure occurs at a high rate in seriously ill people who are in intensive care units. What is the most common indicator of acute renal failure?

You selected:

Azotemia and a decrease in the GFR

Correct Explanation:

The most common indicator of acute renal failure is azotemia, an accumulation of nitrogenous wastes (urea nitrogen, uric acid, and creatinine) in the blood and a decrease in ... (more)

Reference:

Grossman S, Porth CM (2014). Porth's Pathophysiology: Concepts of Altered Health States. 9th ed. Philadelphia: Lippincott Williams & Wilkins, Chapter 42: Acute Renal Failure and Chronic Kidney Disease, p. 1113.

Question 5:

The nurse is reviewing the diagnosis of four male clients. Select the diagnosis that places the clients at risk for developing postrenal kidney failure.

(see full question)

You selected:

Intratubular obstruction

Incorrect Correct response:

Prostatic hyperplasia

Explanation:

The most common cause of postrenal kidney failure is prostatic hyperplasia. Postrenal failure results from conditions that obstruct urine outflow. The obstruction can occur in the ... (more)

Reference:

Grossman S, Porth CM (2014). Porth’s Pathophysiology: Concepts of Altered Health States. 9th ed. Philadelphia: Lippincott Williams & Wilkins, Chapter 42, Acute Renal Injury and Chronic Kidney Disease, p. 1114.

Question 6: (see full question)

Chronic kidney disease impacts many systems in the body. What is the


number one hematologic disorder caused by CKD? You selected:

Anemia

Correct Explanation:

The number one hematologic disorder that accompanies CKD is anemia. The other answers are incorrect.

Reference:

Grossman S, Porth CM (2014). Porth's Pathophysiology: Concepts of Altered Health States. 9th ed. Philadelphia: Lippincott Williams & Wilkins, Chapter 42: Acute Renal Failure and Chronic Kidney Disease, p. 1122.

Question 7:

An 86-year-old female client has been admitted to the hospital for the treatment of dehydration and hyponatremia after she curtailed her fluid intake to minimize urinary incontinence. The client's admitting laboratory results are suggestive of prerenal failure. The nurse should be assessing this client for which of the following early signs of prerenal injury?

(see full question)

You selected:

Sharp decrease in urine output

Correct Explanation:

Dehydration and its consequent hypovolemia can result in acute renal failure that is prerenal in etiology. The kidney normally responds to a decrease in GFR with a decrease in urin ... (more)

Reference:

Grossman, S. & Porth, C.M. Porth's Pathophysiology: Concepts of Altered Health States, 9th ed., Philadelphia: Lippincott Williams & Wilkins, 2014, Chapter 42, Acute Renal Injury and Chronic Kidney Disease, p. 1113.

Question 8: (see full question)

The primary care provider for a newly admitted hospital patient has added the glomerular filtration rate (GFR) to the blood work scheduled for this morning. The patient's GFR will be extrapolated from serum levels of:

You selected:

creatinine.

Correct Explanation:

In clinical practice, GFR is usually estimated using the serum creatinine concentration. The presence of myoglobin or large amounts of protein in the


urine is suggestive of renal f ... (more)

Reference:

Grossman S, Porth CM (2014). Porth’s Pathophysiology: Concepts of Altered Health States. 9th ed. Philadelphia: Lippincott Williams & Wilkins, Chapter 42, Acute Renal Injury and Chronic Kidney Disease, p. 1118.

Question 9:

The nurse is instructing a patient with advanced kidney disease (AKD) about a dietary regimen. Which of the following restrictions should the nurse be sure to include in the treatment plan to decrease the progress of renal impairment in people with AKD?

(see full question)

You selected:

Dietary protein

Correct Explanation:

Restriction of dietary proteins may decrease the progress of renal impairment in people with advanced renal disease. Proteins are broken down to form nitrogenous wastes, and reduci ... (more)

Reference:

Grossman S, Porth CM (2014). Porth’s Pathophysiology: Concepts of Altered Health States. 9th ed. Philadelphia: Lippincott Williams & Wilkins, Chapter 42, Acute Renal Injury and Chronic Kidney Disease, p. 1127.

Question 10: (see full question)

A client with a diagnosis of chronic kidney disease (CKD) may require the administration of which of the following drugs to treat coexisting conditions that carry a high mortality?

You selected:

Antihypertensive medications

Correct Explanation:

Hypertension is a common result of CKD, and the mechanisms that produce hypertension in CKD include increased vascular volume, elevation of peripheral vascular resistance, decrease ... (more)

Question 1:

A patient is to begin peritoneal dialysis. The nurse knows that complications of peritoneal dialysis include which of the following? Select all that apply.

(see full question)

You selected:

• Hyperglycemia


• Edema • Catheter-related infection • Pain Incorrect Correct response:

• Catheter-related infection • Hyperglycemia • Dehydration

Explanation:

The most serious complication of peritoneal dialysis is infection, which can occur at the catheter exit site, in the subcutaneous tunnel, or in the peritoneal cavity. Hyperglycemia ... (more)

Reference:

Grossman S, Porth CM (2014). Porth’s Pathophysiology: Concepts of Altered Health States. 9th ed. Philadelphia: Lippincott Williams & Wilkins, Chapter 42, Acute Renal Injury and Chronic Kidney Disease, p. 1125.

Question 2:

A patient with stage 5 chronic kidney disease (CKD) is presenting with fever and chest pain, especially when taking a deep breath. The nurse detects a pericardial friction rub on auscultation. Which of the following conditions does the nurse suspect is common with this stage of kidney disease?

(see full question)

You selected:

Pericarditis

Correct Explanation:

Pericarditis occurs in many people with stage 5 CKD due to the uremia and prolonged diaylsis. The manifestations of uremic pericarditis resemble those of viral pericarditis with al ... (more)

Reference:

Grossman S, Porth CM (2014). Porth’s Pathophysiology: Concepts of Altered Health States. 9th ed. Philadelphia: Lippincott Williams & Wilkins, Chapter 42, Acute Renal Injury and Chronic Kidney Disease, p. 1123.

Question 3:

A client has acute tubular necrosis (ATN) caused by myoglobinuria. The nurse knows that myoglobinuria can result from which of the following? Select all that apply.

(see full question)

You selected:

• Prolonged seizures • Muscle damage


• Hyperthermia Incorrect Correct response:

• Muscle damage • Hyperthermia • Alcohol abuse • Prolonged seizures

Explanation:

Myoglobin, which stores oxygen in muscle fibers, is released when muscle is damaged. Large amounts of myoglobin, as in myoglobinuria, will cause tubular obstruction and damage to n ... (more)

Reference:

Grossman S, Porth CM (2014). Porth’s Pathophysiology: Concepts of Altered Health States. 9th ed. Philadelphia: Lippincott Williams & Wilkins, Chapter 42, Acute Renal Injury and Chronic Kidney Disease, p. 1115.

Question 4:

Hypertension frequently accompanies chronic kidney disease (CKD). The nurse knows that which of the following mechanisms can contribute to this hypertension? Select all that apply.

(see full question)

You selected:

• Increased vasoconstriction • Sodium retention • Renin-angiotensin aldosterone (RAAS) activation

Incorrect Correct response:

• Renin-angiotensin aldosterone (RAAS) activation • Sodium retention • Increased vasoconstriction • Increased blood volume

Explanation:

Activation of the RAAS, resulting in vasoconstriction and sodium and water retention, contribute to hypertension in CKD. Prostaglandins that maintain renal artery perfusion are dec ... (more)

Reference:

Grossman S, Porth CM (2014). Porth’s Pathophysiology: Concepts of Altered Health States. 9th ed. Philadelphia: Lippincott Williams & Wilkins, Chapter 42, Acute Renal Injury and Chronic Kidney Disease, p. 1122.

Question 5:


(see full question)

Neuromuscular disorders can be triggered by CKD. For those clients on dialysis, approximately two thirds suffer from what peripheral neuropathy?

You selected:

Burning hands and feet

Incorrect Correct response:

Restless legs syndrome

Explanation:

Restless legs syndrome is a manifestation of peripheral nerve involvement and can be seen in as many as two thirds of clients on dialysis. The other answers are not correct ..... (more)

Reference:

Grossman S, Porth CM (2014). Porth's Pathophysiology: Concepts of Altered Health States. 9th ed. Philadelphia: Lippincott Williams & Wilkins, Chapter 42: Acute Renal Failure and Chronic Kidney Disease, p. 1123.

Question 6:

A client is diagnosed with renal failure. The nurse must monitor for failure of which of the following functions? Select all that apply.

(see full question)

You selected:

• Removal of metabolic waste from blood • Acid-base balance • Adrenal secretion

Incorrect Correct response:

• Maintenance of body water • Electrolyte balance • Acid-base balance • Removal of metabolic waste from blood

Explanation:

Functions of the kidney include removal of metabolic end products and regulation of body water, electrolyte balance and acid-base balance. Although the adrenal glands are located ... (more)

Reference:

Grossman S, Porth CM (2014). Porth’s Pathophysiology: Concepts of Altered Health States. 9th ed. Philadelphia: Lippincott Williams & Wilkins, Chapter 42, Acute Renal Injury and Chronic Kidney Disease, 1112.

Question 7:

A client with chronic kidney disease (CKD) has developed asterixis. The nurse knows that asterixis is which of the following?

(see full question)


You selected:

Dorsiflexion of hands and feet

Correct Explanation:

Asterixis, which is involuntary dorsiflexion of hands and feet, can develop as CKD worsens. Burning sensation in feet, unsteady gait and demyelination of nerves can also accompany ... (more)

Reference:

Grossman S, Porth CM (2014). Porth’s Pathophysiology: Concepts of Altered Health States. 9th ed. Philadelphia: Lippincott Williams & Wilkins, Chapter 42, Acute Renal Injury and Chronic Kidney Disease, p. 1123.

Question 8: (see full question)

People with CKD have impaired immune responses to infection because of high levels of urea and metabolic wastes in the blood. What is one thing that is missing in an immune response in people with CKD?

You selected:

Failure to mount a fever with infection

Correct Explanation:

Many persons with CKD fail to mount a fever with infection, making diagnosis more difficult. All of the other answers occur.

Reference:

Grossman S, Porth CM (2014). Porth's Pathophysiology: Concepts of Altered Health States. 9th ed. Philadelphia: Lippincott Williams & Wilkins, Chapter 42: Acute Renal Failure and Chronic Kidney Disease, p. 1123.

Question 9:

A client is diagnosed with acute renal injury. The nurse will evaluate the client for which of the following possible causes for this disorder? Select all that apply.

(see full question)

You selected:

• Ischemic injury • Obstruction of urinary outflow • Nephrotoxic substances

Incorrect Correct response:

• Ischemic injury • Nonischemic reduced renal blood flow


• Obstruction of urinary outflow • Nephrotoxic substances Explanation:

Acute renal injury can result from ischemia, reduced renal blood without ischemia, nephrotoxic substances including some drugs, and obstruction of urinary outflow. Hormonal imbalan ... (more)

Reference:

Grossman S, Porth CM (2014). Porth’s Pathophysiology: Concepts of Altered Health States. 9th ed. Philadelphia: Lippincott Williams & Wilkins, Chapter 42, Acute Renal Injury and Chronic Kidney Disease, p.1113.

Question 10: (see full question)

Which of the following have the potential to cause chronic kidney disease? (Select all that apply.)

You selected:

• Diabetes • Hypertension

Incorrect Correct response:

• Hypertension • Diabetes • Glomerulonephritis

Explanation:

CKD can result from a number of conditions including diabetes, hypertension, glomerulonephritis as well as systemic lupus erythematosus, and polycystic kidney disease. These ... (more)

Question 1:

The nurse recognizes that acute renal injury is characterized by which of the following?

(see full question)

You selected:

Rapid decline in renal function

Correct Explanation:

Acute renal injury is a rapid decline in kidney function. BUN rises as nitrogenous wastes are not removed from the circulation. If the cause can be ameliorated, the injury is usual ... (more)

Reference:

Grossman S, Porth CM (2014). Porth’s Pathophysiology: Concepts of Altered Health States. 9th ed. Philadelphia: Lippincott Williams & Wilkins, Chapter 42,


Acute Renal Injury and Chronic Kidney Disease, p. 1112.

Question 2: (see full question)

You selected:

Which of the following individuals likely faces the greatest risk for the development of chronic kidney disease? A client with a recent diagnosis of type 2 diabetes who does not monitor his blood sugars or control his diet

Correct Explanation:

Chronic kidney disease (CKD) is a pathophysiologic process that results in the loss of nephrons and a decline in renal function that has persisted for more than 3 months. CKD can r ... (more)

Reference:

Grossman S, Porth CM (2014). Porth's Pathophysiology: Concepts of Altered Health States. 9th ed., Philadelphia: Wolters Kluwer Health/Lippincott Williams & Wilkins, 2014

Question 3:

A client is suspected to have renal disease. The nurse would suggest which of the following determinations to confirm this diagnosis? Select all that apply.

(see full question)

You selected:

• Ultrasonography • Urinary sediment of red and white cells • Imaging studies

Incorrect Correct response:

• Urinary sediment of red and white cells • Ultrasonography • Measurement of cystatin C • Imaging studies

Explanation:

Abnormal urinary sediment suggests the presence of renal disease, and imaging studies such as ultrasonography can detect some types of renal damage. Blood levels of the protein cys ... (more)

Reference:

Grossman S, Porth CM (2014). Porth’s Pathophysiology: Concepts of Altered Health States. 9th ed. Philadelphia: Lippincott Williams & Wilkins, Chapter 42, Acute Renal Injury and Chronic Kidney Disease, p. 1119.


Question 4: (see full question)

You selected:

A nurse is assessing a client diagnosed with CKD for neuromuscular manifestation. Select the manifestations the nurse may expect to find. Select all that apply. • Restless leg syndrome • Peripheral neuropathy • Perceptual errors • Loss of recent memory

Incorrect Correct response:

• Restless leg syndrome • Loss of recent memory • Peripheral neuropathy • Perceptual errors • Increased muscle strength

Explanation:

Many persons with CKD have alterations in peripheral and central nervous system function. Restless leg syndrome is a manifestation of peripheral nerve involvement, and muscle weakn ... (more)

Reference:

Grossman, S. & Porth, C.M. Porth's Pathophysiology: Concepts of Altered Health States, 9th ed., Philadelphia: Lippincott Williams & Wilkins, 2014, Chapter 42, Acute Renal Injury and Chronic Kidney Disease, p. 1123.

Question 5:

A client is diagnosed with chronic kidney disease (CKD). The nurse will monitor this client for which gastrointestinal signs and symptoms? Select all that apply.

(see full question)

You selected:

• Anorexia • Early morning nausea • Metallic taste • Decreased gastric secretion

Incorrect Correct response:

• Early morning nausea • Gastrointestinal ulceration • Metallic taste • Anorexia

Explanation:

Anorexia and nausea, especially in the early morning, occur with CKD. A metallic taste may exacerbate anorexia. Gastrointestinal ulceration and bleeding can develop. Increased lev ... (more)


Reference:

Grossman S, Porth CM (2014). Porth’s Pathophysiology: Concepts of Altered Health States. 9th ed. Philadelphia: Lippincott Williams & Wilkins, Chapter 42, Acute Renal Injury and Chronic Kidney Disease, p. 1123.

Question 6:

Older adults often have other chronic diseases that influence the early symptoms and signs of renal dysfunction. The nurse knows that which of the following can be the dominant clinical events in older adults with early kidney disease? Select all that apply.

(see full question)

You selected:

• Oliguria • Discolored urine • Hypertension

Incorrect Correct response:

• Heart failure • Hypertension

Explanation:

In older adults, heart failure and hypertension may be the dominant presenting signs of chronic kidney disease (CKD). Oliguria and discolored urine are more characteristic of young ... (more)

Reference:

Grossman S, Porth CM (2014). Porth’s Pathophysiology: Concepts of Altered Health States. 9th ed. Philadelphia: Lippincott Williams & Wilkins, Chapter 42, Acute Renal Injury and Chronic Kidney Disease, p. 1129.

Question 7: (see full question)

Reduced glomerular filtration rate (GFR), with a serum creatinine level that remains in the normal range, is associated with aging because elderly persons tend to have reduced:

You selected:

Muscle mass

Correct Explanation:

Serum creatinine level is directly related to muscle metabolism. Because muscle mass is reduced in elderly persons, the creatinine level does not increase as readily with a lower G ... (more)


Reference:

Grossman S, Porth CM (2014). Porth’s Pathophysiology: Concepts of Altered Health States. 9th ed. Philadelphia: Lippincott Williams & Wilkins, Chapter 42, Acute Renal Injury and Chronic Kidney Disease, p. 1129.

Question 8: (see full question)

A 45-year-old client with chronic kidney disease (CKD) voices concern about her dialysis treatment. The client would like to work and spend time with her family. Which type of dialysis will best fit this client’s lifestyle?

You selected:

Hemodialysis

Incorrect Correct response:

Nocturnal intermittent peritoneal dialysis (NIPD)

Explanation:

In NIPD, the client is given 10 hours of automatic cycling each night, with the abdomen left dry during the day. This is the most beneficial for this client. Individual preference, ... (more)

Reference:

Grossman S, Porth CM (2014). Porth’s Pathophysiology: Concepts of Altered Health States. 9th ed. Philadelphia: Lippincott Williams & Wilkins, Chapter 42, Acute Renal Injury and Chronic Kidney Disease, p. 1126.

Question 9:

A patient is evaluated for possible acute renal failure. The nurse knows that the most common indicator of acute renal failure is which of the following?

(see full question)

You selected:

Azotemia

Correct Explanation:

Azotemia, accumulation of urea nitrogen, uric acid, and creatinine in the blood, is the most common indicator of acute renal failure. Edema and anemia occur with chronic kidney dis ... (more)

Reference:

Grossman S, Porth CM (2014). Porth’s Pathophysiology: Concepts of Altered Health States. 9th ed. Philadelphia: Lippincott Williams & Wilkins, Chapter 42, Acute Renal Injury and Chronic Kidney Disease, p. 1113.

Question 10:

The nurse is reviewing the diagnosis of four male clients. Select the diagnosis


(see full question)

that places the clients at risk for developing postrenal kidney failure.

You selected:

Prostatic hyperplasia

Correct Explanation:

The most common cause of postrenal kidney failure is prostatic hyperplasia. Postrenal failure results from conditions that obstruct urine outflow. The obstruction can occur in the ... (more)

Question 1: (see full question)

A patient has an obstructive urine outflow related to benign prostatic hyperplasia. Due to the inability to excrete adequate amounts of urine, which of the following types of renal failure should the nurse closely monitor for?

You selected:

Intrarenal failure

Incorrect Correct response:

Postrenal failure

Explanation:

Postrenal failure results from obstruction of urine outflow from the kidneys. The obstruction can occur in the ureter, bladder, or urethra. Due to the increased urine not being ab ... (more)

Reference:

Grossman S, Porth CM (2014). Porth’s Pathophysiology: Concepts of Altered Health States. 9th ed. Philadelphia: Lippincott Williams & Wilkins, Chapter 42, Acute Renal Injury and Chronic Kidney Disease, p. 1114.

Question 2:

A client has acute tubular necrosis (ATN) caused by myoglobinuria. The nurse knows that myoglobinuria can result from which of the following? Select all that apply.

(see full question)

You selected:

• Hyperthermia • Muscle damage • Alcohol abuse • Prolonged seizures

Correct Explanation:

Myoglobin, which stores oxygen in muscle fibers, is released when muscle is damaged. Large amounts of myoglobin, as in myoglobinuria, will cause tubular obstruction and damage to n ... (more)


Reference:

Grossman S, Porth CM (2014). Porth’s Pathophysiology: Concepts of Altered Health States. 9th ed. Philadelphia: Lippincott Williams & Wilkins, Chapter 42, Acute Renal Injury and Chronic Kidney Disease, p. 1115.

Question 3: (see full question)

A patient sustained acute tubular injury approximately 2 hours ago. Which of the following phases of this disorder does the nurse recognize that the patient is in at this time?

You selected:

Initiating phase

Correct Explanation:

The onset, or initiating phase, which lasts hours or days, is the time from the onset of the precipitating event until tubular injury occurs.

Reference:

Grossman S, Porth CM (2014). Porth’s Pathophysiology: Concepts of Altered Health States. 9th ed. Philadelphia: Lippincott Williams & Wilkins, Chapter 42, Acute Renal Injury and Chronic Kidney Disease, p. 1115.

Question 4: (see full question)

The GFR is considered to be the best measure of renal function. What is used to estimate the GFR?

You selected:

Serum creatinine

Correct Explanation:

In clinical practice, GFR is usually estimated using the serum creatinine concentration. The other answers are not used to estimate the GFR.

Reference:

Grossman S, Porth CM (2014). Porth's Pathophysiology: Concepts of Altered Health States. 9th ed. Philadelphia: Lippincott Williams & Wilkins, Chapter 42: Acute Renal Failure and Chronic Kidney Disease, p. 1118.

Question 5:

The primary care provider for a newly admitted hospital client has added the glomerular filtration rate (GFR) to the blood work scheduled for this morning. The client's GFR results return as 50 mL/minute/1.73 m2. The nurse explains to the client that this result represents:

(see full question)


You selected:

A loss of over half the client's normal kidney function

Correct Explanation:

In clinical practice, GFR is usually estimated using the serum creatinine concentration. A GFR below 60 mL/minute/1.73 m2 represents a loss of one half or more of the level of normal ... (more)

Reference:

Grossman, S. & Porth, C.M. Porth's Pathophysiology: Concepts of Altered Health States, 9th ed., Philadelphia: Lippincott Williams & Wilkins, 2014, Chapter 42, Acute Renal Injury and Chronic Kidney Disease, p. 1118.

Question 6: (see full question)

The nurse is caring for a patient who has had acute blood loss from ruptured esophageal varices. Which of the following does the nurse recognize is an early sign of prerenal failure?

You selected:

Baseline urine output of 50 mL/hr that is now 10 mL/hr

Correct Explanation:

The kidney normally responds to a decrease in the glomerular filtration rate with a decrease in urine output. Thus, an early sign of prerenal failure is a sharp decrease in urine o ... (more)

Reference:

Question 7:

You selected:

(see full question)

You selected:

Incorrect

Correct

Correct response:

Explanation:

Explanation:

Reference:

Reference:


Question 8:

Question 2:

(see full question)

(see full question)

You selected:

You selected:

Correct

Correct

Explanation:

Explanation:

Reference:

Reference:

Question 9:

Question 3:

(see full question)

(see full question)

You selected:

You selected:

Incorrect

Incorrect

Correct response:

Correct response:

Explanation: Explanation: Reference:

Grossman S, Porth CM (2014). Porth's Pathophysiology: Concepts of Altered Health States. 9th ed. Philadelphia: Lippincott Williams & Wilkins, Chapter 42: Acute Renal Failure and Chronic Kidney Disease, p. 1114-1115.

Question 10: (see full question)

A drug abuser was found unconscious after shooting up heroin 2 days prior. Because of the pressure placed on the hip and arm, the client has developed rhabdomyolysis. The nurse knows this can:

You selected:

Obstruct the renal tubules with myoglobin and damage tubular cells

Correct Explanation:

Myoglobin normally is not found in the serum or urine. It has a low molecular weight; if it escapes into the circulation, it is rapidly filtered in the glomerulus. A life-threateni ... (more)


Question 1: (see full question)

You selected:

The health care provider is reviewing lab results of a client. Select the test that is the best measurement of overall kidney function? Serum creatinine levels

Incorrect Correct response:

Glomerular filtration rate (GFR)

Explanation:

GFR is the best overall measure of kidney function. GFR is usually estimated using the serum creatinine concentration. Creatinine, a by-product of muscle metabolism, is produced at ... (more)

Reference:

Grossman S, Porth CM (2014). Porth’s Pathophysiology: Concepts of Altered Health States. 9th ed. Philadelphia: Lippincott Williams & Wilkins, Chapter 42, Acute Renal Injury and Chronic Kidney Disease, p. 1118.

Question 2: (see full question)

A 45-year-old client with chronic kidney disease (CKD) voices concern about her dialysis treatment. The client would like to work and spend time with her family. Which type of dialysis will best fit this client’s lifestyle?

You selected:

Nocturnal intermittent peritoneal dialysis (NIPD)

Correct Explanation:

In NIPD, the client is given 10 hours of automatic cycling each night, with the abdomen left dry during the day. This is the most beneficial for this client. Individual preference, ... (more)

Reference:

Grossman S, Porth CM (2014). Porth’s Pathophysiology: Concepts of Altered Health States. 9th ed. Philadelphia: Lippincott Williams & Wilkins, Chapter 42, Acute Renal Injury and Chronic Kidney Disease, p. 1126.

Question 3:

A client has acute pyelonephritis. The nurse will monitor the client for development of which of the following?

(see full question)

You selected: Incorrect

Post renal failure


Correct response:

Intrarenal failure

Explanation:

Acute pyelonephritis, an active bacterial infection, can cause tubular cell necrosis and intrarenal failure. Acute pyelonephritis does not cause prerenal or postrenal failure or ch ... (more)

Reference:

Grossman S, Porth CM (2014). Porth’s Pathophysiology: Concepts of Altered Health States. 9th ed. Philadelphia: Lippincott Williams & Wilkins, Chapter 42, Acute Renal Injury and Chronic Kidney Disease, p. 1114.

Question 4:

A nurse is assessing a client for early manifestations of chronic kidney disease (CKD). Which would the nurse expect the client to display?

(see full question)

You selected:

Hypertension

Correct Explanation:

Hypertension is commonly an early manifestation of CKD. The mechanisms that cause the hypertension are multifactorial; they include increased vascular volume, increased peripheral ... (more)

Reference:

Grossman S, Porth CM (2014). Porth’s Pathophysiology: Concepts of Altered Health States. 9th ed. Philadelphia: Lippincott Williams & Wilkins, Chapter 42, Acute Renal Injury and Chronic Kidney Disease, p. 1119.

Question 5:

A client diagnosed with chronic kidney disease (CKD) is experiencing nausea and vomiting. Which would be the best instruction for the nurse to provide?

(see full question)

You selected:

Restrict intake of dietary protein

Correct Explanation:

Early morning nausea is common in CKD. Nausea and vomiting often improve with the restriction of dietary protein and after initiation of dialysis and disappears after kidney transp ... (more)

Reference:

Grossman S, Porth CM (2014). Porth’s Pathophysiology: Concepts of Altered Health States. 9th ed. Philadelphia: Lippincott Williams & Wilkins, Chapter 42, Acute Renal Injury and Chronic Kidney Disease, p. 1123.


Question 6: (see full question)

You selected:

The nurse is instructing a patient with advanced kidney disease (AKD) about a dietary regimen. Which of the following restrictions should the nurse be sure to include in the treatment plan to decrease the progress of renal impairment in people with AKD? Dietary protein

Correct Explanation:

Restriction of dietary proteins may decrease the progress of renal impairment in people with advanced renal disease. Proteins are broken down to form nitrogenous wastes, and reduci ... (more)

Reference:

Grossman S, Porth CM (2014). Porth’s Pathophysiology: Concepts of Altered Health States. 9th ed. Philadelphia: Lippincott Williams & Wilkins, Chapter 42, Acute Renal Injury and Chronic Kidney Disease, p. 1127.

Question 7: (see full question)

Accumulation of nitrogenous wastes such a urea in the circulatory system is an early sigh of chronic kidney disease (CKD). The nurse knows that normal levels of urea in blood are approximately which of the following?

You selected:

20 mg/dL

Correct Explanation:

Normal levels of blood urea nitrogen are approximately 20 mg/dL or less. The level increases as CKD worsens and can go as high as 800 mg/dL.

Reference:

Grossman S, Porth CM (2014). Porth’s Pathophysiology: Concepts of Altered Health States. 9th ed. Philadelphia: Lippincott Williams & Wilkins, Chapter 42, Acute Renal Injury and Chronic Kidney Disease, p. 1120.

Question 8: (see full question)

Acute renal failure occurs at a high rate in seriously ill people who are in intensive care units. What is the most common indicator of acute renal failure?

You selected:

Azotemia and a decrease in the GFR


Correct Explanation:

The most common indicator of acute renal failure is azotemia, an accumulation of nitrogenous wastes (urea nitrogen, uric acid, and creatinine) in the blood and a decrease in ... (more)

Reference:

Grossman S, Porth CM (2014). Porth's Pathophysiology: Concepts of Altered Health States. 9th ed. Philadelphia: Lippincott Williams & Wilkins, Chapter 42: Acute Renal Failure and Chronic Kidney Disease, p. 1113.

Question 9: (see full question)

Chronic kidney disease impacts many systems in the body. What is the number one hematologic disorder caused by CKD?

You selected:

Anemia

Correct Explanation:

The number one hematologic disorder that accompanies CKD is anemia. The other answers are incorrect.

Reference:

Grossman S, Porth CM (2014). Porth's Pathophysiology: Concepts of Altered Health States. 9th ed. Philadelphia: Lippincott Williams & Wilkins, Chapter 42: Acute Renal Failure and Chronic Kidney Disease, p. 1122.

Question 10: (see full question)

Vitamin D metabolism is deranged in clients with chronic kidney disease (CKD). The nurse recognizes that which of the following statements regarding vitamin D is correct?

You selected:

Kidneys convert inactive vitamin D to its active form, calcitriol.

Correct Explanation:

Inactive vitamin D is converted to active calcitriol in the kidneys. Calcitriol enhances gastrointestinal absorption of calcium, and suppresses release of PTH. Elevated levels of P ... (more)


(see full question)

A client is beginning to recover from acute tubular necrosis. The nurse would likely be assessing which of the following manifestations of the recovery phase of ATN?

You selected:

Diuresis

Question 1:

Correct Explanation:

The recovery phase is first noticed as increased/excessive output (diuresis) of dilute urine and a fall in serum creatinine, indicating that the nephrons have recovered to the point ... (more)

Reference:

Grossman S, Porth CM (2014). Porth’s Pathophysiology: Concepts of Altered Health States. 9th ed. Philadelphia: Lippincott Williams & Wilkins, Chapter 42, Acute Renal Injury and Chronic Kidney Disease, p. 1116.

Question 2:

While assessing a peritoneal dialysis client in his or her home, the nurse notes that the fluid draining from the abdomen is cloudy, is white in color, and contains a strong odor. The nurse suspects this client has developed a serious complication known as:

(see full question)

You selected:

Peritonitis

Correct Explanation:

Potential problems with peritoneal dialysis include infection, catheter malfunction, dehydration, hyperglycemia, and hernia. Bowel perforation can occur, but the fluid would be sto ... (more)

Reference:

Grossman, S. & Porth, C.M. Porth's Pathophysiology: Concepts of Altered Health States, 9th ed., Philadelphia: Lippincott Williams & Wilkins, 2014, Chapter 42, Acute Renal Injury and Chronic Kidney Disease, p. 1126.

Question 3:

A patient with stage 5 chronic kidney disease (CKD) is presenting with fever and chest pain, especially when taking a deep breath. The nurse detects a pericardial friction rub on auscultation. Which of the following conditions does the nurse suspect is common with this stage of kidney disease?

(see full question)

You selected: Correct

Pericarditis


Explanation:

Pericarditis occurs in many people with stage 5 CKD due to the uremia and prolonged diaylsis. The manifestations of uremic pericarditis resemble those of viral pericarditis with al ... (more)

Reference:

Grossman S, Porth CM (2014). Porth’s Pathophysiology: Concepts of Altered Health States. 9th ed. Philadelphia: Lippincott Williams & Wilkins, Chapter 42, Acute Renal Injury and Chronic Kidney Disease, p. 1123.

Question 4: (see full question)

Which of the following have the potential to cause chronic kidney disease? (Select all that apply.)

You selected:

• Hypertension • Glomerulonephritis • Diabetes

Correct Explanation:

CKD can result from a number of conditions including diabetes, hypertension, glomerulonephritis as well as systemic lupus erythematosus, and polycystic kidney disease. These ... (more)

Reference:

Grossman S, Porth CM (2014). Porth's Pathophysiology: Concepts of Altered Health States. 9th ed. Philadelphia: Lippincott Williams & Wilkins, Chapter 42: Acute Renal Failure and Chronic Kidney Disease, p. 1117.

Question 5:

Which of the following integumentary problems most often accompanies chronic kidney disease?

(see full question)

You selected:

Dry skin and pruritus

Correct Explanation:

Dry, itchy skin is a common consequence of CKD. Petechiae, purpura, hirsutism, and psoriasis are not noted to accompany or result from CKD.

Reference:

Grossman S, Porth CM (2014). Porth’s Pathophysiology: Concepts of Altered Health States. 9th ed. Philadelphia: Lippincott Williams & Wilkins, Chapter 42,


Acute Renal Injury and Chronic Kidney Disease, p. 1123.

(see full question)

A patient has prerenal failure. The nurse knows that this type of failure is characterized by which relationship of blood urea nitrogen (BUN) to serum creatinine levels?

You selected:

A BUN to creatinine level ratio of 20:1

Question 6:

Correct Explanation:

In prerenal failure, glomerular filtration rate (GFR) decreases, allowing more filtered urea to be reabsorbed into the circulatory system. Creatinine is filtered but remains in the ... (more)

Reference:

Grossman S, Porth CM (2014). Porth’s Pathophysiology: Concepts of Altered Health States. 9th ed. Philadelphia: Lippincott Williams & Wilkins, Chapter 42, Acute Renal Injury and Chronic Kidney Disease, p. 1114.

Question 7:

A client is diagnosed with a tumor in the urinary bladder. The nurse will monitor the client for which of the following?

(see full question)

You selected:

Intrarenal failure

Incorrect Correct response:

Postrenal failure

Explanation:

Postrenal failure is due to blockage of urine outflow. The obstruction can be in the ureter, the bladder, or the urethra. A tumor in the bladder can obstruct outflow, and cause int ... (more)

Reference:

Grossman S, Porth CM (2014). Porth’s Pathophysiology: Concepts of Altered Health States. 9th ed. Philadelphia: Lippincott Williams & Wilkins, Chapter 42, Acute Renal Injury and Chronic Kidney Disease, p. 1114.

Question 8:

A patient in renal failure has marked decrease in renal blood flow caused by hypovolemia, caused by gastrointestinal bleeding. The nurse is aware that this form of renal failure can be reversed if the bleeding is under control. Which of

(see full question)


the following forms of acute renal injury does this patient have? You selected:

Prerenal failure

Correct Explanation:

Prerenal failure, the most common form of acute renal failure, is characterized by a marked decrease in renal blood flow. It is reversible if the cause of the decreased renal blood ... (more)

Reference:

Grossman S, Porth CM (2014). Porth’s Pathophysiology: Concepts of Altered Health States. 9th ed. Philadelphia: Lippincott Williams & Wilkins, Chapter 42, Acute Renal Injury and Chronic Kidney Disease, p. 1113.

Question 9:

A client with chronic kidney disease (CKD) has developed asterixis. The nurse knows that asterixis is which of the following?

(see full question)

You selected:

Dorsiflexion of hands and feet

Correct Explanation:

Asterixis, which is involuntary dorsiflexion of hands and feet, can develop as CKD worsens. Burning sensation in feet, unsteady gait and demyelination of nerves can also accompany ... (more)

Reference:

Grossman S, Porth CM (2014). Porth’s Pathophysiology: Concepts of Altered Health States. 9th ed. Philadelphia: Lippincott Williams & Wilkins, Chapter 42, Acute Renal Injury and Chronic Kidney Disease, p. 1123.

Question 10:

A client is diagnosed with chronic kidney disease (CKD). The nurse will monitor this client for which of the following Select all that apply.

(see full question)

You selected:

• Polyuria • Hyperkalemia • Metabolic alkalosis

Incorrect Correct response:

• Polyuria • Hyperkalemia • Hypocalcemia


• Hyponatremia Explanation:

The failing kidneys lose ability to concentrate urine and to reabsorb sodium. Hyperkalemia develops late in CKD, as nephrons can no longer regulate potassium excretion. Metabolic a ... (more)

Question 1:

The nurse recognizes that acute renal injury is characterized by which of the following?

(see full question)

You selected:

Rapid decline in renal function

Correct Explanation:

Acute renal injury is a rapid decline in kidney function. BUN rises as nitrogenous wastes are not removed from the circulation. If the cause can be ameliorated, the injury is usual ... (more)

Reference:

Grossman S, Porth CM (2014). Porth’s Pathophysiology: Concepts of Altered Health States. 9th ed. Philadelphia: Lippincott Williams & Wilkins, Chapter 42, Acute Renal Injury and Chronic Kidney Disease, p. 1112.

Question 2:

The health care provider is reviewing lab results of a client. Select the test that is the best measurement of overall kidney function?

(see full question)

You selected:

Glomerular filtration rate (GFR)

Correct Explanation:

GFR is the best overall measure of kidney function. GFR is usually estimated using the serum creatinine concentration. Creatinine, a by-product of muscle metabolism, is produced at ... (more)

Reference:

Grossman S, Porth CM (2014). Porth’s Pathophysiology: Concepts of Altered Health States. 9th ed. Philadelphia: Lippincott Williams & Wilkins, Chapter 42, Acute Renal Injury and Chronic Kidney Disease, p. 1118.

Question 3:

The nurse is educating a patient with chronic kidney disease (CKD). What is the recommended daily fluid intake for this patient?

(see full question)


You selected:

A minimum of 2000 mL/day to flush out the kidneys

Incorrect Correct response:

A daily fluid intake of 500 to 800 mL/day to maintain hydration

Explanation:

Daily fluid intake of 500 to 800 mL/day will replace insensible water loss plus a quantity equal to the 24-hour urine output. Intake of 2000 mL will exceed the renal ability to excr ... (more)

Reference:

Grossman S, Porth CM (2014). Porth’s Pathophysiology: Concepts of Altered Health States. 9th ed. Philadelphia: Lippincott Williams & Wilkins, Chapter 42, Acute Renal Injury and Chronic Kidney Disease, p. 1127.

Question 4: (see full question)

A patient has an obstructive urine outflow related to benign prostatic hyperplasia. Due to the inability to excrete adequate amounts of urine, which of the following types of renal failure should the nurse closely monitor for?

You selected:

Postrenal failure

Correct Explanation:

Postrenal failure results from obstruction of urine outflow from the kidneys. The obstruction can occur in the ureter, bladder, or urethra. Due to the increased urine not being ab ... (more)

Reference:

Grossman S, Porth CM (2014). Porth’s Pathophysiology: Concepts of Altered Health States. 9th ed. Philadelphia: Lippincott Williams & Wilkins, Chapter 42, Acute Renal Injury and Chronic Kidney Disease, p. 1114.

Question 5: (see full question)

Manifestations of childhood renal disease are varied and may differ from adultonset renal failure. A school-aged child with chronic kidney disease may exhibit:

You selected:

Inability to control bladder, resulting in incontinence

Incorrect Correct response:

Developmental delays such as uncoordinated gait and minimal fine motor skills

Explanation:

Childhood chronic kidney disease is manifested by growth and developmental


delays and late onset sexual maturity as a result of the uremic effects on endocrine function, bone abnor ... (more)

Reference:

Grossman S, Porth CM (2014). Porth’s Pathophysiology: Concepts of Altered Health States. 9th ed. Philadelphia: Lippincott Williams & Wilkins, Chapter 42, Acute Renal Injury and Chronic Kidney Disease, p. 1129.

Question 6: (see full question)

In hemodialysis, access to the vascular system is most commonly through what?

You selected:

External arteriovenous shunt

Incorrect Correct response:

Internal arteriovenous fistula

Explanation:

Access to the vascular system is accomplished through an external arteriovenous shunt (i.e., tubing implanted into an artery and a vein) or, more commonly, through an interna ... (more)

Reference:

Grossman S, Porth CM (2014). Porth's Pathophysiology: Concepts of Altered Health States. 9th ed. Philadelphia: Lippincott Williams & Wilkins, Chapter 42: Acute Renal Failure and Chronic Kidney Disease, p. 1125.

Question 7: (see full question)

Impaired skin integrity and skin manifestations are common in persons with chronic kidney disease. Pale skin and subcutaneous bruising are often present as a result of:

You selected:

Thrombocytopenia

Incorrect Correct response:

Impaired platelet function

Explanation:

Bruising and pale skin are present with chronic kidney disease because platelet function is impaired. Adequate platelets are available, but the function is abnormal. Renal clients ... (more)


Reference:

Grossman S, Porth CM (2014). Porth’s Pathophysiology: Concepts of Altered Health States. 9th ed. Philadelphia: Lippincott Williams & Wilkins, Chapter 42, Acute Renal Injury and Chronic Kidney Disease, p. 1122.

Question 8:

The health care provider has prescribed an aminoglycoside (gentamicin) for a client. The nurse is aware that the client is at risk for:

(see full question)

You selected:

Nephrotoxic acute tubular necrosis

Correct Explanation:

Pharmacologic agents that are directly toxic to the renal tubule include aminoglycosides (e.g., gentamicin), cancer chemotherapeutic agents such as cystplastin and ifosfamide, and ... (more)

Reference:

Grossman S, Porth CM (2014). Porth’s Pathophysiology: Concepts of Altered Health States. 9th ed. Philadelphia: Lippincott Williams & Wilkins, Chapter 42, Acute Renal Injury and Chronic Kidney Disease, p. 1115.

Question 9: (see full question)

A 72-year-old patient is scheduled for a kidney transplant. The nurse knows that which aspect of advanced age has a positive effect on the success of kidney transplant survival?

You selected:

Acceptance of immunosuppressive therapy

Incorrect Correct response:

Reduction in T-lymphocyte function

Explanation:

The general reduction in T-lymphocyte function with subsequent decrease in immune system activity that occurs with aging would foster transplant survival. The other options would n ... (more)

Reference:

Grossman S, Porth CM (2014). Porth’s Pathophysiology: Concepts of Altered Health States. 9th ed. Philadelphia: Lippincott Williams & Wilkins, Chapter 42, Acute Renal Injury and Chronic Kidney Disease, p. 1129.

Question 10:

A client diagnosed with CKD has begun to experience periods of epistaxis and


(see full question)

developed bruising of skin and subcutaneous tissues. The nurse recognizes these manifestations as:

You selected:

Decreased erythropoietin

Incorrect Correct response:

Impaired platelet function

Explanation:

The coagulation disorders of CKD are mainly caused by platelet dysfunction. Platelet counts may be slightly decreased, and the bleeding time is prolonged because of abnormal adhesi ... (more)

Question 1:

The nurse is scheduled to teach a client experiencing urinary incontinence about Kegel exercises. Which of the following descriptors should the nurse include in this education?

(see full question)

You selected:

“Contract and relax the pelvic floor muscles at least 10 times every hour while awake.”

Correct Explanation:

Exercises for the pelvic muscles or Kegel exercises involve repetitive contraction and relaxation of the pelvic floor muscles and are an essential component of client-dependent beh ... (more)

Reference:

Grossman, S. & Porth, C.M. Porth's Pathophysiology: Concepts of Altered Health States, 9th ed., Philadelphia: Lippincott Williams & Wilkins, 2014, Chapter 43, Disorders of the Bladder and Lower Urinary Tract, p. 1144.

Question 2:

A nurse is caring for a patient in spinal shock. Which of the following interventions is appropriate in relation to the patient’s urinary status?

(see full question)

You selected:

Perform intermittent catheterization

Correct Explanation:

After a spinal cord injury, the early effects on bladder function are quite different from those that follow recovery from the initial injury. During the period immediately after s ... (more)

Reference:

Grossman S, Porth CM (2014). Porth’s Pathophysiology: Concepts of Altered


Health States. 9th ed. Philadelphia: Lippincott Williams & Wilkins, Chapter 43, Disorders of the Bladder and Lower Urinary Tract, p. 1140.

Question 3: (see full question)

You selected:

The nursing students have learned in class that causes of urinary obstruction and urinary incontinence include which of the following? Select all that apply. • Structural changes in the urethra • Structural changes in the bladder • Impairment of neurologic control of bladder function

Correct Explanation:

Urinary obstruction and urinary incontinence can be caused by several factors, including structural changes in the bladder, structural changes in the urethra, and impairment of neu ... (more)

Reference:

Grossman S, Porth CM (2014). Porth’s Pathophysiology: Concepts of Altered Health States. 9th ed. Philadelphia: Lippincott Williams & Wilkins, Chapter 43, Disorders of the Bladder and Lower Urinary Tract, p. 1137.

Question 4:

A patient is admitted with lower urinary tract obstruction and stasis. Which of the following is the primary intervention?

(see full question)

You selected:

Urinary catheterization

Correct Explanation:

The relief of lower urinary tract obstruction is directed toward relief of bladder distension through urinary catheterization. This is the primary intervention. The other intervent ... (more)

Reference:

Grossman S, Porth CM (2014). Porth’s Pathophysiology: Concepts of Altered Health States. 9th ed. Philadelphia: Lippincott Williams & Wilkins, Chapter 43, Disorders of the Bladder and Lower Urinary Tract, p. 1139.

Question 5:

Which physiologic change in the elderly population contributes to urinary incontinence?

(see full question)


You selected:

Decline in detrusor muscle function

Correct Explanation:

In the elderly population, overall bladder capacity is reduced as is urethral closing pressure. Changes associated with aging include decline in detrusor muscle function, decrease ... (more)

Reference:

Grossman S, Porth CM (2014). Porth’s Pathophysiology: Concepts of Altered Health States. 9th ed. Philadelphia: Lippincott Williams & Wilkins, Chapter 43, Disorders of the Bladder and Lower Urinary Tract, p. 1145.

Question 6: (see full question)

A client who has a problem with incontinence loses a small amount of urine every time she coughs or sneezes. This type of incontinence is known as which of the following?

You selected:

Stress

Correct Explanation:

Stress incontinence is the involuntary loss of urine associated with activities such as coughing and sneezing. Urge incontinence is the urgency and frequency associated with hypera ... (more)

Reference:

Grossman S, Porth CM (2014). Porth’s Pathophysiology: Concepts of Altered Health States. 9th ed. Philadelphia: Lippincott Williams & Wilkins, Chapter 43, Disorders of the Bladder and Lower Urinary Tract, p. 1142.

Question 7:

A client informs the nurse that she is afraid of developing bladder cancer because her mother had it. She asks the nurse what signs and symptoms are present with this cancer. What does the nurse tell the client is the most common sign of bladder cancer?

(see full question)

You selected:

Gross hematuria

Correct Explanation:

The most common sign of bladder cancer is painless hematuria. Gross hematuria is a presenting sign in the majority of cases. Frequency, urgency and dysuria occasionally accompany t ... (more)


Reference:

Grossman S, Porth CM (2014). Porth’s Pathophysiology: Concepts of Altered Health States. 9th ed. Philadelphia: Lippincott Williams & Wilkins, Chapter 43, Disorders of the Bladder and Lower Urinary Tract, p. 1147.

Question 8:

A client reports that she frequently suffers from UTIs after engaging in sexual intercourse. Which would be the best information for the nurse to provide?

(see full question)

You selected:

Decrease fluid intake before intercourse

Incorrect Correct response:

Increase fluid intake before intercourse

Explanation:

A nonpharmacologic approach to the treatment of frequent UTIs associated with sexual intercourse is to increase fluid intake before intercourse and to void soon after intercourse ..... (more)

Reference:

Grossman, S. & Porth, C.M. Porth's Pathophysiology: Concepts of Altered Health States, 9th ed., Philadelphia: Lippincott Williams & Wilkins, 2014, Chapter 41, Disorders of Renal Function, p. 1097.

Question 9: (see full question)

A female client asks the nurse if there is any noninvasive treatment to help with the involuntary loss of urine that occurs when she coughs or sneezes. Which is the best response by the nurse?

You selected:

Kegel exercises

Correct Explanation:

Kegel exercise involves repetitive contraction and relaxation of the pelvic floor muscles and is a noninvasive way of strengthening the pelvic floor muscles and is a client-depende ... (more)

Reference:

Grossman S, Porth CM (2014). Porth’s Pathophysiology: Concepts of Altered Health States. 9th ed. Philadelphia: Lippincott Williams & Wilkins, Chapter 43, Disorders of the Bladder and Lower Urinary Tract, p. 1144.


Question 10: (see full question)

Acute overdistention of the bladder can occur in anyone with a neurogenic bladder that does not empty. What is the maximum amount of urine should a nurse empty out of a neurogenic bladder at one time?

You selected:

No more than 500 cc of urine at one time

Incorrect Correct response:

No more than 1000 cc of urine at one time

Explanation:

With acute overdistention of the bladder, usually no more than 1000 mL of urine is removed from the bladder at one time. The other answers are incorrect.

Question 1:

Which of the following patients should have a feeling of bladder fullness?

(see full question)

You selected:

The patient with 200 mL of urine in their bladder

Correct Explanation:

In a normally functioning bladder, the sensation of bladder fullness is first perceived when the bladder contains 100–200 mL of urine. The patient who has neuropathy or is in ... (more)

Reference:

Grossman S, Porth CM (2014). Porth’s Pathophysiology: Concepts of Altered Health States. 9th ed. Philadelphia: Lippincott Williams & Wilkins, Chapter 43, Disorders of the Bladder and Lower Urinary Tract, p. 1140.

Question 2:

A parent asks the nurse what signs and symptoms a child would display if the child had a UTI. Which is the best response by the nurse?

(see full question)

You selected:

They do not present with typical symptoms.

Correct Explanation:

Unlike adults, children frequently do not present with the typical signs of a UTI. Fever is a common sign of UTI in children. Neonates: bacteremia and signs and symptoms of septice ... (more)


Reference:

Grossman, S. & Porth, C.M. Porth's Pathophysiology: Concepts of Altered Health States, 9th ed., Philadelphia: Lippincott Williams & Wilkins, 2014, Chapter 41, Disorders of Renal Function, p. 1097.

Question 3: (see full question)

The patient who has been admitted with a problem with his bladder has a postvoid residual (PVR) of 250 ml. The nurse understands that this indicates which of the following?

You selected:

Inadequate bladder emptying

Correct Explanation:

Postvoid residual (PVR) urine volume provides information about bladder emptying and NOT storage. A PVR value of less than 50ml is considered adequate bladder emptying and more tha ... (more)

Reference:

Grossman S, Porth CM (2014). Porth’s Pathophysiology: Concepts of Altered Health States. 9th ed. Philadelphia: Lippincott Williams & Wilkins, Chapter 43, Disorders of the Bladder and Lower Urinary Tract, p. 1136.

Question 4: (see full question)

The client has just been diagnosed with bladder cancer and asks the nurse what causes it. Which of the following would be the nurse's best response to the client?

You selected:

"It is usually inherited."

Incorrect Correct response:

"The cause is unknown."

Explanation:

The nurse should tell the patient that the cause of bladder cancer is unknown; however, evidence suggests that its origin is due to local influences such as carcinogens that are ex ... (more)

Reference:

Grossman S, Porth CM (2014). Porth’s Pathophysiology: Concepts of Altered Health States. 9th ed. Philadelphia: Lippincott Williams & Wilkins, Chapter 43, Disorders of the Bladder and Lower Urinary Tract, p. 1147.


Question 5: (see full question)

You selected:

Although urinary obstruction and urinary incontinence have almost opposite effects on urination, they can both result from: Bladder distensibility loss

Incorrect Correct response:

Bladder structure changes

Explanation:

Disorders of lower urinary tract structure and function include urinary obstruction with retention or stasis of urine and urinary incontinence with involuntary loss of urine. Both ... (more)

Reference:

Grossman S, Porth CM (2014). Porth’s Pathophysiology: Concepts of Altered Health States. 9th ed. Philadelphia: Lippincott Williams & Wilkins, Chapter 43, Disorders of the Bladder and Lower Urinary Tract, p. 1137.

Question 6: (see full question)

A neighbor is complaining to a friend (who happens to be a nurse) about several changes in their body. Which of the following complaints raises a “red flag” because it could be a sign of epithelial cell bladder cancer?

You selected:

“I noticed my urine is pinkish red, but I’m not having any pain when I pee.”

Correct Explanation:

The most common sign of bladder cancer is intermittent painless hematuria. Fluid retention, stress incontinence, and pain with exercise are not usual signs of cancer.

Reference:

Grossman, S. & Porth, C.M. Porth's Pathophysiology: Concepts of Altered Health States, 9th ed., Philadelphia: Lippincott Williams & Wilkins, 2014, Chapter 43, Disorders of the Bladder and Lower Urinary Tract, p. 1137.

Question 7:

The nurse is conducting a staff development program on urinary disorders. The nurse evaluates that the participants are understanding the information when they identify that disruption of which muscle's contraction can lead to the inability to expel urine from the bladder.

(see full question)

You selected: Correct

Detrusor


Explanation:

The detrusor muscle, also called the muscle of micturition, contracts to allow urine flow from the bladder. The detrusor muscle coordinates the functions of the internal and extern ... (more)

Reference:

Grossman S, Porth CM (2014). Porth’s Pathophysiology: Concepts of Altered Health States. 9th ed. Philadelphia: Lippincott Williams & Wilkins, Chapter 43, Disorders of the Bladder and Lower Urinary Tract, p. 1133.

Question 8:

A patient has a postvoid residual (PVR) volume of 250 mL. Which of the following information would the nurse tell the patient?

(see full question)

You selected:

“This value indicates you are having difficulty emptying your bladder.”

Correct Explanation:

The nurse should inform the client that values over 200 mL indicate that the patient is not able to adequately empty the bladder and that further evaluation is necessary. It is not ... (more)

Reference:

Grossman S, Porth CM (2014). Porth’s Pathophysiology: Concepts of Altered Health States. 9th ed. Philadelphia: Lippincott Williams & Wilkins, Chapter 43, Disorders of the Bladder and Lower Urinary Tract, p. 1136.

Question 9:

A client who has had recurrent UTIs asks the nurse about the old wise tale of drinking cranberry juice daily. The nurse can respond:

(see full question)

You selected:

“Research suggests cranberry juice will reduce bacterial adherence to the lining of the urinary tract.”

Correct Explanation:

Cranberry juice or blueberry juice has been suggested as a preventive measure for persons with frequent UTIs. Studies suggest that these juices reduce bacterial adherence to the ep ... (more)

Reference:

Grossman, S. & Porth, C.M. Porth's Pathophysiology: Concepts of Altered Health States, 9th ed., Philadelphia: Lippincott Williams & Wilkins, 2014, Chapter 41, Disorders of Renal Function, p. 1096.


(see full question)

The nursing student, while studying anatomy and physiology, correctly identifies which of the following to be responsible for carrying urine to the bladder?

You selected:

Ureters

Question 10:

Correct Explanation:

Urine passes from the kidneys to the bladder through the ureters. The bowman's capsule is a component of the kidney. The external sphincter serves as a reserve mechanism to stop mi ... (more)

Question 1: (see full question)

Which of the following types of pharmacological therapy does the nurse anticipate administering to a patient for treatment of a spastic bladder in order to decrease bladder hyperactivity?

You selected:

Calcium channel blockers

Incorrect Correct response:

Anticholinergic medications

Explanation:

Among the methods used to treat spastic bladder and detrusor-sphincter dyssynergia are the administration of anticholinergic medications to decrease bladder hyperactivity and urina ... (more)

Reference:

Grossman S, Porth CM (2014). Porth’s Pathophysiology: Concepts of Altered Health States. 9th ed. Philadelphia: Lippincott Williams & Wilkins, Chapter 43, Disorders of the Bladder and Lower Urinary Tract, p. 1140.

Question 2:

The nursing instructor who is teaching about disorders of the lower urinary tract realizes a need for further instruction when one of the students makes which of the following statements?

(see full question)

You selected:

"Alterations in bladder function can include urinary obstruction with retention or stasis of urine."

Incorrect Correct response:

"Alterations in bladder function can only occur when there is incontinence."


Explanation:

Alterations in bladder function include urinary obstruction with retention or stasis of urine and urinary incontinence with involuntary loss of urine. Alterations in bladder functi ... (more)

Reference:

Grossman S, Porth CM (2014). Porth’s Pathophysiology: Concepts of Altered Health States. 9th ed. Philadelphia: Lippincott Williams & Wilkins, Chapter 43, Disorders of the Bladder and Lower Urinary Tract, p. 1137.

Question 3:

A female teenager has experienced three uncomplicated urinary tract infections in the last 3 months. Knowing the anatomical location of the urethra, the nurse should educate this teenager about:

(see full question)

You selected:

Wiping from front to back to preventEscherichia coli contamination of the urethra

Correct Explanation:

Most commonly, urinary tract infections (UTIs) are caused by Escherichia coli that enter through the urethra. Escherichia coli are abundant in fecal matter. Other uropathic pathoge ... (more)

Reference:

Grossman, S. & Porth, C.M. Porth's Pathophysiology: Concepts of Altered Health States, 9th ed., Philadelphia: Lippincott Williams & Wilkins, 2014, Chapter 41, Disorders of Renal Function, p. 1094.

Question 4:

The nurse understands that medications although very beneficial to clients can also have harmful effects. When working with elderly clients the nurse should recognize which of the following is a common result of potent, fast-acting diuretics?

(see full question)

You selected:

Urge incontinence

Correct Explanation:

Medications prescribed for other health problems may prevent a healthy bladder from functioning properly. Potent, fast-acting diuretics are known for their ability to cause urge in ... (more)


Reference:

Grossman S, Porth CM (2014). Porth’s Pathophysiology: Concepts of Altered Health States. 9th ed. Philadelphia: Lippincott Williams & Wilkins, Chapter 43, Disorders of the Bladder and Lower Urinary Tract, p. 1145.

Question 5:

A patient is concerned about the possibility of having bladder cancer after his brother was diagnosed with it 2 years ago. Which of the following assessment data obtained by the nurse would indicate that the patient should be screened for this disease?

(see full question)

You selected:

Patient reports that he occasionally has blood in his urine but has no pain with it.

Correct Explanation:

The most common sign of bladder cancer is painless hematuria. Gross hematuria is a presenting sign in the majority of people with the disease, and microscopic hematuria is present ... (more)

Reference:

Grossman S, Porth CM (2014). Porth’s Pathophysiology: Concepts of Altered Health States. 9th ed. Philadelphia: Lippincott Williams & Wilkins, Chapter 43, Disorders of the Bladder and Lower Urinary Tract, p. 1147.

Question 6:

A 56-year-old client complains of urinary incontinence when she laughs. The nurse documents this as which type of incontinence?

(see full question)

You selected:

Stress

Correct Explanation:

In women, pelvic floor weakness may cause involuntary loss of urine (stress incontinence) during any activity that increases intra-abdominal pressure such as laughing or coughing ..... (more)

Reference:

Grossman S, Porth CM (2014). Porth’s Pathophysiology: Concepts of Altered Health States. 9th ed. Philadelphia: Lippincott Williams & Wilkins, Chapter 43, Disorders of the Bladder and Lower Urinary Tract, p. 1142.

Question 7:

The nurse is teaching a patient with a spinal cord injury about self-


(see full question)

catheterization. Which of the following measurements is important to include about the amount of urine that should be allowed to collect in the bladder between catheterizations?

You selected:

400-500 mL

Incorrect Correct response:

300-400 mL

Explanation:

The patient procedure typically is used for self-catheterization. It is performed at 3- to 4-hour intervals to prevent overdistention of the bladder. The best results are obtaine ... (more)

Reference:

Grossman S, Porth CM (2014). Porth’s Pathophysiology: Concepts of Altered Health States. 9th ed. Philadelphia: Lippincott Williams & Wilkins, Chapter 43, Disorders of the Bladder and Lower Urinary Tract, p. 1141.

Question 8: (see full question)

A patient is describing difficulty with urinating and informs the nurse that every time she coughs or laughs, she urinates and has begun to wear a thin pad. Which of the following types of urinary incontinence is the patient describing?

You selected:

Stress incontinence

Correct Explanation:

Stress incontinence is the loss of urine associated with activities, such as coughing, that increase intra-abdominal pressure. Overactive bladder/urge incontinence is urgency and f ... (more)

Reference:

Grossman S, Porth CM (2014). Porth’s Pathophysiology: Concepts of Altered Health States. 9th ed. Philadelphia: Lippincott Williams & Wilkins, Chapter 43, Disorders of the Bladder and Lower Urinary Tract, p. 1142.

Question 9:

After reviewing the 24-hour intake and output of a hospital client, the nurse suspects that the client may be experiencing flaccid bladder dysfunction. Which of the following diagnostic methods is most likely to confirm or rule out whether the client is retaining urine?

(see full question)

You selected:

Measurement of postvoid residual (PVR) by ultrasound


Correct Explanation:

Measurement of PVR can be achieved quickly, accurately, and painlessly by the use of ultrasonography. A PVR value of less than 50 mL is considered adequate bladder emptying, and mo ... (more)

Reference:

Grossman S, Porth CM (2014). Porth’s Pathophysiology: Concepts of Altered Health States. 9th ed. Philadelphia: Lippincott Williams & Wilkins, Chapter 43, Disorders of the Bladder and Lower Urinary Tract, p. 1136.

Question 10: (see full question)

A client who suffers from spastic bladder has been catheterized to promote bladder emptying. Which of the following medications should the nurse plan on the physician ordering to also treat this problem?

You selected:

Anticholinergic medication

Correct Explanation:

Among the methods used to treat spastic bladder are the administration of anticholinergic medications to decrease bladder hyperactivity and urinary catheterization to produce bladd ... (more)

Question 1:

A client reports that she frequently suffers from UTIs after engaging in sexual intercourse. Which would be the best information for the nurse to provide?

(see full question)

You selected:

Increase fluid intake before intercourse

Correct Explanation:

A nonpharmacologic approach to the treatment of frequent UTIs associated with sexual intercourse is to increase fluid intake before intercourse and to void soon after intercourse ..... (more)

Reference:

Grossman, S. & Porth, C.M. Porth's Pathophysiology: Concepts of Altered Health States, 9th ed., Philadelphia: Lippincott Williams & Wilkins, 2014, Chapter 41, Disorders of Renal Function, p. 1097.


Question 2: (see full question)

You selected:

Which clients have an increased risk for developing UTIs? A client who is or is diagnosed with: Select all that apply. • Prostate disease • Urinary obstruction • A premenopausal woman • Elderly

Incorrect Correct response:

• Urinary obstruction • Neurogenic disorders • Elderly • Prostate disease

Explanation:

There is an increased risk for UTIs in persons with urinary obstruction and reflux, in people with neurogenic disorders that impair bladder emptying, in women who are sexually acti ... (more)

Reference:

Grossman, S. & Porth, C.M. Porth's Pathophysiology: Concepts of Altered Health States, 9th ed., Philadelphia: Lippincott Williams & Wilkins, 2014, Chapter 41, Disorders of Renal Function, p. 1094.

Question 3:

An older male comes to the clinic with the chief complaint of having difficulty voiding. The physician diagnoses him with a lower urinary tract obstruction and stasis. Which of the following should the nurse suspect to be the most frequent cause of this client's problem?

(see full question)

You selected:

Enlargement of prostate gland

Correct Explanation:

In males the most important acquired cause of urinary stasis and urinary obstruction is external compression of the urethra caused by the enlargement of the prostate gland. Bladder ... (more)

Reference:

Grossman S, Porth CM (2014). Porth’s Pathophysiology: Concepts of Altered Health States. 9th ed. Philadelphia: Lippincott Williams & Wilkins, Chapter 43, Disorders of the Bladder and Lower Urinary Tract, p. 1138.

Question 4:

A patient with stress incontinence states, "every time I laugh or cough, I


(see full question)

urinate on myself. Which of the following behavioral measures would be beneficial for the nurse to teach the patient?

You selected:

Kegel exercises

Correct Explanation:

Behavioral methods for incontinence include fluid management, timed/prompted voiding, pelvic floor exercises (Kegel exercises), bladder retraining, and toileting assistance. Bladde ... (more)

Reference:

Grossman S, Porth CM (2014). Porth’s Pathophysiology: Concepts of Altered Health States. 9th ed. Philadelphia: Lippincott Williams & Wilkins, Chapter 43, Disorders of the Bladder and Lower Urinary Tract, p. 1144.

Question 5: (see full question)

The client has just been diagnosed with bladder cancer and asks the nurse what causes it. Which of the following would be the nurse's best response to the client?

You selected:

"The cause is unknown."

Correct Explanation:

The nurse should tell the patient that the cause of bladder cancer is unknown; however, evidence suggests that its origin is due to local influences such as carcinogens that are ex ... (more)

Reference:

Grossman S, Porth CM (2014). Porth’s Pathophysiology: Concepts of Altered Health States. 9th ed. Philadelphia: Lippincott Williams & Wilkins, Chapter 43, Disorders of the Bladder and Lower Urinary Tract, p. 1147.

Question 6:

A mother asks, “why can’t my 1 year old go to the bathroom by himself?” Which of the following is the nurse’s best response?

(see full question)

You selected:

“How long have you been trying to train him?”

Incorrect Correct response:

“Your child is too young to begin toilet training.”

Explanation:

Conscious control of the bladder begins at about 2 to 3 years of age. A 1-yearold child is too young to be able to consciously control toileting.


Reference:

Grossman S, Porth CM (2014). Porth’s Pathophysiology: Concepts of Altered Health States. 9th ed. Philadelphia: Lippincott Williams & Wilkins, Chapter 43, Disorders of the Bladder and Lower Urinary Tract, p. 1135.

Question 7: (see full question)

A nurse is planning a staff development program on urinary dysfunction. The nurse should include that the lower motor neurons in the sacral segment of the spinal column control micturition:

You selected:

inhibition.

Incorrect Correct response:

reflexes.

Explanation:

The lower motor neuron (LMN) parasympathetic and sympathetic neuron spinal cord centers control micturition reflexes. The cortical and subcortical centers control micturition inhib ... (more)

Reference:

Grossman S, Porth CM (2014). Porth’s Pathophysiology: Concepts of Altered Health States. 9th ed. Philadelphia: Lippincott Williams & Wilkins, Chapter 43, Disorders of the Bladder and Lower Urinary Tract, p. 1134.

Question 8: (see full question)

A patient with diabetes mellitus type 1 has bladder atony with dysfunction. Which of the following assessments indicates a primary concern for this patient?

You selected:

Incontinence

Incorrect Correct response:

Ascending urinary tract infection

Explanation:

Patients with diabetes mellitus type 1 are at high risk for kidney disease. An ascending urinary tract infection can affect the kidneys and is of primary concern. Residual volume i ... (more)

Reference:

Grossman S, Porth CM (2014). Porth’s Pathophysiology: Concepts of Altered Health States. 9th ed. Philadelphia: Lippincott Williams & Wilkins, Chapter 43,


Disorders of the Bladder and Lower Urinary Tract, p. 1141.

Question 9: (see full question)

Urinary obstruction in the lower urinary tract triggers changes to the urinary system to compensate for the obstruction. What is an early change the system makes in its effort to cope with an obstruction?

You selected:

Ability to suppress urination is increased.

Incorrect Correct response:

The stretch receptors in the bladder wall become hypersensitive.

Explanation:

During the early stage of obstruction, the bladder begins to hypertrophy and becomes hypersensitive to afferent stimuli arising from stretch receptors in the bladder wall. Th ... (more)

Reference:

Grossman S, Porth CM (2014). Porth's Pathophysiology: Concepts of Altered Health States. 9th ed. Philadelphia: Lippincott Williams & Wilkins, Chapter 43: Disorders of the Bladder and Lower Urinary Tract, p. 1138.

Question 10: (see full question)

Urinary incontinence can be a problem with the elderly. One method of treatment is habit training, or bladder training. When using this treatment with an elderly person, how frequently should he or she be voiding?

You selected:

Every 3 to 5 hours

Incorrect Correct response:

Every 2 to 4 hours

Explanation:

Habit training with regularly scheduled toileting—usually every 2 to 4 hours— often is effective. The other answers are incorrect.

Question 1:

A young mother asks,“Why can my 3-year-old daughter have a bowel movement on the toilet but she wets her pants?” Which of the following is the nurse’s best response?

(see full question)


You selected:

“Bowel control occurs earlier than bladder control.”

Correct Explanation:

Bowel control occurs earlier than bladder control in young children, and girls gain control earlier than boys. This would not be a symptom of urinary tract infection or difficulty ... (more)

Reference:

Grossman S, Porth CM (2014). Porth’s Pathophysiology: Concepts of Altered Health States. 9th ed. Philadelphia: Lippincott Williams & Wilkins, Chapter 43, Disorders of the Bladder and Lower Urinary Tract, p. 1136.

Question 2:

A patient is admitted with lower urinary tract obstruction and stasis. Which of the following is the primary intervention?

(see full question)

You selected:

Urinary catheterization

Correct Explanation:

The relief of lower urinary tract obstruction is directed toward relief of bladder distension through urinary catheterization. This is the primary intervention. The other intervent ... (more)

Reference:

Grossman S, Porth CM (2014). Porth’s Pathophysiology: Concepts of Altered Health States. 9th ed. Philadelphia: Lippincott Williams & Wilkins, Chapter 43, Disorders of the Bladder and Lower Urinary Tract, p. 1139.

Question 3: (see full question)

The nursing instructor who is teaching about incontinence in the elderly recognizes a need for further instruction when one of the students makes which of the following statements?

You selected:

"Frequency is not a major problem for the elderly."

Correct Explanation:

Incontinence can increase social isolation in the older adult population and frequency can lead to institutionalization of older adults. Many factors can contribute to incontinence ... (more)


Reference:

Grossman S, Porth CM (2014). Porth’s Pathophysiology: Concepts of Altered Health States. 9th ed. Philadelphia: Lippincott Williams & Wilkins, Chapter 43, Disorders of the Bladder and Lower Urinary Tract, p. 1145.

Question 4:

The nurse is conducting a community health education program on urinary retention and urinary incontinence. The nurse determines that the participants are understanding the education when they state that the most common cause of urinary retention is which of the following?

(see full question)

You selected:

Pelvic inflammatory disease

Incorrect Correct response:

Prostate enlargement

Explanation:

In men, the enlarged prostate (due to hypertrophy or hyperplasia) frequently causes nonrelaxing external sphincter with urine retention. The stress response ca ... (more)

Reference:

Grossman S, Porth CM (2014). Porth’s Pathophysiology: Concepts of Altered Health States. 9th ed. Philadelphia: Lippincott Williams & Wilkins, Chapter 43, Disorders of the Bladder and Lower Urinary Tract, p. 1138.

Question 5:

An elderly woman comes to the hospital and is diagnosed with urinary obstruction and retention. Which of the following symptoms would the nurse expect this client to demonstrate? Select all that apply.

(see full question)

You selected:

• Hesitancy • Nladder distention • Overflow incontinence

Incorrect Correct response:

• Nladder distention • Hesitancy • Frequency • Overflow incontinence

Explanation:

Signs of outflow obstruction and urine retention are: bladder distention, hesitancy, straining when initiating urination, small and weak stream, frequency, feeling of incomplete bl ... (more)


Reference:

Grossman S, Porth CM (2014). Porth’s Pathophysiology: Concepts of Altered Health States. 9th ed. Philadelphia: Lippincott Williams & Wilkins, Chapter 43, Disorders of the Bladder and Lower Urinary Tract, p. 1139.

Question 6:

A nurse is caring for a patient in spinal shock. Which of the following interventions is appropriate in relation to the patient’s urinary status?

(see full question)

You selected:

Perform intermittent catheterization

Correct Explanation:

After a spinal cord injury, the early effects on bladder function are quite different from those that follow recovery from the initial injury. During the period immediately after s ... (more)

Reference:

Grossman S, Porth CM (2014). Porth’s Pathophysiology: Concepts of Altered Health States. 9th ed. Philadelphia: Lippincott Williams & Wilkins, Chapter 43, Disorders of the Bladder and Lower Urinary Tract, p. 1140.

Question 7:

The nursing students have learned in class that causes of urinary obstruction and urinary incontinence include which of the following? Select all that apply.

(see full question)

You selected:

• Impairment of neurologic control of bladder function • Structural changes in the bladder • Structural changes in the urethra

Correct Explanation:

Urinary obstruction and urinary incontinence can be caused by several factors, including structural changes in the bladder, structural changes in the urethra, and impairment of neu ... (more)

Reference:

Grossman S, Porth CM (2014). Porth’s Pathophysiology: Concepts of Altered Health States. 9th ed. Philadelphia: Lippincott Williams & Wilkins, Chapter 43, Disorders of the Bladder and Lower Urinary Tract, p. 1137.

Question 8:

Which of the following types of pharmacological therapy does the nurse anticipate administering to a patient for treatment of a spastic bladder in order

(see full question)


to decrease bladder hyperactivity? You selected:

Calcium channel blockers

Incorrect Correct response:

Anticholinergic medications

Explanation:

Among the methods used to treat spastic bladder and detrusor-sphincter dyssynergia are the administration of anticholinergic medications to decrease bladder hyperactivity and urina ... (more)

Reference:

Grossman S, Porth CM (2014). Porth’s Pathophysiology: Concepts of Altered Health States. 9th ed. Philadelphia: Lippincott Williams & Wilkins, Chapter 43, Disorders of the Bladder and Lower Urinary Tract, p. 1140.

Question 9: (see full question)

The nursing instructor, while teaching about renal function and disorders, informs the students that the most frequent form of urinary tract cancer is which of the following?

You selected:

Bladder

Correct Explanation:

Bladder cancer is the most frequent form of urinary cancer in the United States. Uterine cancer and testicular cancer are not considered to be cancers of the urinary tract.

Reference:

Grossman S, Porth CM (2014). Porth’s Pathophysiology: Concepts of Altered Health States. 9th ed. Philadelphia: Lippincott Williams & Wilkins, Chapter 43, Disorders of the Bladder and Lower Urinary Tract, p. 1146.

Question 10: (see full question)

One of the many tests done during urodynamic studies is the sphincter electromyelogram. What does this test study?

You selected:

Activity of the voluntary muscles of the perineal area

Correct Explanation:

Sphincter electromyelogram allows the activity of the striated (voluntary)


muscles of the perineal area to be studied. Cystometry measures the ability of the bladder to store ... (more)

Question 1: (see full question)

You selected:

A mother asks, “why can’t my 1 year old go to the bathroom by himself?” Which of the following is the nurse’s best response? “Your child is too young to begin toilet training.”

Correct Explanation:

Conscious control of the bladder begins at about 2 to 3 years of age. A 1-yearold child is too young to be able to consciously control toileting.

Reference:

Grossman S, Porth CM (2014). Porth’s Pathophysiology: Concepts of Altered Health States. 9th ed. Philadelphia: Lippincott Williams & Wilkins, Chapter 43, Disorders of the Bladder and Lower Urinary Tract, p. 1135.

Question 2: (see full question)

Urinary obstruction in the lower urinary tract triggers changes to the urinary system to compensate for the obstruction. What is an early change the system makes in its effort to cope with an obstruction?

You selected:

The stretch receptors in the bladder wall become hypersensitive.

Correct Explanation:

During the early stage of obstruction, the bladder begins to hypertrophy and becomes hypersensitive to afferent stimuli arising from stretch receptors in the bladder wall. Th ... (more)

Reference:

Grossman S, Porth CM (2014). Porth's Pathophysiology: Concepts of Altered Health States. 9th ed. Philadelphia: Lippincott Williams & Wilkins, Chapter 43: Disorders of the Bladder and Lower Urinary Tract, p. 1138.

Question 3: (see full question)

Urinary incontinence can be a problem with the elderly. One method of treatment is habit training, or bladder training. When using this treatment with


an elderly person, how frequently should he or she be voiding? You selected:

Every 2 to 4 hours

Correct Explanation:

Habit training with regularly scheduled toileting—usually every 2 to 4 hours— often is effective. The other answers are incorrect.

Reference:

Grossman S, Porth CM (2014). Porth's Pathophysiology: Concepts of Altered Health States. 9th ed. Philadelphia: Lippincott Williams & Wilkins, Chapter 43: Disorders of the Bladder and Lower Urinary Tract, p. 1146.

Question 4: (see full question)

The nursing instructor who is teaching about disorders of the lower urinary tract realizes a need for further instruction when one of the students makes which of the following statements?

You selected:

"Alterations in bladder function can only occur when there is incontinence."

Correct Explanation:

Alterations in bladder function include urinary obstruction with retention or stasis of urine and urinary incontinence with involuntary loss of urine. Alterations in bladder functi ... (more)

Reference:

Grossman S, Porth CM (2014). Porth’s Pathophysiology: Concepts of Altered Health States. 9th ed. Philadelphia: Lippincott Williams & Wilkins, Chapter 43, Disorders of the Bladder and Lower Urinary Tract, p. 1137.

Question 5: (see full question)

The patient who has been admitted with a problem with his bladder has a postvoid residual (PVR) of 250 ml. The nurse understands that this indicates which of the following?

You selected:

Inadequate bladder emptying

Correct Explanation:

Postvoid residual (PVR) urine volume provides information about bladder emptying and NOT storage. A PVR value of less than 50ml is considered adequate bladder emptying and more tha ... (more)


Reference:

Grossman S, Porth CM (2014). Porth’s Pathophysiology: Concepts of Altered Health States. 9th ed. Philadelphia: Lippincott Williams & Wilkins, Chapter 43, Disorders of the Bladder and Lower Urinary Tract, p. 1136.

Question 6:

A patient has a postvoid residual (PVR) volume of 250 mL. Which of the following information would the nurse tell the patient?

(see full question)

You selected:

“This value indicates you are having difficulty emptying your bladder.”

Correct Explanation:

The nurse should inform the client that values over 200 mL indicate that the patient is not able to adequately empty the bladder and that further evaluation is necessary. It is not ... (more)

Reference:

Grossman S, Porth CM (2014). Porth’s Pathophysiology: Concepts of Altered Health States. 9th ed. Philadelphia: Lippincott Williams & Wilkins, Chapter 43, Disorders of the Bladder and Lower Urinary Tract, p. 1136.

Question 7: (see full question)

A female client asks the nurse if there is any noninvasive treatment to help with the involuntary loss of urine that occurs when she coughs or sneezes. Which is the best response by the nurse?

You selected:

Kegel exercises

Correct Explanation:

Kegel exercise involves repetitive contraction and relaxation of the pelvic floor muscles and is a noninvasive way of strengthening the pelvic floor muscles and is a client-depende ... (more)

Reference:

Grossman S, Porth CM (2014). Porth’s Pathophysiology: Concepts of Altered Health States. 9th ed. Philadelphia: Lippincott Williams & Wilkins, Chapter 43, Disorders of the Bladder and Lower Urinary Tract, p. 1144.


(see full question)

A client tells the nurse that he is experiencing involuntary loss of urine associated with a strong desire to void (urgency). The nurse would recognize this as:

You selected:

Stress incontinence

Question 8:

Incorrect Correct response:

Urge incontinence

Explanation:

Urge incontinence is the involuntary loss of urine associated with a strong desire to void (urgency). Stress incontinence represents the involuntary loss of urine that occurs when, ... (more)

Reference:

Grossman, S. & Porth, C.M. Porth's Pathophysiology: Concepts of Altered Health States, 9th ed., Philadelphia: Lippincott Williams & Wilkins, 2014, Chapter 43, Disorders of the Bladder and Lower Urinary Tract, p. 1142.

Question 9:

In anatomy class, the instructor asks, “Explain how urine is expelled from the bladder during voiding.” The student with the most accurate response would be:

(see full question)

You selected:

“The detrusor muscle contract down on the urine and the ureteral orifices are forced shut. The external sphincter relaxes as urine moves out of the bladder.”

Correct Explanation:

During the act of micturition, the detrusor muscle of the bladder fundus and bladder neck contracts down on the urine and the ureteral orifices are forced shut. The bladder neck is ... (more)

Reference:

Grossman, S. & Porth, C.M. Porth's Pathophysiology: Concepts of Altered Health States, 9th ed., Philadelphia: Lippincott Williams & Wilkins, 2014, Chapter 43, Disorders of Bladder and Lower Urinary Tract, p. 1135.

Question 10:

The nursing instructor informs the students during a lecture that the control of the emptying functions of the bladder involves which of the following? Select all that apply.

(see full question)

You selected:

• Involuntary autonomic nervous system • Voluntary somatic nervous system


Incorrect Correct response:

• Iinvoluntary autonomic nervous system • Voluntary somatic nervous system

Explanation:

The control of the emptying functions of the bladder involves both the involuntary (ANS) and voluntary (somatic nervous system). The parasympathetic nervous system is not involved ... (more)

Question 1:

A patient has a postvoid residual (PVR) volume of 40 mL. Which of the following information would the nurse teach the patient?

(see full question)

You selected:

“This value indicates you are having difficulty emptying your bladder.”

Incorrect Correct response:

“This is a normal value.”

Explanation:

The nurse should teach the patient that PVR values under 50 mL indicate that the patient is adequately emptying the bladder and further evaluation is not necessary.

Reference:

Grossman S, Porth CM (2014). Porth’s Pathophysiology: Concepts of Altered Health States. 9th ed. Philadelphia: Lippincott Williams & Wilkins, Chapter 43, Disorders of the Bladder and Lower Urinary Tract, p. 1136.

Question 2:

A client asks the nurse what the most common sign/symptom of bladder cancer is. Which is the best response by the nurse?

(see full question)

You selected:

Painless hematuria

Correct Explanation:

The most common sign of bladder cancer is painless hematuria. Gross hematuria is a presenting sign in 75% of persons with the disease, and microscopic hematuria is present in most ... (more)

Reference:

Grossman S, Porth CM (2014). Porth’s Pathophysiology: Concepts of Altered Health States. 9th ed. Philadelphia: Lippincott Williams & Wilkins, Chapter 43, Disorders of the Bladder and Lower Urinary Tract, p. 1147.


(see full question)

A patient is describing difficulty with urinating and informs the nurse that every time she coughs or laughs, she urinates and has begun to wear a thin pad. Which of the following types of urinary incontinence is the patient describing?

You selected:

Stress incontinence

Question 3:

Correct Explanation:

Stress incontinence is the loss of urine associated with activities, such as coughing, that increase intra-abdominal pressure. Overactive bladder/urge incontinence is urgency and f ... (more)

Reference:

Grossman S, Porth CM (2014). Porth’s Pathophysiology: Concepts of Altered Health States. 9th ed. Philadelphia: Lippincott Williams & Wilkins, Chapter 43, Disorders of the Bladder and Lower Urinary Tract, p. 1142.

Question 4:

The nurse is conducting a community health education program on urinary retention and urinary incontinence. The nurse determines that the participants are understanding the education when they state that the most common cause of urinary retention is which of the following?

(see full question)

You selected:

Prostate enlargement

Correct Explanation:

In men, the enlarged prostate (due to hypertrophy or hyperplasia) frequently causes nonrelaxing external sphincter with urine retention. The stress response ca ... (more)

Reference:

Grossman S, Porth CM (2014). Porth’s Pathophysiology: Concepts of Altered Health States. 9th ed. Philadelphia: Lippincott Williams & Wilkins, Chapter 43, Disorders of the Bladder and Lower Urinary Tract, p. 1138.

Question 5: (see full question)

A patient with diabetes mellitus type 1 has bladder atony with dysfunction. Which of the following assessments indicates a primary concern for this patient?

You selected:

Ascending urinary tract infection


Correct Explanation:

Patients with diabetes mellitus type 1 are at high risk for kidney disease. An ascending urinary tract infection can affect the kidneys and is of primary concern. Residual volume i ... (more)

Reference:

Grossman S, Porth CM (2014). Porth’s Pathophysiology: Concepts of Altered Health States. 9th ed. Philadelphia: Lippincott Williams & Wilkins, Chapter 43, Disorders of the Bladder and Lower Urinary Tract, p. 1141.

Question 6: (see full question)

Acute overdistention of the bladder can occur in anyone with a neurogenic bladder that does not empty. What is the maximum amount of urine should a nurse empty out of a neurogenic bladder at one time?

You selected:

No more than 1000 cc of urine at one time

Correct Explanation:

With acute overdistention of the bladder, usually no more than 1000 mL of urine is removed from the bladder at one time. The other answers are incorrect.

Reference:

Grossman S, Porth CM (2014). Porth's Pathophysiology: Concepts of Altered Health States. 9th ed. Philadelphia: Lippincott Williams & Wilkins, Chapter 43: Disorders of the Bladder and Lower Urinary Tract, p. 1141.

Question 7:

While studying the GU system the nursing students learn that which of the following group achieves continence first?

(see full question)

You selected:

No definite proof that one sex or the other achieves it sooner

Incorrect Correct response:

Girls

Explanation:

Girls typically achieve continence before boys and bowel control is typically achieved before bladder control.


Reference:

Grossman S, Porth CM (2014). Porth’s Pathophysiology: Concepts of Altered Health States. 9th ed. Philadelphia: Lippincott Williams & Wilkins, Chapter 43, Disorders of the Bladder and Lower Urinary Tract, p. 1135.

Question 8: (see full question)

Which of the following clinical manifestations would tell a nurse that a patient is having progressive decompensation related to obstruction of urinary outflow?

You selected:

Patient complains of waking up several times in the night to void.

Incorrect Correct response:

When tested for residual urine volume, 1400 mL of urine is obtained when patient is catheterized.

Explanation:

When compensatory mechanisms are no longer effective, signs of decompensation begin to appear. The period of detrusor muscle contraction becomes too short to expel the urine comple ... (more)

Reference:

Grossman S, Porth CM (2014). Porth’s Pathophysiology: Concepts of Altered Health States. 9th ed. Philadelphia: Lippincott Williams & Wilkins, Chapter 43, Disorders of the Bladder and Lower Urinary Tract, p. 1138.

Question 9:

A nurse working on a GU floor is advising a new graduate. The new graduate comments that there are so many different ways to classify urinary obstructions that she will never learn all of the types. The nurse informs the new nurse that they are usually classified according to which of the following? Select all that apply.

(see full question)

You selected:

• Upper or lower • Simple or complex • Partial or complete

Incorrect Correct response:

• Congenital or acquired • Partial or complete • Acute or chronic • Upper or lower

Explanation:

Urinary tract obstructions are classified according to cause (congenital or acquired), degree (partial or complete), duration (acute or chronic), and level (upper or lower).


Reference:

Grossman S, Porth CM (2014). Porth’s Pathophysiology: Concepts of Altered Health States. 9th ed. Philadelphia: Lippincott Williams & Wilkins, Chapter 43, Disorders of the Bladder and Lower Urinary Tract, p. 1138.

Question 10:

The nurse working on a GU floor caring for clients who have bladder cancer identifies which of the following to be acceptable treatments for this cancer? Select all that apply.

(see full question)

You selected:

• Radiation therapy • Chemotherapy • Surgical removal

Correct Explanation:

The methods used in treatment depend upon the grade of the tumor and degree of invasiveness. They include surgical removal, radiation therapy, and chemotherapy. Hypnosis and herbal ... (more)

Question 1:

A patient is concerned about the possibility of having bladder cancer after his brother was diagnosed with it 2 years ago. Which of the following assessment data obtained by the nurse would indicate that the patient should be screened for this disease?

(see full question)

You selected:

Patient reports that he occasionally has blood in his urine but has no pain with it.

Correct Explanation:

The most common sign of bladder cancer is painless hematuria. Gross hematuria is a presenting sign in the majority of people with the disease, and microscopic hematuria is present ... (more)

Reference:

Grossman S, Porth CM (2014). Porth’s Pathophysiology: Concepts of Altered Health States. 9th ed. Philadelphia: Lippincott Williams & Wilkins, Chapter 43, Disorders of the Bladder and Lower Urinary Tract, p. 1147.


Question 2: (see full question)

You selected:

A client reports that she frequently suffers from UTIs after engaging in sexual intercourse. Which would be the best information for the nurse to provide? Increase fluid intake before intercourse

Correct Explanation:

A nonpharmacologic approach to the treatment of frequent UTIs associated with sexual intercourse is to increase fluid intake before intercourse and to void soon after intercourse ..... (more)

Reference:

Grossman, S. & Porth, C.M. Porth's Pathophysiology: Concepts of Altered Health States, 9th ed., Philadelphia: Lippincott Williams & Wilkins, 2014, Chapter 41, Disorders of Renal Function, p. 1097.

Question 3:

A client informs the nurse that she is afraid of developing bladder cancer because her mother had it. She asks the nurse what signs and symptoms are present with this cancer. What does the nurse tell the client is the most common sign of bladder cancer?

(see full question)

You selected:

Gross hematuria

Correct Explanation:

The most common sign of bladder cancer is painless hematuria. Gross hematuria is a presenting sign in the majority of cases. Frequency, urgency and dysuria occasionally accompany t ... (more)

Reference:

Grossman S, Porth CM (2014). Porth’s Pathophysiology: Concepts of Altered Health States. 9th ed. Philadelphia: Lippincott Williams & Wilkins, Chapter 43, Disorders of the Bladder and Lower Urinary Tract, p. 1147.

Question 4:

An older male comes to the clinic with the chief complaint of having difficulty voiding. The physician diagnoses him with a lower urinary tract obstruction and stasis. Which of the following should the nurse suspect to be the most frequent cause of this client's problem?

(see full question)

You selected: Correct

Enlargement of prostate gland


Explanation:

In males the most important acquired cause of urinary stasis and urinary obstruction is external compression of the urethra caused by the enlargement of the prostate gland. Bladder ... (more)

Reference:

Grossman S, Porth CM (2014). Porth’s Pathophysiology: Concepts of Altered Health States. 9th ed. Philadelphia: Lippincott Williams & Wilkins, Chapter 43, Disorders of the Bladder and Lower Urinary Tract, p. 1138.

Question 5: (see full question)

A public health nurse is conducting a health promotion class for a group of older adults. Which of the participants' following statements demonstrates an accurate understanding of the risk factors for bladder cancer?

You selected:

“More than ever, I guess it would worthwhile for me to quit smoking.”

Correct Explanation:

Smoking is implicated in 30% to 50% of all bladder cancers among males who are current or past smokers. Cranberry juice may be of benefit in the prevention of UTIs, not cancer, and ... (more)

Reference:

Grossman S, Porth CM (2014). Porth’s Pathophysiology: Concepts of Altered Health States. 9th ed. Philadelphia: Lippincott Williams & Wilkins, Chapter 43, Disorders of the Bladder and Lower Urinary Tract, p. 1147.

Question 6:

A newly diagnosed paraplegic client who suffered an automobile accident appears to have control of bladder emptying. The health care provider explains this process to the client/family stating, “This function is allowing the motor component of the neural reflex to assist with bladder emptying and is primarily controlled by the:

(see full question)

You selected:

Somatic nervous system.”

Incorrect Correct response:

Parasympathetic division of the ANS.“

Explanation:

The motor component of the neural reflex to assist with bladder emptying is primarily controlled by the parasympathetic division of the ANS, and the relaxation and storage function ... (more)


Reference:

Grossman, S. & Porth, C.M. Porth's Pathophysiology: Concepts of Altered Health States, 9th ed., Philadelphia: Lippincott Williams & Wilkins, 2014, Chapter 17, Organization and Control of Neural Function, p. 413.

Question 7: (see full question)

A client tells the nurse that he is experiencing involuntary loss of urine associated with a strong desire to void (urgency). The nurse would recognize this as:

You selected:

Urge incontinence

Correct Explanation:

Urge incontinence is the involuntary loss of urine associated with a strong desire to void (urgency). Stress incontinence represents the involuntary loss of urine that occurs when, ... (more)

Reference:

Grossman, S. & Porth, C.M. Porth's Pathophysiology: Concepts of Altered Health States, 9th ed., Philadelphia: Lippincott Williams & Wilkins, 2014, Chapter 43, Disorders of the Bladder and Lower Urinary Tract, p. 1142.

Question 8: (see full question)

Which of the following types of pharmacological therapy does the nurse anticipate administering to a patient for treatment of a spastic bladder in order to decrease bladder hyperactivity?

You selected:

Anticholinergic medications

Correct Explanation:

Among the methods used to treat spastic bladder and detrusor-sphincter dyssynergia are the administration of anticholinergic medications to decrease bladder hyperactivity and urina ... (more)

Reference:

Grossman S, Porth CM (2014). Porth’s Pathophysiology: Concepts of Altered Health States. 9th ed. Philadelphia: Lippincott Williams & Wilkins, Chapter 43, Disorders of the Bladder and Lower Urinary Tract, p. 1140.


(see full question)

The client has just been diagnosed with bladder cancer and asks the nurse what causes it. Which of the following would be the nurse's best response to the client?

You selected:

"The cause is unknown."

Question 9:

Correct Explanation:

The nurse should tell the patient that the cause of bladder cancer is unknown; however, evidence suggests that its origin is due to local influences such as carcinogens that are ex ... (more)

Reference:

Grossman S, Porth CM (2014). Porth’s Pathophysiology: Concepts of Altered Health States. 9th ed. Philadelphia: Lippincott Williams & Wilkins, Chapter 43, Disorders of the Bladder and Lower Urinary Tract, p. 1147.

Question 10:

When conducting an admission interview with a client with a history of urinary incontinence, the nurse will specifically ask whether the client is prescribed which classification of medications in order to determine a possible cause? Select all that apply.

(see full question)

You selected:

• Sedatives • Diuretics • Tricyclic antidepressants

Incorrect Correct response:

• Diuretics • Hypnotics • Sedatives

Explanation:

Medication prescribed for other health problems may prevent a healthy bladder from functioning normally. Drugs such as hypnotics, tranquilizers, and sedatives can interfere with th ... (more)

Question 1:

A patient has bladder distension. Which of the following is the nurse's best action?

(see full question)

You selected:

Assessing the patient's temperature

Incorrect Correct response:

Assisting the patient to urinate


Explanation:

The nurse should assess if the patient can urinate and assist with this before moving to catheterizing the patient. The nurse should not call the healthcare provider without first ... (more)

Reference:

Grossman S, Porth CM (2014). Porth’s Pathophysiology: Concepts of Altered Health States. 9th ed. Philadelphia: Lippincott Williams & Wilkins, Chapter 43, Disorders of the Bladder and Lower Urinary Tract, p. 1139

Question 2: (see full question)

A nurse is planning a staff development program on urinary dysfunction. The nurse should include that the lower motor neurons in the sacral segment of the spinal column control micturition:

You selected:

reflexes.

Correct Explanation:

The lower motor neuron (LMN) parasympathetic and sympathetic neuron spinal cord centers control micturition reflexes. The cortical and subcortical centers control micturition inhib ... (more)

Reference:

Grossman S, Porth CM (2014). Porth’s Pathophysiology: Concepts of Altered Health States. 9th ed. Philadelphia: Lippincott Williams & Wilkins, Chapter 43, Disorders of the Bladder and Lower Urinary Tract, p. 1134.

Question 3:

A patient has a postvoid residual (PVR) volume of 40 mL. Which of the following information would the nurse teach the patient?

(see full question)

You selected:

“This is a normal value.”

Correct Explanation:

The nurse should teach the patient that PVR values under 50 mL indicate that the patient is adequately emptying the bladder and further evaluation is not necessary.

Reference:

Grossman S, Porth CM (2014). Porth’s Pathophysiology: Concepts of Altered Health States. 9th ed. Philadelphia: Lippincott Williams & Wilkins, Chapter 43, Disorders of the Bladder and Lower Urinary Tract, p. 1136.


Question 4: (see full question)

You selected:

Which of the following assessments indicates to the nurse that a patient may have a spastic bladder dysfunction? Incontinence

Correct Explanation:

A spastic bladder condition causes inability to store urine. Incontinence would be a symptom of inability to store urine.

Reference:

Grossman S, Porth CM (2014). Porth’s Pathophysiology: Concepts of Altered Health States. 9th ed. Philadelphia: Lippincott Williams & Wilkins, Chapter 43, Disorders of the Bladder and Lower Urinary Tract, p. 1139.

Question 5:

While studying about the process of urination, the nursing student learns that which of the following is known as the "muscle of micturition"?

(see full question)

You selected:

External sphincter muscle

Incorrect Correct response:

Detrusor muscle

Explanation:

The detrusor muscle is the uncle of micturition. In the bladder neck is the continuation of the detrusor muscle known as the internal urethral sphincter, which acts as a sphincter ... (more)

Reference:

Grossman S, Porth CM (2014). Porth’s Pathophysiology: Concepts of Altered Health States. 9th ed. Philadelphia: Lippincott Williams & Wilkins, Chapter 43, Disorders of the Bladder and Lower Urinary Tract, p. 1133.

Question 6: (see full question)

The nursing instructor, while teaching about renal function and disorders, informs the students that the most frequent form of urinary tract cancer is which of the following?

You selected:

Bladder

Correct


Explanation:

Bladder cancer is the most frequent form of urinary cancer in the United States. Uterine cancer and testicular cancer are not considered to be cancers of the urinary tract.

Reference:

Grossman S, Porth CM (2014). Porth’s Pathophysiology: Concepts of Altered Health States. 9th ed. Philadelphia: Lippincott Williams & Wilkins, Chapter 43, Disorders of the Bladder and Lower Urinary Tract, p. 1146.

Question 7:

A bladder cancer client asks the nurse, “What did the doctor mean by intravesicular chemotherapy? Am I going to lose all my hair and have to do for treatments over months and months?” The best response would be:

(see full question)

You selected:

“This is when they put the chemotherapy directly into the bladder to kill any cancer cells.”

Correct Explanation:

Surgical treatment of superficial bladder cancer is often followed by intravesicular chemotherapy or immunotherapy, a procedure in which the therapeutic agent is directly instilled ... (more)

Reference:

Grossman, S. & Porth, C.M. Porth's Pathophysiology: Concepts of Altered Health States, 9th ed., Philadelphia: Lippincott Williams & Wilkins, 2014, Chapter 43, Disorders of the Bladder and Lower Urinary Tract, p. 1147.

Question 8: (see full question)

One of the many tests done during urodynamic studies is the sphincter electromyelogram. What does this test study?

You selected:

Activity of the voluntary muscles of the perineal area

Correct Explanation:

Sphincter electromyelogram allows the activity of the striated (voluntary) muscles of the perineal area to be studied. Cystometry measures the ability of the bladder to store ... (more)

Reference:

Grossman S, Porth CM (2014). Porth's Pathophysiology: Concepts of Altered


Health States. 9th ed. Philadelphia: Lippincott Williams & Wilkins, Chapter 43: Disorders of the Bladder and Lower Urinary Tract, p. 1137.

(see full question)

A young mother asks,“Why can my 3-year-old daughter have a bowel movement on the toilet but she wets her pants?” Which of the following is the nurse’s best response?

You selected:

“Bowel control occurs earlier than bladder control.”

Question 9:

Correct Explanation:

Bowel control occurs earlier than bladder control in young children, and girls gain control earlier than boys. This would not be a symptom of urinary tract infection or difficulty ... (more)

Reference:

Grossman S, Porth CM (2014). Porth’s Pathophysiology: Concepts of Altered Health States. 9th ed. Philadelphia: Lippincott Williams & Wilkins, Chapter 43, Disorders of the Bladder and Lower Urinary Tract, p. 1136.

Question 10: (see full question)

The nurse is conducting a health education program on bladder cancer. The nurse teaches that which of the following is the most common type of bladder cancer?

You selected:

Transitional carcinoma

Correct Explanation:

The most common type of bladder cancer is transitional carcinoma, also known as urothelial carcinoma.

Question 1:

Which of the following clinical manifestations would tell a nurse that a patient is having progressive decompensation related to obstruction of urinary outflow?

(see full question)

You selected:

Correct

When tested for residual urine volume, 1400 mL of urine is obtained when patient is catheterized.


Explanation:

When compensatory mechanisms are no longer effective, signs of decompensation begin to appear. The period of detrusor muscle contraction becomes too short to expel the urine comple ... (more)

Reference:

Grossman S, Porth CM (2014). Porth’s Pathophysiology: Concepts of Altered Health States. 9th ed. Philadelphia: Lippincott Williams & Wilkins, Chapter 43, Disorders of the Bladder and Lower Urinary Tract, p. 1138.

Question 2:

While studying the GU system the nursing students learn that which of the following group achieves continence first?

(see full question)

You selected:

Girls

Correct Explanation:

Girls typically achieve continence before boys and bowel control is typically achieved before bladder control.

Reference:

Grossman S, Porth CM (2014). Porth’s Pathophysiology: Concepts of Altered Health States. 9th ed. Philadelphia: Lippincott Williams & Wilkins, Chapter 43, Disorders of the Bladder and Lower Urinary Tract, p. 1135.

Question 3:

Many factors contribute to the incontinence that is common among the elderly. A major factor is increased:

(see full question)

You selected:

Use of multiple medications

Correct Explanation:

Use of multiple medications for other health problems can affect bladder function, especially diuretics. Drugs such as hypnotics, tranquilizers, and sedatives can interfere with th ... (more)

Reference:

Grossman S, Porth CM (2014). Porth’s Pathophysiology: Concepts of Altered Health States. 9th ed. Philadelphia: Lippincott Williams & Wilkins, Chapter 43, Disorders of the Bladder and Lower Urinary Tract, pp. 1145-1146.


Question 4: (see full question)

Children usually achieve bladder control by age 5 years. Girls generally achieve bladder control before boys do. What is the general rule for bladder capacity in a child?

You selected:

The capacity of the bladder is equal to the child’s age in years.

Incorrect Correct response:

Up to the age of 12 to 14 years, the capacity of the bladder is the child’s age in years plus 2.

Explanation:

As the child grows, the bladder gradually enlarges, with an increase in capacity, in ounces, that approximates the age of the child plus 2. The other answers are not true ..... (more)

Reference:

Grossman S, Porth CM (2014). Porth's Pathophysiology: Concepts of Altered Health States. 9th ed. Philadelphia: Lippincott Williams & Wilkins, Chapter 43: Disorders of the Bladder and Lower Urinary Tract, p. 1135.

Question 5:

The client who has just been diagnosed with bladder cancer asks the nurse what treatment he will have to undergo. Which of the following would be the nurse's best response to this question?

(see full question)

You selected:

• "The methods used depend on the grade of the tumor and the lesion's invasiveness." • "You will need to ask your physician."

Incorrect Correct response:

• "The methods used depend on the grade of the tumor and the lesion's invasiveness."

Explanation:

The methods of treatment for bladder cancer depend upon grade of the tumor and the lesion's degree of invasiveness. The other answers are not therapeutic and telling the client it' ... (more)

Reference:

Grossman S, Porth CM (2014). Porth’s Pathophysiology: Concepts of Altered Health States. 9th ed. Philadelphia: Lippincott Williams & Wilkins, Chapter 43, Disorders of the Bladder and Lower Urinary Tract, p. 1147.


Question 6: (see full question)

You selected:

A female client asks, “Why do I leak urine every time I cough or sneeze?” The health care worker’s response is based on which physiologic principle? A pressure difference between the urethra and bladder

Incorrect Correct response:

When intravesical pressure exceeds maximal urethral closure pressure

Explanation:

Stress incontinence represents the involuntary loss of urine that occurs when, in the absence of detrusor muscle action, the intravesical pressure exceeds the maximum urethral clos ... (more)

Reference:

Grossman, S. & Porth, C.M. Porth's Pathophysiology: Concepts of Altered Health States, 9th ed., Philadelphia: Lippincott Williams & Wilkins, 2014, Chapter 43, Disorders of the Bladder and Lower Urinary Tract, p. 1143.

Question 7: (see full question)

The nursing instructor who is teaching about incontinence in the elderly recognizes a need for further instruction when one of the students makes which of the following statements?

You selected:

"Frequency is not a major problem for the elderly."

Correct Explanation:

Incontinence can increase social isolation in the older adult population and frequency can lead to institutionalization of older adults. Many factors can contribute to incontinence ... (more)

Reference:

Grossman S, Porth CM (2014). Porth’s Pathophysiology: Concepts of Altered Health States. 9th ed. Philadelphia: Lippincott Williams & Wilkins, Chapter 43, Disorders of the Bladder and Lower Urinary Tract, p. 1145.

Question 8:

Which of the following accurately describes the etiology of stress incontinence?

(see full question)

You selected:

An increase in intra-abdominal pressure which results in involuntary urination

Correct Explanation:

Stress incontinence results in involuntary passage of urine related to increased intra-abdominal pressure with coughing, sneezing or laughing. The


other options refer to the etiolo ... (more)

Reference:

Grossman S, Porth CM (2014). Porth’s Pathophysiology: Concepts of Altered Health States. 9th ed. Philadelphia: Lippincott Williams & Wilkins, Chapter 43, Disorders of the Bladder and Lower Urinary Tract, p. 1142.

Question 9:

In anatomy class, the instructor asks, “Explain how urine is expelled from the bladder during voiding.” The student with the most accurate response would be:

(see full question)

You selected:

“The detrusor muscle contract down on the urine and the ureteral orifices are forced shut. The external sphincter relaxes as urine moves out of the bladder.”

Correct Explanation:

During the act of micturition, the detrusor muscle of the bladder fundus and bladder neck contracts down on the urine and the ureteral orifices are forced shut. The bladder neck is ... (more)

Reference:

Grossman, S. & Porth, C.M. Porth's Pathophysiology: Concepts of Altered Health States, 9th ed., Philadelphia: Lippincott Williams & Wilkins, 2014, Chapter 43, Disorders of Bladder and Lower Urinary Tract, p. 1135.

Question 10: (see full question)

A nurse is caring for a patient admitted for chemotherapy due to bladder cancer. Which route would the nurse anticipate that the chemotherapy will be administered when the goal is to minimize systemic side effects?

You selected:

Intravesical

Correct Explanation:

In many cases, chemotherapeutic or immunotherapeutic agents can be administered intravesically (instilled directly into the bladder), thereby avoiding the side effects of systemic ... (more)

Question 1:

The nurse is conducting a community health education program on urinary retention and urinary incontinence. The nurse determines that the participants

(see full question)


are understanding the education when they state that the most common cause of urinary retention is which of the following? You selected:

Prostate enlargement

Correct Explanation:

In men, the enlarged prostate (due to hypertrophy or hyperplasia) frequently causes nonrelaxing external sphincter with urine retention. The stress response ca ... (more)

Reference:

Grossman S, Porth CM (2014). Porth’s Pathophysiology: Concepts of Altered Health States. 9th ed. Philadelphia: Lippincott Williams & Wilkins, Chapter 43, Disorders of the Bladder and Lower Urinary Tract, p. 1138.

Question 2:

While studying about the process of urination, the nursing student learns that which of the following is known as the "muscle of micturition"?

(see full question)

You selected:

Detrusor muscle

Correct Explanation:

The detrusor muscle is the uncle of micturition. In the bladder neck is the continuation of the detrusor muscle known as the internal urethral sphincter, which acts as a sphincter ... (more)

Reference:

Grossman S, Porth CM (2014). Porth’s Pathophysiology: Concepts of Altered Health States. 9th ed. Philadelphia: Lippincott Williams & Wilkins, Chapter 43, Disorders of the Bladder and Lower Urinary Tract, p. 1133.

Question 3: (see full question)

In women, stress incontinence is a common problem. The loss of the angle between the urethrovesical junction and the bladder contributes to stress incontinence. What is the normal angle between the bladder and the urethrovesical junction?

You selected:

90 to 100 degrees

Correct Explanation:

In women, the angle between the bladder and the posterior proximal urethra


(i.e., urethrovesical junction) is important to continence. This angle normally is 90 to 100 degree ... (more)

Reference:

Grossman S, Porth CM (2014). Porth's Pathophysiology: Concepts of Altered Health States. 9th ed. Philadelphia: Lippincott Williams & Wilkins, Chapter 43: Disorders of the Bladder and Lower Urinary Tract, p. 1142.

Question 4:

An elderly woman comes to the hospital and is diagnosed with urinary obstruction and retention. Which of the following symptoms would the nurse expect this client to demonstrate? Select all that apply.

(see full question)

You selected:

• Hesitancy • Overflow incontinence • Nladder distention • Frequency

Correct Explanation:

Signs of outflow obstruction and urine retention are: bladder distention, hesitancy, straining when initiating urination, small and weak stream, frequency, feeling of incomplete bl ... (more)

Reference:

Grossman S, Porth CM (2014). Porth’s Pathophysiology: Concepts of Altered Health States. 9th ed. Philadelphia: Lippincott Williams & Wilkins, Chapter 43, Disorders of the Bladder and Lower Urinary Tract, p. 1139.

Question 5:

An instructor is assisting a nursing student with inserting an indwelling catheter for a patient with urinary retention and acute overdistention of the bladder. The student inserts the catheter and gets an immediate return of clear yellow urine. When should the student clamp the catheter?

(see full question)

You selected:

When the patient returns 1000 mL of urine from the bladder at once

Correct Explanation:

With acute overdistention of the bladder, usually no more than 1000 mL of urine is removed from the bladder at one time. The theory behind this limitation is that removing more tha ... (more)


Reference:

Grossman S, Porth CM (2014). Porth’s Pathophysiology: Concepts of Altered Health States. 9th ed. Philadelphia: Lippincott Williams & Wilkins, Chapter 43, Disorders of the Bladder and Lower Urinary Tract, p. 1141.

Question 6:

Although urinary obstruction and urinary incontinence have almost opposite effects on urination, they can both result from:

(see full question)

You selected:

Bladder structure changes

Correct Explanation:

Disorders of lower urinary tract structure and function include urinary obstruction with retention or stasis of urine and urinary incontinence with involuntary loss of urine. Both ... (more)

Reference:

Grossman S, Porth CM (2014). Porth’s Pathophysiology: Concepts of Altered Health States. 9th ed. Philadelphia: Lippincott Williams & Wilkins, Chapter 43, Disorders of the Bladder and Lower Urinary Tract, p. 1137.

Question 7:

The nurse working on a GU floor caring for clients who have bladder cancer identifies which of the following to be acceptable treatments for this cancer? Select all that apply.

(see full question)

You selected:

• Radiation therapy • Chemotherapy • Surgical removal

Correct Explanation:

The methods used in treatment depend upon the grade of the tumor and degree of invasiveness. They include surgical removal, radiation therapy, and chemotherapy. Hypnosis and herbal ... (more)

Reference:

Grossman S, Porth CM (2014). Porth’s Pathophysiology: Concepts of Altered Health States. 9th ed. Philadelphia: Lippincott Williams & Wilkins, Chapter 43, Disorders of the Bladder and Lower Urinary Tract, p. 1147.

Question 8:

A newly diagnosed paraplegic client who suffered an automobile accident


(see full question)

appears to have control of bladder emptying. The health care provider explains this process to the client/family stating, “This function is allowing the motor component of the neural reflex to assist with bladder emptying and is primarily controlled by the:

You selected:

Parasympathetic division of the ANS.“

Correct Explanation:

The motor component of the neural reflex to assist with bladder emptying is primarily controlled by the parasympathetic division of the ANS, and the relaxation and storage function ... (more)

Reference:

Grossman, S. & Porth, C.M. Porth's Pathophysiology: Concepts of Altered Health States, 9th ed., Philadelphia: Lippincott Williams & Wilkins, 2014, Chapter 17, Organization and Control of Neural Function, p. 413.

Question 9:

A client asks the nurse what the most common sign/symptom of bladder cancer is. Which is the best response by the nurse?

(see full question)

You selected:

Painless hematuria

Correct Explanation:

The most common sign of bladder cancer is painless hematuria. Gross hematuria is a presenting sign in 75% of persons with the disease, and microscopic hematuria is present in most ... (more)

Reference:

Grossman S, Porth CM (2014). Porth’s Pathophysiology: Concepts of Altered Health States. 9th ed. Philadelphia: Lippincott Williams & Wilkins, Chapter 43, Disorders of the Bladder and Lower Urinary Tract, p. 1147.

Question 10:

A client who is experiencing bladder overfilling will more than likely have which of the following disease? Select all that apply.

(see full question)

You selected:

Incorrect

• Stroke • Spinal cord injury • Diabetic neuropathies


Correct response:

• Diabetic neuropathies • Multiple sclerosis

Explanation:

Common causes of bladder overfilling that lead to a loss of ability to perceive bladder filling are diabetic neuropathies and multiple sclerosis. Spinal cord injury results in a lo ... (more)

Question 1: (see full question)

A client has been diagnosed with cholecystitis (gallbladder inflammation) that has impaired the normal release of bile. Which of the following gastrointestinal consequences is this client likely to experience?

You selected:

Impaired digestion of fats

Correct Explanation:

Bile performs a central role in fat metabolism. Gallbladder disease and the accompanying disruption of normal bile release do not result in impaired digestion of carbohydrates, imp ... (more)

Reference:

Grossman S, Porth CM (2014). Porth’s Pathophysiology: Concepts of Altered Health States. 9th ed. Philadelphia: Lippincott Williams & Wilkins, Chapter 44, Structure and Function of the Gastrointestinal System, p. 1167.

Question 2:

A student is comparing the different parts of the gastrointestinal (GI) tract. Which of the following statements demonstrates understanding?

(see full question)

You selected:

"The mouth is the receptacle where initial digestive processes take place."

Correct Explanation:

The GI tract can be divided into three parts: upper, middle, and lower segments. The upper part contains the mouth and esophagus and acts as a receptacle through which food passes ... (more)

Reference:

Grossman S, Porth CM (2014). Porth’s Pathophysiology: Concepts of Altered Health States. 9th ed. Philadelphia: Lippincott Williams & Wilkins, Chapter 44, Structure and Function of the Gastrointestinal System, p. 1151.

Question 3:

While assessing a client who is experiencing diarrhea caused by Clostridium


(see full question)

difficile, the nurse should anticipate hearing:

You selected:

Hyperactive bowel sounds

Correct Explanation:

Inflammatory changes increase motility, which would lead to hyperactive bowel sounds. In many instances, it is not certain whether changes in motility occur because of inflammation ... (more)

Reference:

Grossman, S. & Porth, C.M. Porth's Pathophysiology: Concepts of Altered Health States, 9th ed., Philadelphia: Lippincott Williams & Wilkins, 2014, Chapter 44, Structure and Function of the Gastrointestinal System, p. 1159.

Question 4:

Which of the following functions is performed by saliva?

(see full question)

You selected:

Antimicrobial protection

Correct Explanation:

Saliva performs three roles: lubrication, antimicrobial protection, and initiation of starch digestion. Saliva does not promote the growth of intestinal flora, buffer gastric secre ... (more)

Reference:

Grossman S, Porth CM (2014). Porth’s Pathophysiology: Concepts of Altered Health States. 9th ed. Philadelphia: Lippincott Williams & Wilkins, Chapter 44, Structure and Function of the Gastrointestinal System, p. 1163.

Question 5:

Which of the following is the primary purpose of the mucosal barrier in the gastrointestinal tract?

(see full question)

You selected:

Protection from acid secretion

Correct Explanation:

The primary purpose of gastric mucosa is resistance to the highly acidic secretion that it produces.

Reference:

Grossman S, Porth CM (2014). Porth’s Pathophysiology: Concepts of Altered


Health States. 9th ed. Philadelphia: Lippincott Williams & Wilkins, Chapter 43, Disorders of Gastrointestinal Function, p. 1141.

(see full question)

A nurse is concerned about a patient's continual use of nonsteroidal antiinflammatory drugs (NSAIDs). Which of the following is the nurse concerned about?

You selected:

Damage to the gastric mucosa

Question 6:

Correct Explanation:

One of the important characteristics of the gastric mucosa is resistance to highly acidic secretions that it produces. However, when aspirin, NSAIDs, Heliocbacter pylori, ethyl al ... (more)

Reference:

Grossman S, Porth CM (2014). Porth’s Pathophysiology: Concepts of Altered Health States. 9th ed. Philadelphia: Lippincott Williams & Wilkins, Chapter 44, Structure and Function of the Gastrointestinal System, p. 1163.

Question 7:

A patient tells the nurse that he is having diarrhea. Which of the following would the nurse recommend to decrease the diarrhea?

(see full question)

You selected:

Electrolyte drink containing sodium and glucose

Correct Explanation:

Water absorption from the intestines is linked to osmotically active particles, such as glucose and sodium. It follows that an important consideration in facilitating the transport ... (more)

Reference:

Grossman S, Porth CM (2014). Porth’s Pathophysiology: Concepts of Altered Health States. 9th ed. Philadelphia: Lippincott Williams & Wilkins, Chapter 44, Structure and Function of the Gastrointestinal System, p. 1167.

Question 8:

Colonic microorganisms play a role in the synthesis of which of the following vitamins?

(see full question)

You selected:

K


Correct Explanation:

Colonic microorganisms play a role in vitamin synthesis and in absorption of calcium, magnesium, and iron. Colonic flora synthesize vitamin K.

Reference:

Grossman S, Porth CM (2014). Porth’s Pathophysiology: Concepts of Altered Health States. 9th ed. Philadelphia: Lippincott Williams & Wilkins, Chapter 44, Structure and Function of the Gastrointestinal System, p. 1165.

Question 9: (see full question)

A client who has had his gallbladder (GB) removed asks the nurse, “Why do I feel sick every time I go through a drive-in fast-food restaurant and order burgers and fries?” The nurse bases the response knowing that:

You selected:

Bile from the GB is needed to breakdown lipids.

Correct Explanation:

The common bile duct and pancreatic duct empty their juices into the duodenum. Bile, a fluid synthesized by the liver that breaks down lipids, and pancreatic juices, which facilita ... (more)

Reference:

Grossman, S. & Porth, C.M. Porth's Pathophysiology: Concepts of Altered Health States, 9th ed., Philadelphia: Lippincott Williams & Wilkins, 2014, Chapter 44, Structure and Function of the Gastrointestinal System, p. 1152.

Question 10:

A student is comparing the actions of the gastrointestinal hormones. Which of the following hormones acts as an appetite-stimulating signal?

(see full question)

You selected:

Ghrelin

Correct Explanation:

Ghrelin stimulates secretion of growth hormone and acts as a appetitestimulating signal from the stomach. GLP-1 decreases appetite. The other options do not have an effect on ap ... (more)

Question 1:

Which enzyme is found in the mouth and is the first to initiate the breakdown of starches?

(see full question)


You selected:

Amylase

Correct Explanation:

Digestion of starch begins in the mouth with the action of amylase. Fat emulsification is influenced by bile. Protein digestion begins in the stomach with the action of pepsin and ... (more)

Reference:

Grossman S, Porth CM (2014). Porth’s Pathophysiology: Concepts of Altered Health States. 9th ed. Philadelphia: Lippincott Williams & Wilkins, Chapter 44, Structure and Function of the Gastrointestinal System, p. 1166.

Question 2: (see full question)

A patient takes 650 mg of aspirin every 4 hours daily for complaints of joint pain. Which of the following statements should be included in the patient's teaching plan?

You selected:

“This medication can damage gastric mucosa.“

Correct Explanation:

Aspirin and nonsteroidal anti-inflammatory drugs can damage the gastric mucosa. Partial thromboplastin is not affected, nor is hemoglobin or hematocrit. The medication should help ... (more)

Reference:

Grossman S, Porth CM (2014). Porth’s Pathophysiology: Concepts of Altered Health States. 9th ed. Philadelphia: Lippincott Williams & Wilkins, Chapter 43, Disorders of Gastrointestinal Function, p. 1141.

Question 3:

Which of the following carbohydrates is matched to its correct enzyme needed for digestion?

(see full question)

You selected:

Sucrose and sucrase

Correct Explanation:

Sucrose is a dietary carbohydrate digested by sucrase, an enzyme. This produces the monosaccharides fructose and glucose.

Reference:

Grossman S, Porth CM (2014). Porth’s Pathophysiology: Concepts of Altered


Health States. 9th ed. Philadelphia: Lippincott Williams & Wilkins, Chapter 43, Disorders of Gastrointestinal Function, p. 1167.

Question 4:

Which of the following statements is true concerning food digestion?

(see full question)

You selected:

Pepsin helps to digest food in the small intestine.

Incorrect Correct response:

Protein digestion begins in the stomach.

Explanation:

Protein digestion begins in the stomach with the action of pepsin. Pepsin cannot act in the small intestine, as it is broken down by the alkaline pH in the gastro intestinal tract...... (more)

Reference:

Grossman S, Porth CM (2014). Porth’s Pathophysiology: Concepts of Altered Health States. 9th ed. Philadelphia: Lippincott Williams & Wilkins, Chapter 43, Disorders of Gastrointestinal Function, p. 1168.

Question 5:

While reviewing the colonic absorption and role of flora in the GI system, the instructor will stress that the large intestine contains:

(see full question)

You selected:

a complex microbial system that contains hundreds of different species.

Correct Explanation:

The stomach and small intestine contain only a few species of bacteria, probably because the composition of luminal contents (i.e., acids, bile, pancreatic secretions) kills most i ... (more)

Reference:

Grossman S, Porth CM (2014). Porth’s Pathophysiology: Concepts of Altered Health States. 9th ed. Philadelphia: Lippincott Williams & Wilkins, Chapter 44, Structure and Function of the Gastrointestinal System, p. 1165.

Question 6:

Digestion of starch begins in which of the following structures?

(see full question)

You selected:

Mouth


Correct Explanation:

Digestion of starch begins in the mouth by the action of amylase.

Reference:

Grossman S, Porth CM (2014). Porth’s Pathophysiology: Concepts of Altered Health States. 9th ed. Philadelphia: Lippincott Williams & Wilkins, Chapter 44, Structure and Function of the Gastrointestinal System, p. 1166.

Question 7:

Which of the following statements most accurately describes the function of the secretory glands in the gastrointestinal (GI) tract?

(see full question)

You selected:

The secretory glands produce mucus to lubricate and protect the mucosal layer of the GI tract wall.

Correct Explanation:

Secretory glands in the GI tract serve two basic functions: production of mucus to lubricate and protect the GI tract wall and secretion of fluids and enzymes to aid in digestion a ... (more)

Reference:

Grossman S, Porth CM (2014). Porth’s Pathophysiology: Concepts of Altered Health States. 9th ed. Philadelphia: Lippincott Williams & Wilkins, Chapter 44, Structure and Function of the Gastrointestinal System, p. 1162.

Question 8:

Which substance necessary for vitamin B12 absorption is produced by the parietal cells in the stomach?

(see full question)

You selected:

Intrinsic factor

Correct Explanation:

Intrinsic factor is produced by the parietal cells in the stomach and is necessary for the absorption of vitamin B12.

Reference:

Grossman S, Porth CM (2014). Porth’s Pathophysiology: Concepts of Altered Health States. 9th ed. Philadelphia: Lippincott Williams & Wilkins, Chapter 44, Structure and Function of the Gastrointestinal System, p. 1166.


Question 9: (see full question)

You selected:

A patient asks the nurse what part of the brain regulates appetite. Which is the best response by the nurse? Midbrain

Incorrect Correct response:

Hypothalamus

Explanation:

Appetite or the desire for food is regulated by the hypothalamus and other associated parts of the brain. The medulla deals with the autonomic (involuntary) functions of breathing, ... (more)

Reference:

Grossman S, Porth CM (2014). Porth’s Pathophysiology: Concepts of Altered Health States. 9th ed. Philadelphia: Lippincott Williams & Wilkins, Chapter 44, Structure and Function of the Gastrointestinal System, p. 1156.

Question 10:

The swallowing reflex is an entirely voluntary activity.

(see full question)

You selected:

False

Correct Explanation:

Although swallowing is initiated as a voluntary activity, it becomes involuntary as food or fluid reaches the pharynx.

Question 1:

A patient is not able to absorb vitamin B12. The nurse determines that the patient is deficient in:

(see full question)

You selected:

Parietal (oxyntic) cells, which secrete HCl and intrinsic factor

Correct Explanation:

Parietal (oxyntic) cells secrete HCl and intrinsic factor, which is needed for vitamin B12 absorption. Mucous neck cells secrete mainly mucus; peptic (chief) cells, which secrete l ... (more)

Reference:

Grossman, S. & Porth, C.M. Porth's Pathophysiology: Concepts of Altered Health States, 9th ed., Philadelphia: Lippincott Williams & Wilkins, 2014


Question 2:

Which of the following statements is true concerning gastric enterocytes?

(see full question)

You selected:

They secrete lubrication for the gastrointestinal tract.

Incorrect Correct response:

They secrete enzymes that aid in digestion of proteins.

Explanation:

Gastric enterocytes secrete enzymes that aid in the digestion of carbohydrates and proteins.

Reference:

Grossman S, Porth CM (2014). Porth’s Pathophysiology: Concepts of Altered Health States. 9th ed. Philadelphia: Lippincott Williams & Wilkins, Chapter 43, Disorders of Gastrointestinal Function, p. 1166.

Question 3:

Which of the following clients likely faces the greatest risk of a gastrointestinal bleed?

(see full question)

You selected:

A client who takes aspirin with each meal to control symptoms of osteoarthritis

Correct Explanation:

The gastric mucosa can be easily damaged by drugs such as aspirin, resulting in local ischemia, vascular stasis, hypoxia, and tissue necrosis. Antihypertensives, diuretics, antibio ... (more)

Reference:

Grossman S, Porth CM (2014). Porth’s Pathophysiology: Concepts of Altered Health States. 9th ed. Philadelphia: Lippincott Williams & Wilkins, Chapter 44, Structure and Function of the Gastrointestinal System, p. 1164.

Question 4:

The results of a patient’s 24-hour stool specimen indicate 20 g or more of fat. The nurse would interpret this as:

(see full question)

You selected:

Hyperlipidemia

Incorrect Correct response:

Steatorrhea

Explanation:

Steatorrhea is the term used to describe fatty stools. It usually indicates that


there is 20 g or more of fat in a 24-hour stool sample.

Reference:

Grossman, S. & Porth, C.M. Porth's Pathophysiology: Concepts of Altered Health States, 9th ed., Philadelphia: Lippincott Williams & Wilkins, 2014, Chapter 44, Structure and Function of the Gastrointestinal System, p. 1167.

Question 5:

While studying the esophagus, a student is able to correctly state which of the following to a faculty member?

(see full question)

You selected:

"The smooth muscle layers provide the peristaltic movements needed to move food along the length of the esophagus."

Correct Explanation:

The smooth muscle layers provide the peristaltic movements needed to move food along the length of the esophagus. The pharyngoesophageal sphincter, the upper sphincter, keeps air f ... (more)

Reference:

Grossman S, Porth CM (2014). Porth’s Pathophysiology: Concepts of Altered Health States. 9th ed. Philadelphia: Lippincott Williams & Wilkins, Chapter 44, Structure and Function of the Gastrointestinal System, p. 1152.

Question 6:

Fat that is not absorbed in the intestine is excreted in the stool. Which of the following terms is used to describe fatty stools?

(see full question)

You selected:

Steatorrhea

Correct Explanation:

Fat that is not absorbed in the intestine is excreted in the stool. Steatorrhea is the term used to describe fatty stools. Chyme is produced in the stomach as proteins are broken d ... (more)

Reference:

Grossman S, Porth CM (2014). Porth’s Pathophysiology: Concepts of Altered Health States. 9th ed. Philadelphia: Lippincott Williams & Wilkins, Chapter 44, Structure and Function of the Gastrointestinal System, p. 1167.


Question 7:

Which enzymes are responsible for the breakdown of protein?

(see full question)

You selected:

Trypsin and pepsin

Correct Explanation:

The digestion of protein begins in the stomach with the action of pepsin and is facilitated in the intestine by the enzymes trypsin, chymotrypsin, carboxypeptidase, and elastase. A ... (more)

Reference:

Grossman S, Porth CM (2014). Porth’s Pathophysiology: Concepts of Altered Health States. 9th ed. Philadelphia: Lippincott Williams & Wilkins, Chapter 44, Structure and Function of the Gastrointestinal System, p. 1168.

Question 8: (see full question)

During a lecture about the function of the intestine related to food digestion, the faculty mentions that when the students consume foods high in acid, the intestines will:

You selected:

stimulate the release of secretin which then inhibits release of gastrin.

Correct Explanation:

Secretin, which is secreted by S cells in the mucosa of the duodenum and jejunum, inhibits gastric acid secretion. The entry of an acid chyme into the intestine stimulates the rele ... (more)

Reference:

Grossman S, Porth CM (2014). Porth’s Pathophysiology: Concepts of Altered Health States. 9th ed. Philadelphia: Lippincott Williams & Wilkins, Chapter 44, Structure and Function of the Gastrointestinal System, p. 161.

Question 9:

Place the following steps relating to swallowing in order.

(see full question)

You selected:

Incorrect

The tongue moves the food to the back of the mouth. The soft palate is pulled upward and the nasopharynx is blocked. The walls of the esophagus are stretched. The food is lifted to the posterior wall of the pharynx. The pharynx uses involuntary, constrictive movements to move the food. Local and central nervous system reflexes initiate peristalsis.


Correct response:

The tongue moves the food to the back of the mouth. The food is lifted to the posterior wall of the pharynx. The soft palate is pulled upward and the nasopharynx is blocked. The pharynx uses involuntary, constrictive movements to move the food. The walls of the esophagus are stretched. Local and central nervous system reflexes initiate peristalsis.

Explanation:

Food is moved to the back of the throat by the tongue. The tongue then lifts the food to the posterior wall of the pharynx. The soft palate is pulled upward and respiration is inh ... (more)

Reference:

Grossman S, Porth CM (2014). Porth’s Pathophysiology: Concepts of Altered Health States. 9th ed. Philadelphia: Lippincott Williams & Wilkins, Chapter 44, Structure and Function of the Gastrointestinal System, p. 1156.

Question 10: (see full question)

The physiological rationale for hanging normal saline (0.9% NS) or 5% dextrose in water (D5W) to a client who has been experiencing diarrhea includes:

You selected:

Facilitating the absorption of osmotically active particles

Correct Explanation:

Water absorption from the intestine is linked to absorption of osmotically active particles, such as glucose and sodium. It follows that an important consideration in facilitating ... (more)

Question 1:

A patient asks the nurse what part of the brain regulates appetite. Which is the best response by the nurse?

(see full question)

You selected:

Hypothalamus

Correct Explanation:

Appetite or the desire for food is regulated by the hypothalamus and other associated parts of the brain. The medulla deals with the autonomic (involuntary) functions of breathing, ... (more)

Reference:

Grossman S, Porth CM (2014). Porth’s Pathophysiology: Concepts of Altered Health States. 9th ed. Philadelphia: Lippincott Williams & Wilkins, Chapter 44, Structure and Function of the Gastrointestinal System, p. 1156.


(see full question)

A nurse explains to her patient that food is moved along the gastrointestinal (GI) tract with intermittent contractions that mix the food and move it along. These movements are found in which of the following organs?

You selected:

Small intestine

Question 2:

Correct Explanation:

The movements of the GI tract can be either rhythmic or tonic. Rhythmic movements consist of intermittent contractions that are responsible for mixing and moving food along the dig ... (more)

Reference:

Grossman S, Porth CM (2014). Porth’s Pathophysiology: Concepts of Altered Health States. 9th ed. Philadelphia: Lippincott Williams & Wilkins, Chapter 44, Structure and Function of the Gastrointestinal System, p. 1155.

Question 3:

A client who is quadriplegic following a motor vehicle accident adheres to a bowel protocol to promote regular bowel movements and prevent constipation. Which of the following actions performed by the client's caregiver is likely to promote defecation?

(see full question)

You selected:

Seating the client in an upright position

Incorrect Correct response:

Digital stimulation of the client's rectum

Explanation:

When the nerve endings in the rectum are stimulated, signals are transmitted first to the sacral cord and then reflexively back to the descending colon, sigmoid colon, rectum, and ... (more)

Reference:

Grossman S, Porth CM (2014). Porth’s Pathophysiology: Concepts of Altered Health States. 9th ed. Philadelphia: Lippincott Williams & Wilkins, Chapter 44, Structure and Function of the Gastrointestinal System, pp. 1160-1161.

Question 4:

A patient is experiencing reflux of stomach contents into the esophagus. The nurse determines that the problem may result from:

(see full question)


You selected:

Gastroesophageal sphincter

Correct Explanation:

The gastroesophageal sphincter (also called the lower esophageal sphincter) prevents the contents of the stomach from rising into the esophagus. The upper esophageal sphincter (als ... (more)

Reference:

Grossman S, Porth CM (2014). Porth’s Pathophysiology: Concepts of Altered Health States. 9th ed. Philadelphia: Lippincott Williams & Wilkins, Chapter 44, Structure and Function of the Gastrointestinal System, p. 1152.

Question 5:

Which of the following statements is true concerning food digestion?

(see full question)

You selected:

Pepsin helps to digest food in the small intestine.

Incorrect Correct response:

Protein digestion begins in the stomach.

Explanation:

Protein digestion begins in the stomach with the action of pepsin. Pepsin cannot act in the small intestine, as it is broken down by the alkaline pH in the gastro intestinal tract...... (more)

Reference:

Grossman S, Porth CM (2014). Porth’s Pathophysiology: Concepts of Altered Health States. 9th ed. Philadelphia: Lippincott Williams & Wilkins, Chapter 43, Disorders of Gastrointestinal Function, p. 1168.

Question 6:

Which of the following structures in the gastrointestinal (GI) tract has absorptive function focusing mainly on water resorption?

(see full question)

You selected:

Large instestine

Correct Explanation:

Absorption is the process of moving nutrients and other materials from the external environment in the lumen of the GI tract into the internal environment. The absorptive function ... (more)


Reference:

Grossman S, Porth CM (2014). Porth’s Pathophysiology: Concepts of Altered Health States. 9th ed. Philadelphia: Lippincott Williams & Wilkins, Chapter 44, Structure and Function of the Gastrointestinal System, p. 1166.

Question 7:

The layer of the gastrointestinal (GI) tract that facilitates movement of contents in the GI tract is known as which of the following?

(see full question)

You selected:

Muscularis externa

Correct Explanation:

The third layer, the muscularis externa, facilitates movement of contents of the GI tract. The lamina propria and muscularis mucosae are layers in the first (mucosal) layer. The se ... (more)

Reference:

Grossman S, Porth CM (2014). Porth’s Pathophysiology: Concepts of Altered Health States. 9th ed. Philadelphia: Lippincott Williams & Wilkins, Chapter 44, Structure and Function of the Gastrointestinal System, p. 1153.

Question 8:

Which of the following would help a nurse best explain an "incretin effect"?

(see full question)

You selected:

"It is an increase in insulin release after ingestion of food."

Correct Explanation:

An incretin effect means that there is an increase in insulin release after an oral glucose load or food ingestion. The other answers are incorrect.

Reference:

Grossman S, Porth CM (2014). Porth’s Pathophysiology: Concepts of Altered Health States. 9th ed. Philadelphia: Lippincott Williams & Wilkins, Chapter 43, Disorders of Gastrointestinal Function, p. 1141.

Question 9: (see full question)

A patient with pancreatitis has decreased production of pancreatic enzymes such as trypsin. The dietitian knows that the patient may have difficulty digesting which of the following foods?

You selected:

Fried chicken and deviled eggs


Correct Explanation:

Proteins are broken down by pancreatic enzymes, such as trypsin, chymotrypsin, carboxypeptidase, and elastase. Of the meal options, fried chicken and deviled eggs have the highest ... (more)

Reference:

Grossman S, Porth CM (2014). Porth’s Pathophysiology: Concepts of Altered Health States. 9th ed. Philadelphia: Lippincott Williams & Wilkins, Chapter 44, Structure and Function of the Gastrointestinal System, p. 1168.

Question 10: (see full question)

Motility along the length of the GI tract, controlling the function of each segment of the GI tract and integrating signals for absorption of nutrients primarily, is innervated by which portion of the enteric nervous system?

You selected:

Sympathetic

Incorrect Correct response:

Submucosal plexus

Explanation:

The enteric nervous system consists of the intramural neurons (contained in the gastrointestinal tract) of two networks—myenteric and submucosal plexus. The submucosal plexus, wh ... (more)

Question 1:

Which of the following statements is true concerning gastric enterocytes?

(see full question)

You selected:

They secrete enzymes that aid in digestion of proteins.

Correct Explanation:

Gastric enterocytes secrete enzymes that aid in the digestion of carbohydrates and proteins.

Reference:

Grossman S, Porth CM (2014). Porth’s Pathophysiology: Concepts of Altered Health States. 9th ed. Philadelphia: Lippincott Williams & Wilkins, Chapter 43, Disorders of Gastrointestinal Function, p. 1166.

Question 2:

When acidic chyme enters the duodenum, it stimulates secretion of which of the following hormones?

(see full question)

You selected:

Gastrin


Incorrect Correct response:

Secretin

Explanation:

When stomach acid in chyme enters the intestine, it stimulates the release of secretin. Secretin inhibits gastrin secretion. Ghrelin is produced by endocrine cells in the fundus of ... (more)

Reference:

Grossman S, Porth CM (2014). Porth’s Pathophysiology: Concepts of Altered Health States. 9th ed. Philadelphia: Lippincott Williams & Wilkins, Chapter 44, Structure and Function of the Gastrointestinal System, pp. 1161-1162.

Question 3: (see full question)

The patient tells the nurse that she is very anxious and that her pulse is racing. Which of the following will the nurse expect to see in the gastrointestinal (GI) tract?

You selected:

Initiation of the second defecation reflex

Incorrect Correct response:

Dry mouth

Explanation:

Anxiety and a rapid heart rate indicate sympathetic nervous system activity that decreases the flow of secretions (dry mouth). Parasympathetic stimulation is responsible for the re ... (more)

Reference:

Grossman S, Porth CM (2014). Porth’s Pathophysiology: Concepts of Altered Health States. 9th ed. Philadelphia: Lippincott Williams & Wilkins, Chapter 44, Structure and Function of the Gastrointestinal System, p. 1163.

Question 4:

What body part is the largest serous membrane in the body, constitutes the outer wall of the intestine, and contains a serous fluid between its two layers?

(see full question)

You selected:

Mesentery

Incorrect Correct response:

Peritoneum

Explanation:

The peritoneum is the largest serous membrane, and constitutes the outer wall of the intestine, continuous with the mesentery. The greater omentum helps to prevent infection from e ... (more)


Reference:

Grossman S, Porth CM (2014). Porth’s Pathophysiology: Concepts of Altered Health States. 9th ed. Philadelphia: Lippincott Williams & Wilkins, Chapter 44, Structure and Function of the Gastrointestinal System, p. 1153.

Question 5: (see full question)

A pharmacology student is studying gastrointestinal medications. The student explains that proton pump inhibitors (PPIs) are used to inhibit secretion of which of the following?

You selected:

Gastric acid

Correct Explanation:

PPIs (also known as the H+/K+-ATPase transporter), which are used in the treatment of acid reflux and peptic ulcer, inhibit gastric acid secretion. The other options are not inhibit ... (more)

Reference:

Grossman S, Porth CM (2014). Porth’s Pathophysiology: Concepts of Altered Health States. 9th ed. Philadelphia: Lippincott Williams & Wilkins, Chapter 44, Structure and Function of the Gastrointestinal System, p. 1163.

Question 6:

A student is studying gallbladder function. Which of the following gastrointestinal hormones stimulates contraction of the gallbladder?

(see full question)

You selected:

Cholecystokinin

Correct Explanation:

Cholecystokinin, which is secreted in the duodenum and jejunum, stimulates contraction of the gallbladder and secretion of pancreatic enzymes.

Reference:

Grossman S, Porth CM (2014). Porth’s Pathophysiology: Concepts of Altered Health States. 9th ed. Philadelphia: Lippincott Williams & Wilkins, Chapter 44, Structure and Function of the Gastrointestinal System, p. 1162.

Question 7:

Gastrin production, a task that is performed by the stomach, results in which of


(see full question)

the following effects?

You selected:

Stimulation of HCl secretions by parietal cells

Correct Explanation:

The primary function of gastrin is the stimulation of gastric acid secretion. It does not have a direct effect on exocrine or endocrine pancreatic function, and it does not partici ... (more)

Reference:

Grossman S, Porth CM (2014). Porth’s Pathophysiology: Concepts of Altered Health States. 9th ed. Philadelphia: Lippincott Williams & Wilkins, Chapter 44, Structure and Function of the Gastrointestinal System, p. 1163.

Question 8:

The results of a patient’s 24-hour stool specimen indicate 20 g or more of fat. The nurse would interpret this as:

(see full question)

You selected:

Steatorrhea

Correct Explanation:

Steatorrhea is the term used to describe fatty stools. It usually indicates that there is 20 g or more of fat in a 24-hour stool sample.

Reference:

Grossman, S. & Porth, C.M. Porth's Pathophysiology: Concepts of Altered Health States, 9th ed., Philadelphia: Lippincott Williams & Wilkins, 2014, Chapter 44, Structure and Function of the Gastrointestinal System, p. 1167.

Question 9:

The dietitian explains to the patient that food is digested and absorbed in which of the following?

(see full question)

You selected:

Stomach

Incorrect Correct response:

Jejunum

Explanation:

Food is digested and absorbed in the jejunum and ileum. The stomach serves as a food reservoir during the early stages of digestion. The cecum and colon are parts of the large inte ... (more)


Reference:

Grossman S, Porth CM (2014). Porth’s Pathophysiology: Concepts of Altered Health States. 9th ed. Philadelphia: Lippincott Williams & Wilkins, Chapter 44, Structure and Function of the Gastrointestinal System, p. 1152.

Question 10: (see full question)

The patient chews an apple and starts the three phases of swallowing. As food enters the patient's esophagus, stretching and nerve impulses initiate that are controlled by the swallowing center.

You selected:

hypoglossal; contractions

Incorrect Correct response:

vagus; peristaltic waves

Explanation:

The esophageal phase is innervated by the vagus nerve, which initiates the peristaltic waves that will carry the food bolus into the stomach. The phrenic nerve is not involved with ... (more)

Question 1:

Which of the following statements most accurately describes the function of the secretory glands in the gastrointestinal (GI) tract?

(see full question)

You selected:

The secretory glands produce mucus to lubricate and protect the mucosal layer of the GI tract wall.

Correct Explanation:

Secretory glands in the GI tract serve two basic functions: production of mucus to lubricate and protect the GI tract wall and secretion of fluids and enzymes to aid in digestion a ... (more)

Reference:

Grossman S, Porth CM (2014). Porth’s Pathophysiology: Concepts of Altered Health States. 9th ed. Philadelphia: Lippincott Williams & Wilkins, Chapter 44, Structure and Function of the Gastrointestinal System, p. 1162.

Question 2:

A patient tells the nurse that he is having diarrhea. Which of the following would the nurse recommend to decrease the diarrhea?

(see full question)

You selected:

Electrolyte drink containing sodium and glucose


Correct Explanation:

Water absorption from the intestines is linked to osmotically active particles, such as glucose and sodium. It follows that an important consideration in facilitating the transport ... (more)

Reference:

Grossman S, Porth CM (2014). Porth’s Pathophysiology: Concepts of Altered Health States. 9th ed. Philadelphia: Lippincott Williams & Wilkins, Chapter 44, Structure and Function of the Gastrointestinal System, p. 1167.

Question 3:

Colonic microorganisms play a role in the synthesis of which of the following vitamins?

(see full question)

You selected:

K

Correct Explanation:

Colonic microorganisms play a role in vitamin synthesis and in absorption of calcium, magnesium, and iron. Colonic flora synthesize vitamin K.

Reference:

Grossman S, Porth CM (2014). Porth’s Pathophysiology: Concepts of Altered Health States. 9th ed. Philadelphia: Lippincott Williams & Wilkins, Chapter 44, Structure and Function of the Gastrointestinal System, p. 1165.

Question 4:

When explaining absorption to a client, the nurse mentions that a number of substances require a specific carrier or transport system. An example the nurse could use relates to the transport of amino acids and glucose, which requires the presence of which of the following for absorption to occur?

(see full question)

You selected:

Calcium

Incorrect Correct response:

Sodium

Explanation:

Absorption is accomplished by active transport and diffusion. A number of substances require a specific carrier or transport system. For example, transport of amino acids and gluco ... (more)


Reference:

Grossman, S. & Porth, C.M. Porth's Pathophysiology: Concepts of Altered Health States, 9th ed., Philadelphia: Lippincott Williams & Wilkins, 2014, Chapter 44, Structure and Function of the Gastrointestinal System, p. 1166.

Question 5: (see full question)

An anatomy student is studying the stomach. The student explains that the funnel-shaped portion that connects with the intestine is which of the following?

You selected:

Fundus

Incorrect Correct response:

Pyloric region

Explanation:

The funnel-shaped portion that connects with the small intestine is called the pyloric region. The small part of the stomach that surrounds the cardiac oriface is called the cardia ... (more)

Reference:

Grossman S, Porth CM (2014). Porth’s Pathophysiology: Concepts of Altered Health States. 9th ed. Philadelphia: Lippincott Williams & Wilkins, Chapter 44, Structure and Function of the Gastrointestinal System, p. 1152.

Question 6:

Which of the following carbohydrates is matched to its correct enzyme needed for digestion?

(see full question)

You selected:

Sucrose and sucrase

Correct Explanation:

Sucrose is a dietary carbohydrate digested by sucrase, an enzyme. This produces the monosaccharides fructose and glucose.

Reference:

Grossman S, Porth CM (2014). Porth’s Pathophysiology: Concepts of Altered Health States. 9th ed. Philadelphia: Lippincott Williams & Wilkins, Chapter 43, Disorders of Gastrointestinal Function, p. 1167.

Question 7:

A student is comparing the different parts of the gastrointestinal (GI) tract. Which of the following statements demonstrates understanding?

(see full question)


You selected:

"The mouth is the receptacle where initial digestive processes take place."

Correct Explanation:

The GI tract can be divided into three parts: upper, middle, and lower segments. The upper part contains the mouth and esophagus and acts as a receptacle through which food passes ... (more)

Reference:

Grossman S, Porth CM (2014). Porth’s Pathophysiology: Concepts of Altered Health States. 9th ed. Philadelphia: Lippincott Williams & Wilkins, Chapter 44, Structure and Function of the Gastrointestinal System, p. 1151.

Question 8:

Which of the following statements is true concerning food digestion?

(see full question)

You selected:

Protein digestion begins in the stomach.

Correct Explanation:

Protein digestion begins in the stomach with the action of pepsin. Pepsin cannot act in the small intestine, as it is broken down by the alkaline pH in the gastro intestinal tract...... (more)

Reference:

Grossman S, Porth CM (2014). Porth’s Pathophysiology: Concepts of Altered Health States. 9th ed. Philadelphia: Lippincott Williams & Wilkins, Chapter 43, Disorders of Gastrointestinal Function, p. 1168.

Question 9:

Which of the following functions is performed by saliva?

(see full question)

You selected:

Antimicrobial protection

Correct Explanation:

Saliva performs three roles: lubrication, antimicrobial protection, and initiation of starch digestion. Saliva does not promote the growth of intestinal flora, buffer gastric secre ... (more)

Reference:

Grossman S, Porth CM (2014). Porth’s Pathophysiology: Concepts of Altered Health States. 9th ed. Philadelphia: Lippincott Williams & Wilkins, Chapter 44,


Structure and Function of the Gastrointestinal System, p. 1163.

Question 10: (see full question)

You selected:

Bile and pancreatic juices enter which of the following through the common bile duct and the pancreatic duct? Duodenum

Correct Explanation:

The common bile duct and pancreatic duct empty their juices into the duodenum. The antrum is the lower portion of the stomach. Although the ileum and jejunum are also parts of the ... (more)

Question 1:

Which of the following has a surface area approximately equal to that of the skin?

(see full question)

You selected:

Mucosal layer

Incorrect Correct response:

Peritoneum

Explanation:

The peritoneum is the largest serous membrane in the body with a surface area approximately equal to that of the skin. The submucosal (second) layer consists of dense connective ti ... (more)

Reference:

Grossman S, Porth CM (2014). Porth’s Pathophysiology: Concepts of Altered Health States. 9th ed. Philadelphia: Lippincott Williams & Wilkins, Chapter 44, Structure and Function of the Gastrointestinal System, p. 1153.

Question 2:

What body part is the largest serous membrane in the body, constitutes the outer wall of the intestine, and contains a serous fluid between its two layers?

(see full question)

You selected:

Peritoneum

Correct Explanation:

The peritoneum is the largest serous membrane, and constitutes the outer wall of the intestine, continuous with the mesentery. The greater omentum helps to prevent infection from e ... (more)


Reference:

Grossman S, Porth CM (2014). Porth’s Pathophysiology: Concepts of Altered Health States. 9th ed. Philadelphia: Lippincott Williams & Wilkins, Chapter 44, Structure and Function of the Gastrointestinal System, p. 1153.

Question 3: (see full question)

During a lecture about the function of the intestine related to food digestion, the faculty mentions that when the students consume foods high in acid, the intestines will:

You selected:

stimulate the release of secretin which then inhibits release of gastrin.

Correct Explanation:

Secretin, which is secreted by S cells in the mucosa of the duodenum and jejunum, inhibits gastric acid secretion. The entry of an acid chyme into the intestine stimulates the rele ... (more)

Reference:

Grossman S, Porth CM (2014). Porth’s Pathophysiology: Concepts of Altered Health States. 9th ed. Philadelphia: Lippincott Williams & Wilkins, Chapter 44, Structure and Function of the Gastrointestinal System, p. 161.

Question 4:

The dietitian explains to the patient that food is digested and absorbed in which of the following?

(see full question)

You selected:

Jejunum

Correct Explanation:

Food is digested and absorbed in the jejunum and ileum. The stomach serves as a food reservoir during the early stages of digestion. The cecum and colon are parts of the large inte ... (more)

Reference:

Grossman S, Porth CM (2014). Porth’s Pathophysiology: Concepts of Altered Health States. 9th ed. Philadelphia: Lippincott Williams & Wilkins, Chapter 44, Structure and Function of the Gastrointestinal System, p. 1152.


Question 5: (see full question)

You selected:

A hospitalized patient has developed anorexia. The nurse is aware that this may result from which of the following? Select all that apply. • Anxiety • Fear • Depression

Incorrect Correct response:

• Smells • Fear • Anxiety • Medication • Depression

Explanation:

Anorexia represents a loss of appetite. Several factors influence appetite. Smell plays an important role, as evidenced by the fact that appetite can be stimulated or suppressed by ... (more)

Reference:

Grossman, S. & Porth, C.M. Porth's Pathophysiology: Concepts of Altered Health States, 9th ed., Philadelphia: Lippincott Williams & Wilkins, 2014, chapter 45, Disorders of Gastrointestinal Function, p. 1171.

Question 6:

While studying the esophagus, a student is able to correctly state which of the following to a faculty member?

(see full question)

You selected:

"The smooth muscle layers provide the peristaltic movements needed to move food along the length of the esophagus."

Correct Explanation:

The smooth muscle layers provide the peristaltic movements needed to move food along the length of the esophagus. The pharyngoesophageal sphincter, the upper sphincter, keeps air f ... (more)

Reference:

Grossman S, Porth CM (2014). Porth’s Pathophysiology: Concepts of Altered Health States. 9th ed. Philadelphia: Lippincott Williams & Wilkins, Chapter 44, Structure and Function of the Gastrointestinal System, p. 1152.

Question 7:

A nurse is concerned about a patient's continual use of nonsteroidal antiinflammatory drugs (NSAIDs). Which of the following is the nurse concerned

(see full question)


about? You selected:

Damage to the gastric mucosa

Correct Explanation:

One of the important characteristics of the gastric mucosa is resistance to highly acidic secretions that it produces. However, when aspirin, NSAIDs, Heliocbacter pylori, ethyl al ... (more)

Reference:

Grossman S, Porth CM (2014). Porth’s Pathophysiology: Concepts of Altered Health States. 9th ed. Philadelphia: Lippincott Williams & Wilkins, Chapter 44, Structure and Function of the Gastrointestinal System, p. 1163.

Question 8:

Which of the following substances is broken down by the emulsification process?

(see full question)

You selected:

Triglycerides

Correct Explanation:

The first step in digestion of lipids (triglycerides) is to break down the large globules of dietary fat into smaller sizes, a process called emulsification. Proteins and carbohyd ... (more)

Reference:

Grossman S, Porth CM (2014). Porth’s Pathophysiology: Concepts of Altered Health States. 9th ed. Philadelphia: Lippincott Williams & Wilkins, Chapter 44, Structure and Function of the Gastrointestinal System, p. 1167.

Question 9: (see full question)

The patient chews an apple and starts the three phases of swallowing. As food enters the patient's esophagus, stretching and nerve impulses initiate that are controlled by the swallowing center.

You selected:

vagus; peristaltic waves

Correct Explanation:

The esophageal phase is innervated by the vagus nerve, which initiates the peristaltic waves that will carry the food bolus into the stomach. The phrenic nerve is not involved with ... (more)


Reference:

Grossman S, Porth CM (2014). Porth’s Pathophysiology: Concepts of Altered Health States. 9th ed. Philadelphia: Lippincott Williams & Wilkins, Chapter 44, Structure and Function of the Gastrointestinal System, p. 1158.

Question 10:

While assessing a client who is experiencing diarrhea caused by Clostridium difficile, the nurse should anticipate hearing:

(see full question)

You selected:

Hyperactive bowel sounds

Correct Explanation:

Inflammatory changes increase motility, which would lead to hyperactive bowel sounds. In many instances, it is not certain whether changes in motility occur because of inflammation ... (more)

(see full question)

A student is studying the interstitial cells of Cajal found in the smooth muscle tissue of the gastrointestinaltract. The student says that these cells produce slow waves that are increased in amplitude by which of the following?

You selected:

Parasympathetic nervous system

Correct Explanation:

The amplitude and, to a lesser extent, the frequency of the slow waves, can be modulated by the parasympathetic and sympathetic divisions of the autonomic nervous system. The paras ... (more)

Reference:

Grossman S, Porth CM (2014). Porth’s Pathophysiology: Concepts of Altered Health States. 9th ed. Philadelphia: Lippincott Williams & Wilkins, Chapter 44, Structure and Function of the Gastrointestinal System, p. 1155.

Question 2:

A client has been told he has abdominal adhesions due to an old appendix infection. Knowing the structures of the abdominal cavity, the nurse will explain which of the following structures can form bands of fibrous scar tissue to wall off the infection, preventing it from spreading to other parts of the body?

(see full question)


You selected:

Omentum

Correct Explanation:

The greater omentum helps to prevent infection from entering the peritoneal cavity and protects the intestines from cold. It often forms adhesions (i.e., bands of fibrous scar tiss ... (more)

Reference:

Grossman, S. & Porth, C.M. Porth's Pathophysiology: Concepts of Altered Health States, 9th ed., Philadelphia: Lippincott Williams & Wilkins, 2014, Chapter 44, Structure and Function of the Gastrointestinal System, p. 1154.

Question 3: (see full question)

A nurse explains to her patient that food is moved along the gastrointestinal (GI) tract with intermittent contractions that mix the food and move it along. These movements are found in which of the following organs?

You selected:

Small intestine

Correct Explanation:

The movements of the GI tract can be either rhythmic or tonic. Rhythmic movements consist of intermittent contractions that are responsible for mixing and moving food along the dig ... (more)

Reference:

Grossman S, Porth CM (2014). Porth’s Pathophysiology: Concepts of Altered Health States. 9th ed. Philadelphia: Lippincott Williams & Wilkins, Chapter 44, Structure and Function of the Gastrointestinal System, p. 1155.

Question 4:

The common bile duct opens into which of the following parts of the gastrointestinal tract?

(see full question)

You selected:

Duodenum

Correct Explanation:

The duodenum contains the opening for the common bile duct and the main pancreatic duct.

Reference:

Grossman S, Porth CM (2014). Porth’s Pathophysiology: Concepts of Altered Health States. 9th ed. Philadelphia: Lippincott Williams & Wilkins, Chapter 44, Structure and Function of the Gastrointestinal System, p. 1152.


Question 5: (see full question)

You selected:

The gastrointestinal laboratory nurse is learning about small intestine secretions. Which of the following explanations is most accurate? "An extensive array of mucus-producing glands, called Brunner glands, is concentrated where contents from the stomach and secretions from the liver and pancreas enter the duodenum."

Correct Explanation:

An extensive array of mucus producing glands, called Brunner glands, is concentrated at the site where the contents of the stomach and secretions from the liver and pancreas enter ... (more)

Reference:

Grossman S, Porth CM (2014). Porth’s Pathophysiology: Concepts of Altered Health States. 9th ed. Philadelphia: Lippincott Williams & Wilkins, Chapter 44, Structure and Function of the Gastrointestinal System, p. 1164.

Question 6: (see full question)

A patient with pancreatitis has decreased production of pancreatic enzymes such as trypsin. The dietitian knows that the patient may have difficulty digesting which of the following foods?

You selected:

Fried chicken and deviled eggs

Correct Explanation:

Proteins are broken down by pancreatic enzymes, such as trypsin, chymotrypsin, carboxypeptidase, and elastase. Of the meal options, fried chicken and deviled eggs have the highest ... (more)

Reference:

Grossman S, Porth CM (2014). Porth’s Pathophysiology: Concepts of Altered Health States. 9th ed. Philadelphia: Lippincott Williams & Wilkins, Chapter 44, Structure and Function of the Gastrointestinal System, p. 1168.

Question 7:

A student identifies enterocytes on a microscope slide of tissue. The student explains that these cells secrete which of the following?

(see full question)

You selected:

Brush border enzymes


Correct Explanation:

Each villus of the small intestine is covered with cells called enterocytes. The enterocytes secrete enzymes that aid in the digestion of carbohydrates and proteins. The enzymes ... (more)

Reference:

Grossman S, Porth CM (2014). Porth’s Pathophysiology: Concepts of Altered Health States. 9th ed. Philadelphia: Lippincott Williams & Wilkins, Chapter 44, Structure and Function of the Gastrointestinal System, p. 1166.

Question 8: (see full question)

A patient has experienced a stroke affecting the reticular formation of the medulla and lower pons. The nurse tells the patient's wife that care must be taken with eating to prevent which of the following?

You selected:

Aspiration pneumonia

Correct Explanation:

Diseases (strokes) that disrupt the brain centers of the reticular formation of the medulla and lower pons disrupt the coordination of swallowing and predispose a person to food an ... (more)

Reference:

Grossman S, Porth CM (2014). Porth’s Pathophysiology: Concepts of Altered Health States. 9th ed. Philadelphia: Lippincott Williams & Wilkins, Chapter 44, Structure and Function of the Gastrointestinal System, p. 1156.

Question 9:

Which substance necessary for vitamin B12 absorption is produced by the parietal cells in the stomach?

(see full question)

You selected:

Intrinsic factor

Correct Explanation:

Intrinsic factor is produced by the parietal cells in the stomach and is necessary for the absorption of vitamin B12.

Reference:

Grossman S, Porth CM (2014). Porth’s Pathophysiology: Concepts of Altered Health States. 9th ed. Philadelphia: Lippincott Williams & Wilkins, Chapter 44, Structure and Function of the Gastrointestinal System, p. 1166.


Question 10:

Serous fluid is contained in which of the following?

(see full question)

You selected:

Peritoneal cavity

Correct Explanation:

Between the viseral and parietal peritoneum, is the peritoneal cavity. The peritoneal cavity contains fluid secreted by the serous membranes. This serous fluid forms a moist and s ... (more)

Question 1: (see full question)

An anatomy student is studying the stomach. The student explains that the funnel-shaped portion that connects with the intestine is which of the following?

You selected:

Fundus

Incorrect Correct response:

Pyloric region

Explanation:

The funnel-shaped portion that connects with the small intestine is called the pyloric region. The small part of the stomach that surrounds the cardiac oriface is called the cardia ... (more)

Reference:

Grossman S, Porth CM (2014). Porth’s Pathophysiology: Concepts of Altered Health States. 9th ed. Philadelphia: Lippincott Williams & Wilkins, Chapter 44, Structure and Function of the Gastrointestinal System, p. 1152.

Question 2:

Place the following components to the gastrointestinal tract in descending order, describing the route food would take through the body.

(see full question)

You selected:

Mouth Esophagus Stomach Pyloric region Duodenum Rectum


Correct Explanation:

Food passes through the mouth and is chewed and enters the esophagus. After the esophagus it passes into the stomach to the pyloric region and into the dudoenum, then out through t ... (more)

Reference:

Grossman S, Porth CM (2014). Porth’s Pathophysiology: Concepts of Altered Health States. 9th ed. Philadelphia: Lippincott Williams & Wilkins, Chapter 44, Structure and Function of the Gastrointestinal System, p. 1152.

Question 3: (see full question)

Where in the GI tract is food digested and absorbed?

You selected:

The stomach and the jejunum

Incorrect Correct response:

The jejunum and the ileum

Explanation:

It is in the jejunum and ileum that food is digested and absorbed. The other answers are incorrect.

Reference:

Grossman S, Porth CM (2014). Porth's Pathophysiology: Concepts of Altered Health States. 9th ed. Philadelphia: Lippincott Williams & Wilkins, Chapter 44: Structure and Function of the Gastrointestinal System, p. 1152.

Question 4:

Which of the following would help a nurse best explain an "incretin effect"?

(see full question)

You selected:

"It is an increase in insulin release after ingestion of food."

Correct Explanation:

An incretin effect means that there is an increase in insulin release after an oral glucose load or food ingestion. The other answers are incorrect.

Reference:

Grossman S, Porth CM (2014). Porth’s Pathophysiology: Concepts of Altered Health States. 9th ed. Philadelphia: Lippincott Williams & Wilkins, Chapter 43, Disorders of Gastrointestinal Function, p. 1141.


(see full question)

When trying to explain to a newly diagnosed irritable bowel syndrome (IBS) client how the nervous system affects gastrointestinal (GI) disorders, the nurse mentions which of the following in regard to IBS?

You selected:

Decrease in gastrointestinal motility

Question 5:

Incorrect Correct response:

Increase in enteric nervous system activity

Explanation:

Stimulation of the parasympathetic nervous system causes a general increase in activity of the entire enteric nervous system. Sympathetic stimulation inhibits activity, causing man ... (more)

Reference:

Grossman S, Porth CM (2014). Porth’s Pathophysiology: Concepts of Altered Health States. 9th ed. Philadelphia: Lippincott Williams & Wilkins, Chapter 44, Structure and Function of the Gastrointestinal System, p. 1161.

Question 6: (see full question)

A patient has been diagnosed with an incompetent pyloric sphincter. This patient will experience abnormal communication between which of the following components of the gastrointestinal tract?

You selected:

Stomach and duodenum

Correct Explanation:

The pylorus is located between the body of the stomach and the duodenum.

Reference:

Grossman S, Porth CM (2014). Porth’s Pathophysiology: Concepts of Altered Health States. 9th ed. Philadelphia: Lippincott Williams & Wilkins, Chapter 44, Structure and Function of the Gastrointestinal System, p. 1158.

Question 7:

Gastrin production, a task that is performed by the stomach, results in which of the following effects?

(see full question)

You selected:

Stimulation of HCl secretions by parietal cells

Correct Explanation:

The primary function of gastrin is the stimulation of gastric acid secretion. It does not have a direct effect on exocrine or endocrine pancreatic function, and


it does not partici ... (more)

Reference:

Grossman S, Porth CM (2014). Porth’s Pathophysiology: Concepts of Altered Health States. 9th ed. Philadelphia: Lippincott Williams & Wilkins, Chapter 44, Structure and Function of the Gastrointestinal System, p. 1163.

Question 8:

Which of the following structures in the gastrointestinal (GI) tract has absorptive function focusing mainly on water resorption?

(see full question)

You selected:

Large instestine

Correct Explanation:

Absorption is the process of moving nutrients and other materials from the external environment in the lumen of the GI tract into the internal environment. The absorptive function ... (more)

Reference:

Grossman S, Porth CM (2014). Porth’s Pathophysiology: Concepts of Altered Health States. 9th ed. Philadelphia: Lippincott Williams & Wilkins, Chapter 44, Structure and Function of the Gastrointestinal System, p. 1166.

Question 9:

Regurgitation of bile salts and duodenal contents can lead to gastric ulcers. Which of the following structures prevents this from happening?

(see full question)

You selected:

Pyloric sphincter

Correct Explanation:

The phyloric sphincter prevents the backflow of gastric contents and allows them to flow into the duodenum at a rate commensurate with the ability of the duodenum to accept them. T ... (more)

Reference:

Grossman S, Porth CM (2014). Porth’s Pathophysiology: Concepts of Altered Health States. 9th ed. Philadelphia: Lippincott Williams & Wilkins, Chapter 44, Structure and Function of the Gastrointestinal System, p. 1158.


Question 10: (see full question)

You selected:

A student is studying gallbladder function. Which of the following gastrointestinal hormones stimulates contraction of the gallbladder? Cholecystokinin

Correct Explanation:

Cholecystokinin, which is secreted in the duodenum and jejunum, stimulates contraction of the gallbladder and secretion of pancreatic enzymes.

Question 1:

Which of the following has a surface area approximately equal to that of the skin?

(see full question)

You selected:

Peritoneum

Correct Explanation:

The peritoneum is the largest serous membrane in the body with a surface area approximately equal to that of the skin. The submucosal (second) layer consists of dense connective ti ... (more)

Reference:

Grossman S, Porth CM (2014). Porth’s Pathophysiology: Concepts of Altered Health States. 9th ed. Philadelphia: Lippincott Williams & Wilkins, Chapter 44, Structure and Function of the Gastrointestinal System, p. 1153.

Question 2:

A patient is experiencing reflux of stomach contents into the esophagus. The nurse determines that the problem may result from:

(see full question)

You selected:

Gastroesophageal sphincter

Correct Explanation:

The gastroesophageal sphincter (also called the lower esophageal sphincter) prevents the contents of the stomach from rising into the esophagus. The upper esophageal sphincter (als ... (more)

Reference:

Grossman S, Porth CM (2014). Porth’s Pathophysiology: Concepts of Altered Health States. 9th ed. Philadelphia: Lippincott Williams & Wilkins, Chapter 44, Structure and Function of the Gastrointestinal System, p. 1152.


(see full question)

A pharmacology student is studying gastrointestinal medications. The student explains that proton pump inhibitors (PPIs) are used to inhibit secretion of which of the following?

You selected:

Gastric acid

Question 3:

Correct Explanation:

PPIs (also known as the H+/K+-ATPase transporter), which are used in the treatment of acid reflux and peptic ulcer, inhibit gastric acid secretion. The other options are not inhibit ... (more)

Reference:

Grossman S, Porth CM (2014). Porth’s Pathophysiology: Concepts of Altered Health States. 9th ed. Philadelphia: Lippincott Williams & Wilkins, Chapter 44, Structure and Function of the Gastrointestinal System, p. 1163.

Question 4:

Which of the following would the nurse say is the function of secretory glands in the gastrointestinal tract? Select all that apply.

(see full question)

You selected:

• To produce mucus to lubricate the wall of the gastrointestinal tract • To secrete fluid and enzymes to aid in digestion

Correct Explanation:

Secretory glands produce mucus to lubricate the gastrointestinal tract and secrete fluid and enzymes to aid in digestion. The other two functions are not accurate.

Reference:

Grossman S, Porth CM (2014). Porth’s Pathophysiology: Concepts of Altered Health States. 9th ed. Philadelphia: Lippincott Williams & Wilkins, Chapter 43, Disorders of Gastrointestinal Function, p. 1162.

Question 5:

The swallowing reflex is an ordered sequence of events. In which of the following phases is the epiglottis moved so that it covers the larynx?

(see full question)

You selected: Correct

Pharyngeal phase


Explanation:

In the pharyngeal phase, which is involuntary, the vocal cords are pulled together and the epiglottis is moved so that it covers the larynx.

Reference:

Grossman S, Porth CM (2014). Porth’s Pathophysiology: Concepts of Altered Health States. 9th ed. Philadelphia: Lippincott Williams & Wilkins, Chapter 44, Structure and Function of the Gastrointestinal System, pp. 1156, 1158.

Question 6:

The production of chyme occurs in which of the following organs?

(see full question)

You selected:

Stomach

Correct Explanation:

The chemical breakdown of protein begins in the stomach where food is converted to a creamy mixture called chyme.

Reference:

Grossman S, Porth CM (2014). Porth’s Pathophysiology: Concepts of Altered Health States. 9th ed. Philadelphia: Lippincott Williams & Wilkins, Chapter 44, Structure and Function of the Gastrointestinal System, p. 1158.

Question 7: (see full question)

The human body secretes approximately 2000ml of intestinal secretions and 1200ml of salivary secretions. What is the total amount of daily gastronintestal tract secretions (round up or down to the nearest 1000ml)?

You selected:

3200

Incorrect Correct response:

7000

Explanation:

The total amount of gastrointestinal tract secrections is: salivary (1200), gastric (2000), pancreatic (1200), biliary (700) and intestinal (200) for a total of 7100. When rounded ... (more)

Reference:

Grossman S, Porth CM (2014). Porth’s Pathophysiology: Concepts of Altered Health States. 9th ed. Philadelphia: Lippincott Williams & Wilkins, Chapter 44, Structure and Function of the Gastrointestinal System, p. 1162.


Question 8: (see full question)

You selected:

Which of the following clients likely faces the greatest risk of a gastrointestinal bleed? A client who takes aspirin with each meal to control symptoms of osteoarthritis

Correct Explanation:

The gastric mucosa can be easily damaged by drugs such as aspirin, resulting in local ischemia, vascular stasis, hypoxia, and tissue necrosis. Antihypertensives, diuretics, antibio ... (more)

Reference:

Grossman S, Porth CM (2014). Porth’s Pathophysiology: Concepts of Altered Health States. 9th ed. Philadelphia: Lippincott Williams & Wilkins, Chapter 44, Structure and Function of the Gastrointestinal System, p. 1164.

Question 9:

Which of the following aspects of gastrointestinal function is performed by the brush border enzymes of the villus structures?

(see full question)

You selected:

Digestion of carbohydrates

Correct Explanation:

Brush border enzymes aid in the digestion of carbohydrates and proteins. They do not metabolize fat or synthesize vitamin A, and they do not directly contribute to fluid and electr ... (more)

Reference:

Grossman S, Porth CM (2014). Porth’s Pathophysiology: Concepts of Altered Health States. 9th ed. Philadelphia: Lippincott Williams & Wilkins, Chapter 44, Structure and Function of the Gastrointestinal System, p. 1169.

Question 10:

Which of the following are accessory organs that also aid in digestion? Select all that apply.

(see full question)

You selected:

Incorrect

• Salivary glands • Thyroid gland


Correct response:

• Salivary glands • Liver • Pancreas

Explanation:

The accessory organs, which include the salivary glands, liver, and pancreas, produce secretions that aid in digestion. The thyroid gland and kidneys do not aid in this process.

Question 1:

The physician suspects a client has developed diverticular disease. The most appropriate test to confirm the diagnosis would be:

(see full question)

You selected:

CT scan

Correct Explanation:

Diverticular disease may be determined by CT scan or ultrasound. CT scans are the safest and most cost-effective. Due to risk of peritonitis, barium enema studies should be avoided ... (more)

Reference:

Grossman S, Porth CM (2014). Porth’s Pathophysiology: Concepts of Altered Health States. 9th ed. Philadelphia: Lippincott Williams & Wilkins, Chapter 45, Disorders of Gastrointestinal Function, p. 1192.

Question 2: (see full question)

One of the accepted methods of screening for colorectal cancer is testing for occult blood in the stool. Because it is possible to get a false-positive result on these tests, the nurse would instruct the client to do what?

You selected:

Avoid nonsteroidal anti-inflammatory drugs for 1 week prior to testing.

Correct Explanation:

To reduce the likelihood of false-positive tests, persons are instructed to avoid nonsteroidal anti-inflammatory drugs such as ibuprofen and aspirin for 7 days prior to testi ... (more)

Reference:

Grossman S, Porth CM (2014). Porth's Pathophysiology: Concepts of Altered Health States. 9th ed. Philadelphia: Lippincott Williams & Wilkins, Chapter 45: Disorders of Gastrointestinal Function, p. 1203.


(see full question)

Peptic ulcers can affect one or all layers of the stomach or duodenum. Ulcers that penetrate through the muscularis layer are classified as which of the following types of ulcer?

You selected:

Perforation

Question 3:

Correct

(see full question)

The nurse is teaching her patient with hepatobiliary disease about her diet. She tells her that she may have steatorrhea, which is the malabsorption of which dietary component?

You selected:

Fat

Question 4:

Correct Explanation:

The condidtions that impair one or more steps involved in digestion and absorption of nutrients can be divided into three broad categories: intraluminal maldigestion, disorders of ... (more)

Reference:

Grossman S, Porth CM (2014). Porth’s Pathophysiology: Concepts of Altered Health States. 9th ed. Philadelphia: Lippincott Williams & Wilkins, Chapter 45, Chapter Title, pp. 1199- 1200.

Question 5:

A patient is admitted with a peptic ulcer. The nurse’s priority on admission is to assess for which of the following? Select all that apply.

(see full question)

You selected:

• Pain assessment • Alcohol usage • Smoking history • dietary history

Incorrect Correct response:

• Smoking history • Family history • Alcohol usage

Explanation:

Epidemiologic studies have identified independent factors that augment the effect of H.pylori infection and NSAID-produced peptic ulcer disease. The factors include advancing age, ... (more)


Reference:

Grossman, S, Porth, CM (2014). Porth’s Pathophysiology Concepts of Altered Health States. 9thed. Philadelphia. Lippincott Williams & Wilkins, Chapter 45, Disorders of Gastrointestinal Function, p. 1181.

Question 6:

An elderly patient presents with loose mucousy stools. The nurse suspects the patient has Clostridium difficile. What is a priority assessment for the nurse?

(see full question)

You selected:

Ask the patient about his or her antibiotic use.

Correct Explanation:

The diagnosis of C. difficile-associated diarrhea requires a careful history, with particular emphasis on antibiotic use.

Question 7:

A nurse caring for a patient who is vomiting assesses for accompanying symptoms including which of the following? Select all that apply.

(see full question)

You selected:

• Lightheadedness • Dizziness

Incorrect Correct response:

• Dizziness • Decreased blood pressure

Explanation:

Vomiting may be accompanied by dizziness, lightheadedness, a decrease in blood pressure, and bradycardia.

Reference:

Grossman S, Porth CM (2014). Porth’s Pathophysiology: Concepts of Altered Health States. 9th ed. Philadelphia: Lippincott Williams & Wilkins, Chapter 45, Disorders of Gastrointestinal Function, p. 1171

Question 8: (see full question)

Celiac disease commonly presents in infancy as failure to thrive. It is an inappropriate T-cell–mediated immune response, and there is no cure for it. What is the treatment of choice for celiac disease?


You selected:

Removal of gluten from the diet

Correct Explanation:

The primary treatment of celiac disease consists of removal of gluten and related proteins from the diet. No other answer is correct.

Reference:

Grossman S, Porth CM (2014). Porth's Pathophysiology: Concepts of Altered Health States. 9th ed. Philadelphia: Lippincott Williams & Wilkins, Chapter 45: Disorders of Gastrointestinal Function, p. 1199.

Question 9: (see full question)

Pharmacologic treatment for peptic ulcers has changed over the past several decades. The nurse knows that the goal for pharmacologic treatment is focused on which of the following?

You selected:

Eradicating Helicobacter pylori (H. pylori)

Correct Explanation:

Treatment of peptic ulcer is aimed at eradicating the cause and promoting a permanent cure for the disease. Pharmacologic treatment focuses on eradicating H. pylori, relieving ulce ... (more)

Question 10: (see full question)

Peritonitis is an inflammatory condition of the lining of the abdominal cavity. What is one of the most important signs of peritonitis?

You selected:

Vomiting of coffee ground–appearing emesis

Incorrect Correct response:

The translocation of extracellular fluid into the peritoneal cavity

Explanation:

One of the most important manifestations of peritonitis is the translocation of extracellular fluid into the peritoneal cavity (through weeping or serous fluid from the infla ... (more)

Question 1: (see full question)

The nurse is reviewing the medical history of four clients. The nurse determines that the client at greatest risk for developing peptic ulcer disease is:

You selected:

A client with a prior diagnosis of Helicobacter pylori and refused treatment


Correct Explanation:

The two most important risk factors for peptic ulcer disease are infection with the bacteria H. pylori and use of aspirin and/or NSAIDs. Tylenol does not place the client at risk fo ... (more)

Reference:

Grossman S, Porth CM (2014). Porth’s Pathophysiology: Concepts of Altered Health States. 9th ed. Philadelphia: Lippincott Williams & Wilkins, Chapter 45, Disorders of Gastrointestinal Function, p. 1180.

Question 2:

A nurse is completing an abdominal assessment on a patient suspected to have appendicitis. When the nurse applies and then releases pressure in the patient's right lower quadrant, the patient experiences tenderness. The nurse is documenting the presence of which of the following?

(see full question)

You selected:

Rebound tenderness

Correct Explanation:

The nurse documents the presence of rebound tenderness, defined as tenderness that occurs when the nurse applies and then releases pressure to an area.

Reference:

Grossman S, Porth CM (2014). Porth’s Pathophysiology: Concepts of Altered Health States. 9th ed. Philadelphia: Lippincott Williams & Wilkins, Chapter 45, Disorders of Gastrointestinal Function, p. 1193.

Question 3:

A client with malabsorption syndrome is experiencing fatty, bulky, yellow-gray, and foul-smelling stools. The nurse interprets this as:

(see full question)

You selected:

Steatorrhea

Correct Explanation:

A hallmark of malabsorption is steatorrhea, characterized by fatty, bulky, yellow-gray, and foul-smelling stools. The other options may occur as a result of the fat loss.

Reference:

Grossman, S. & Porth, C.M. Porth's Pathophysiology: Concepts of Altered


Health States, 9th ed., Philadelphia: Lippincott Williams & Wilkins, 2014, Chapter 45, Disorders of Gastrointestinal Function, p. 1199.

Question 4: (see full question)

You selected:

Which of the following individuals most likely faces the greatest risk of developing Clostridium difficile colitis? A 79-year-old hospital client who is being treated with broad-spectrum antibiotics

Correct Explanation:

C. difficile colitis is associated with antibiotic therapy; C. difficile is noninvasive, and development of C. difficile colitis requires disruption of normal intestinal flora. Pep ... (more)

Reference:

Grossman S, Porth CM (2014). Porth’s Pathophysiology: Concepts of Altered Health States. 9th ed. Philadelphia: Lippincott Williams & Wilkins, Chapter 45, Disorders of Gastrointestinal Function, p. 1190.

Question 5:

The cause of gastric carcinomas has been influenced by which of the following factors? Select all that apply.

(see full question)

You selected:

• Strain of Helicobacter pylori (H. pylori) • Environmental factors • Family history

Correct Explanation:

Chronic infection with H. pylori appears to serve as a cofactor in some types of gastric carcinomas. In addition to genetics, the likelihood of developing gastric cancer from an H...... (more)

Reference:

Grossman, S, Porth, CM (2014). Porth’s Pathophysiology Concepts of Altered Health States. 9thed. Philadelphia. Lippincott Williams & Wilkins, Chapter 45, Disorders of Gastrointestinal Function, p. 1183.

Question 6:

A teenager who has a history of achalasia will likely complain of which of the following clinical manifestations?

(see full question)


You selected:

Feeling like there is food stuck in the back of the throat

Correct Explanation:

Achalasia produces functional obstruction of the esophagus so that food has difficulty passing into the stomach, and the esophagus above the lower esophageal sphincter becomes dist ... (more)

Reference:

Grossman, S. & Porth, C.M. Porth's Pathophysiology: Concepts of Altered Health States, 9th ed., Philadelphia: Lippincott Williams & Wilkins, 2014, Chapter 45, Disorders of Gastrointestinal Function, p. 1173.

Question 7:

A client has developed a ruptured appendix. The nurse is aware the client is at high risk for:

(see full question)

You selected:

Peritonitis

Correct Explanation:

Complications of a ruptured appendix include peritonitis, localized peril abscess formation, and septicemia. Vomiting may occur with peritonitis; however, the physiologic effect of ... (more)

Reference:

Grossman S, Porth CM (2014). Porth’s Pathophysiology: Concepts of Altered Health States. 9th ed. Philadelphia: Lippincott Williams & Wilkins, Chapter 45, Disorders of Gastrointestinal Function, p. 1198.

Question 8: (see full question)

A nurse administering a patient's medication tells the patient that a proton pump inhibitor has been added. When the patient asks the purpose of the medication, the nurse responds that it is to prevent which of the following?

You selected:

Stress ulcer

Correct Explanation:

Proton pump inhibitors are the first line of medications used in the prevention of stress ulcers.

Reference:

Porth CM (2011). Essentials of Pathophysiology: Concepts of Altered Health States. 3rd ed. Philadelphia: Lippincott Williams & Wilkins, Chapter 29, Disorders of Gastrointestinal Function, p. 709.


(see full question)

Crohn's disease not only affects adults but also can occur in children. The nurse assesses for which of the following major manifestations in children with Crohn's disease?

You selected:

Malnutrition

Question 9:

Correct Explanation:

When Crohn's disease occurs in children, one of the major manifestations may be retardation of growth and significant malnutrition.

Reference:

Grossman, S, Porth, CM (2014). Porth’s Pathophysiology Concepts of Altered Health States. 9thed. Philadelphia. Lippincott Williams & Wilkins, Chapter 45, Disorders of Gastrointestinal Function, p. 1186.

Question 10:

In the balance of secretions in the gastric mucosa by the parietal cells, which of the following ions is produced to buffer the production of hydrochloric acid?

(see full question)

You selected:

HCO3-

Correct Explanation:

Normally the secretion of hydrochloric acid by the parietal cells, of the stomach is accompanied by secretion of bicarbonate ions (HCO3-), which protects the mucosa from injury, as ... (more)

Question 1:

Malabsorption syndrome results in loss of fat in the stool and failure to absorb fat-soluble vitamins.

(see full question)

You selected:

True

Correct Explanation:

In malabsorption syndrome, there is loss of fat in the stools and failure to absorb the fat-soluable vitamins.

Reference:

Grossman S, Porth CM (2014). Porth’s Pathophysiology: Concepts of Altered


Health States. 9th ed. Philadelphia: Lippincott Williams & Wilkins, Chapter 45, Disorders of Gastrointestinal Function, p. 1199.

Question 2: (see full question)

You selected:

Symptoms of gastric cancer include vague epigastric pain, which makes early detection difficult. The nurse would expect a patient to undergo which of the following diagnostic examinations to determine the location of the gastric cancer? Barium x-ray

Correct Explanation:

Diagnosis of gastric cancer is accomplished by a variety of techniques, including barium x-ray studies, endoscopic studies with biopsy, and cytologic studies of gastic secretions. .... (more)

Reference:

Grossman, S, Porth, CM (2014). Porth’s Pathophysiology Concepts of Altered Health States. 9thed. Philadelphia. Lippincott Williams & Wilkins, Chapter 45, Disorders of Gastrointestinal Function, p. 1183.

Question 3:

Which symptom is often observed in cases of peritonitis?

(see full question)

You selected:

Abdominal rigidity

Correct Explanation:

The abdomen is rigid and becomes boardlike because of reflex muscle guarding. The patient typically becomes tachycardiac, has increased WBC count, and breathes in a shallow way to ... (more)

Reference:

Grossman S, Porth CM (2014). Porth’s Pathophysiology: Concepts of Altered Health States. 9th ed. Philadelphia: Lippincott Williams & Wilkins, Chapter 45, Disorders of Gastrointestinal Function, p. 1198.

Question 4:

A client tells the doctor that every time he eats something made with wheat, barley, and/or rye, he gets sick. The doctors suspects the client has celiac sprue. The physician should further assess the client for: Select all that apply.

(see full question)


You selected:

• Diarrhea • Abdominal pain • Constipation

Incorrect Correct response:

• Diarrhea • Abdominal pain • Iron deficiency

Explanation:

The classic presentation in adults is diarrhea, which may be accompanied by abdominal pain or discomfort. It has been observed that fewer persons with celiac disease are presenting ... (more)

Reference:

Grossman, S. & Porth, C.M. Porth's Pathophysiology: Concepts of Altered Health States, 9th ed., Philadelphia: Lippincott Williams & Wilkins, 2014, Chapter 45, Disorders of Gastrointestinal Function, p. 1199.

Question 5:

Crohn's disease is treated by several measures. Treatment with corticosteroids focuses on which of the following?

(see full question)

You selected:

Immune suppression

Incorrect Correct response:

Inflammatory suppression

Explanation:

Treatment methods focus on terminating the inflammatory response and promoting healing, maintaining adequate nutrition, and preventing and treating complications. Several medicatio ... (more)

Reference:

Grossman, S, Porth, CM (2014). Porth’s Pathophysiology Concepts of Altered Health States. 9thed. Philadelphia. Lippincott Williams & Wilkins, Chapter 45, Disorders of Gastrointestinal Function, p. 1183.

Question 6: (see full question)

Upon admission, a patient tells the nurse that he takes aspirin every 4 hours every day. The nurse determines that this patient is at risk for which of the following?

You selected:

Peptic ulcer


Correct Explanation:

Peptic ulcers occur in the areas of the upper gastrointestinal tract and are caused by Helicobacter pylori infection and aspirin or nonsteroidal antiinflammatory drug use. Therefo ... (more)

Reference:

Grossman S, Porth CM (2014). Porth’s Pathophysiology: Concepts of Altered Health States. 9th ed. Philadelphia: Lippincott Williams & Wilkins, Chapter 45, Disorders of Gastrointestinal Function, p. 1183.

Question 7:

A 66-year-old woman has been diagnosed with diverticular disease based on her recent complaints and the results of a computed tomography (CT) scan. Which of the client's following statements demonstrates an accurate understanding of this diagnosis?

(see full question)

You selected:

“I've always struggled with heartburn and indigestion, and I guess I shouldn't have ignored those warning signs.”

Incorrect Correct response:

“I suppose I should try to eat a bit more fiber in my diet.”

Explanation:

Increased fiber is important in both the prevention and treatment of diverticular disease. Overuse of laxatives is not linked to diverticular disease, and heartburn and indigestion ... (more)

Reference:

Grossman S, Porth CM (2014). Porth’s Pathophysiology: Concepts of Altered Health States. 9th ed. Philadelphia: Lippincott Williams & Wilkins, Chapter 45, Disorders of Gastrointestinal Function, p. 1192.

Question 8: (see full question)

The nurse has been providing dietary teaching to a client diagnosed with irritable bowel syndrome. The nurse determines that the teaching was effective when the client selects:

You selected:

A bran muffin, fruit, and orange juice.

Correct Explanation:

Clients with irritable bowel syndrome should have adequate intake of fiber. All other options contain foods that should be avoided. Substances that should be avoided include fatty ... (more)


Reference:

Grossman S, Porth CM (2014). Porth’s Pathophysiology: Concepts of Altered Health States. 9th ed. Philadelphia: Lippincott Williams & Wilkins, Chapter 45, Disorders of Gastrointestinal Function, p. 1184.

Question 9:

A nurse providing dietary guidance to a patient with celiac disease tells the patient that which of the following foods should be avoided?

(see full question)

You selected:

Wheat bread

Correct Explanation:

Patients with celiac disease should avoid ingesting gluten, which is the primary protein in wheat, barley, and rye. The nurse should tell the patient to avoid wheat bread. Watermel ... (more)

Reference:

Grossman S, Porth CM (2014). Porth’s Pathophysiology: Concepts of Altered Health States. 9th ed. Philadelphia: Lippincott Williams & Wilkins, Chapter 45, Disorders of Gastrointestinal Function, p. 1200.

Question 10: (see full question)

Crohn's disease is recognized by sharply demarcated, granulomatous lesions that are surrounded by normal-appearing mucosal tissue. The nurse recognizes these lesions to be defined by which of the following descriptions?

You selected:

Mosaic

Incorrect Correct response:

Cobblestone

Explanation:

A characteristic feature of Crohn's disease is the sharply demarcated, granulomatous lesions that are surrounded by normal-appearing mucosal tissue. When there are multiple lesions ... (more)

Question 1:

Select the client at greatest risk for developing colorectal cancer.

(see full question)

You selected: Correct

A 64-year-old female whose mother had colorectal cancer


Explanation:

Colorectal cancer peaks at 60 to 70 years of age, and fewer than 20% of cases occur before age 50. Its incidence is increased among persons with a family history of cancer, persons ... (more)

Reference:

Grossman S, Porth CM (2014). Porth’s Pathophysiology: Concepts of Altered Health States. 9th ed. Philadelphia: Lippincott Williams & Wilkins, Chapter 45, Disorders of Gastrointestinal Function, p. 1202.

Question 2: (see full question)

A 60-year-old male client has presented to his primary care provider to follow up with his ongoing treatment for peptic ulcer disease. What is the most likely goal of this client's pharmacologic treatment?

You selected:

Inhibiting gastric acid production

Correct Explanation:

Current therapies for peptic ulcer disease are aimed at neutralization of gastric acid, inhibition of gastric acid (H2 antagonists and proton pump inhibitors), and promotion of muc ... (more)

Reference:

Grossman S, Porth CM (2014). Porth’s Pathophysiology: Concepts of Altered Health States. 9th ed. Philadelphia: Lippincott Williams & Wilkins, Chapter 45, Disorders of Gastrointestinal Function, p. 1181.

Question 3:

The incidence of stomach cancer has significantly decreased in the United States, yet it remains the leading cause of death worldwide. The nurse understands the reason for the high mortality rate in stomach cancer is because of which of the following reasons?

(see full question)

You selected:

Diagnostic testing is not available for detection of the disease.

Incorrect Correct response:

Patients have few early symptoms of the disease.

Explanation:

Although the incidence of cancer of the stomach has declined over the past 50 years in the United States, it remains the leading cause of death worldwide. Because there are few ear ... (more)


Reference:

Grossman, S, Porth, CM (2014). Porth’s Pathophysiology Concepts of Altered Health States. 9thed. Philadelphia. Lippincott Williams & Wilkins, Chapter 45, Disorders of Gastrointestinal Function, p. 1183.

Question 4:

A client who is diagnosed with Zollinger-Ellison syndrome will exhibit which symptoms?

(see full question)

You selected:

Nausea with vomiting

Incorrect Correct response:

Diarrhea with fat deposits

Explanation:

Zollinger-Ellison syndrome causes clients to have diarrhea from hypersecretion or from the inactivation of intestinal lipase and impaired fat digestion. Diseases associated with ch ... (more)

Reference:

Grossman S, Porth CM (2014). Porth’s Pathophysiology: Concepts of Altered Health States. 9th ed. Philadelphia: Lippincott Williams & Wilkins, Chapter 45, Disorders of Gastrointestinal Function, p. 1182.

Question 5:

A patient is admitted to the medical surgical unit with a history of inflammatory bowel disease. The nurse knows that the clinical manifestations of both Crohn's disease and ulcerative colitis are the result of activation of which of the following cells?

(see full question)

You selected:

Parietal

Incorrect Correct response:

Inflammatory

Explanation:

The term "inflammatory bowel disease" is used to designate two related inflammatory intestinal disorders: Crohn's disease and ulcerative colitis. Both diseases produce inflammation ... (more)

Reference:

Grossman, S, Porth, CM (2014). Porth’s Pathophysiology Concepts of Altered Health States. 9thed. Philadelphia. Lippincott Williams & Wilkins, Chapter 45, Disorders of Gastrointestinal Function, pp. 1182, 1186.


(see full question)

A patient complains of having bouts of diarrhea. The patient wants to know which antidiarrheal would be best to use. Which of the following antidiarrheals does the nurse know will help stimulate water and electrolyte absorption?

You selected:

Bismuth subsalicylate (Pepto-Bismol)

Question 6:

Incorrect Correct response:

Diphenoxylate (Lomotil)

Explanation:

Drugs in the treatment of diarrhea include diphenoxylate (Lomotil) and loperamide (Imodium), which are opiumlike drugs. These drugs decrease gastrointestinal motility and stimulate ... (more)

Reference:

Grossman, S, Porth, CM (2014). Porth’s Pathophysiology Concepts of Altered Health States. 9thed. Philadelphia. Lippincott Williams & Wilkins, Chapter 45, Disorders of Gastrointestinal Function, p. 1195.

Question 7:

A client is admitted to the hospital with a suspected diagnosis of strangulated bowel. The nurse anticipates the client will need:

(see full question)

You selected:

Surgery to release the bowel

Correct Explanation:

Strangulation and complete bowel obstruction require surgical intervention. Nasogastric tubes are used for adynamic obstructions that result from neurogenic or muscular impairment ... (more)

Reference:

Grossman S, Porth CM (2014). Porth’s Pathophysiology: Concepts of Altered Health States. 9th ed. Philadelphia: Lippincott Williams & Wilkins, Chapter 45, Disorders of Gastrointestinal Function, p. 1198.

Question 8: (see full question)

A nurse is teaching a client diagnosed with Crohn disease about potential complications. The most appropriate information for the nurse to include would be:

You selected:

Chronic constipation


Incorrect Correct response:

Fistula formation

Explanation:

Complications of Crohn disease include fistula formation, abdominal abscess formation, and intestinal obstruction. Clients with Crohn disease are at risk for weight loss and/or diar ... (more)

Reference:

Grossman S, Porth CM (2014). Porth’s Pathophysiology: Concepts of Altered Health States. 9th ed. Philadelphia: Lippincott Williams & Wilkins, Chapter 45, Disorders of Gastrointestinal Function, p. 1186.

Question 9: (see full question)

A nurse reading a sigmoidoscopy report notes that a patient was found to have skip lesions. The nurse interprets this as an indication of which of the following?

You selected:

Crohn's disease

Correct Explanation:

Skip lesions, demarcated granulomatous lesions that are surrounded by normal-appearing mucosal tissue, are a characteristic feature of Crohn's disease.

Reference:

Grossman S, Porth CM (2014). Porth’s Pathophysiology: Concepts of Altered Health States. 9th ed. Philadelphia: Lippincott Williams & Wilkins, Chapter 45, Disorders of Gastrointestinal Function, p. 1186.

Question 10:

A patient is diagnosed with Crohn's disease. The nurse instructs the patient on which of the following types of dietary needs?

(see full question)

You selected:

High-fruit and vegetable intake

Incorrect Correct response:

High-calorie, vitamin, and protein diet

Explanation:

Nutritional deficiencies are common in Crohn's disease because of diarrhea, steatorrhea, and other malabsorption problems. A nutritious diet that is high in calories, vitamins, and ... (more)


Question 1: (see full question)

You selected:

Inflammatory bowel disease (IBD) is used to designate two related inflammatory intestinal disorders: Crohn's disease and ulcerative colitis. The nurse recognizes the difference between the distribution pattern between Crohn's disease and ulcerative colitis. Which of the following patterns describes Crohn's disease? Skip lesions

Correct Explanation:

Distribution patterns of disease with skip lesions in Crohn's disease, and continuous involvement of the colon starting at the rectum in ulcerative colitis. Crohn's disease primari ... (more)

Reference:

Grossman, S, Porth, CM (2014). Porth’s Pathophysiology Concepts of Altered Health States. 9thed. Philadelphia. Lippincott Williams & Wilkins, Chapter 45, Disorders of Gastrointestinal Function, p. 1185.

Question 2:

Which symptom is often observed in cases of peritonitis?

(see full question)

You selected:

Abdominal rigidity

Correct Explanation:

The abdomen is rigid and becomes boardlike because of reflex muscle guarding. The patient typically becomes tachycardiac, has increased WBC count, and breathes in a shallow way to ... (more)

Reference:

Grossman S, Porth CM (2014). Porth’s Pathophysiology: Concepts of Altered Health States. 9th ed. Philadelphia: Lippincott Williams & Wilkins, Chapter 45, Disorders of Gastrointestinal Function, p. 1198.

Question 3:

A patient with a history of peptic ulcer disease presents to the emergency department with the following symptoms: early satiety, feeling of epigastric fullness and heaviness after meals, weight loss, and vomiting. The nurse suspects that the peptic ulcer has caused which of the following problems?

(see full question)

You selected:

Obstruction


Correct Explanation:

Outlet obstruction is caused by edema, spasm, or contraction of scar tissue and interference with the free passage of gastric contents through the pylorus or adjacent areas. The pr ... (more)

Reference:

Grossman, S, Porth, CM (2014). Porth’s Pathophysiology Concepts of Altered Health States. 9thed. Philadelphia. Lippincott Williams & Wilkins, Chapter 45, Disorders of Gastrointestinal Function, p. 1181.

Question 4:

Which of the following clients should the nurse observe most closely for the signs and symptoms of paralytic ileus?

(see full question)

You selected:

A client who is first day postoperative following gallbladder surgery

Correct Explanation:

Paralytic ileus is a significant complication of abdominal surgery. The problem is not associated with the use of antidiarrheal medications, obesity, or irritable bowel syndrome.

Reference:

Grossman S, Porth CM (2014). Porth’s Pathophysiology: Concepts of Altered Health States. 9th ed. Philadelphia: Lippincott Williams & Wilkins, Chapter 45, Disorders of Gastrointestinal Function, p. 1197.

Question 5:

The patient who has experienced third-degree burns is susceptible to which of the following specific types of gastrointestinal (GI) ulceration?

(see full question)

You selected:

Stress

Correct Explanation:

Stress ulcer refer to GI ulcerations that develop in people with large-surfacearea burns.

Reference:

Grossman, S, Porth, CM (2014). Porth’s Pathophysiology Concepts of Altered Health States. 9thed. Philadelphia. Lippincott Williams & Wilkins, Chapter 45, Disorders of Gastrointestinal Function, p. 1185.


Question 6: (see full question)

You selected:

An older patient presents with a perforation of a peptic ulcer. The nurse will monitor for signs and symptoms of which of the following problems? Fecal impaction

Incorrect Correct response:

Peritonitis

Explanation:

Perforation occurs when an ulcer erodes through all the layers of the stomach or duodenum wall. With perforation, gastrointestinal contents enter the peritoneum and cause peritonit ... (more)

Reference:

Grossman, S, Porth, CM (2014). Porth’s Pathophysiology Concepts of Altered Health States. 9thed. Philadelphia. Lippincott Williams & Wilkins, Chapter 45, Disorders of Gastrointestinal Function, p. 1181.

Question 7:

The incidence of stomach cancer has significantly decreased in the United States, yet it remains the leading cause of death worldwide. The nurse understands the reason for the high mortality rate in stomach cancer is because of which of the following reasons?

(see full question)

You selected:

Patients have few early symptoms of the disease.

Correct Explanation:

Although the incidence of cancer of the stomach has declined over the past 50 years in the United States, it remains the leading cause of death worldwide. Because there are few ear ... (more)

Reference:

Grossman, S, Porth, CM (2014). Porth’s Pathophysiology Concepts of Altered Health States. 9thed. Philadelphia. Lippincott Williams & Wilkins, Chapter 45, Disorders of Gastrointestinal Function, p. 1183.

Question 8:

Ulcerative colitis is an inflammatory bowel disease. The nurse identifies which of the following as a common sign of ulcerative colitis?

(see full question)

You selected:

Mucus and blood stools

Correct Explanation:

Ulcerative colitis is a nonspecific inflammatory condition of the colon. Unlike


Crohn disease, which can affect various sites in the GI tract, ulcerative colitis is confined to the ... (more)

Reference:

Grossman, S, Porth, CM (2014). Porth’s Pathophysiology Concepts of Altered Health States. 9thed. Philadelphia. Lippincott Williams & Wilkins, Chapter 45, Disorders of Gastrointestinal Function, p. 1186; 1188.

Question 9:

The nurse caring for a client with diverticulitis should assess for which of the following clinical manifestations?

(see full question)

You selected:

Increased abdominal distention

Incorrect Correct response:

Lower left quadrant pain

Explanation:

Diverticulitis is a complication of diverticulosis in which there is inflammation and gross or microscopic perforation of the diverticulum with possible abscess formation and witho ... (more)

Reference:

Grossman S, Porth CM (2014). Porth’s Pathophysiology: Concepts of Altered Health States. 9th ed. Philadelphia: Lippincott Williams & Wilkins, Chapter 45, Disorders of Gastrointestinal Function, p. 1192.

Question 10: (see full question)

A patient is admitted with chronic gastritis. The nurse expects which of the following invasive tests to be performed to establish the presence of Helicobacter pylori (H. pylori)?

You selected:

Endoscopic biopsy

Correct Explanation:

Methods for establishing the presence of H. pylori infection include the carbon urea breath test, the stool antigen test, and an endoscopic biopsy for urease testing. The invasive ... (more)

Question 1:

A patient is admitted with chronic gastritis. The nurse expects which of the following invasive tests to be performed to establish the presence of Helicobacter pylori (H. pylori)?

(see full question)


You selected:

Endoscopic biopsy

Correct Explanation:

Methods for establishing the presence of H. pylori infection include the carbon urea breath test, the stool antigen test, and endoscopic biopsy for urease testing. The invasive tes ... (more)

Reference:

Grossman, S. Porth, CM (2014). Porth’s Pathophysiology Concepts of Altered Health States. 9thed. Philadelphia. Lippincott Williams & Wilkins, Chapter 45, Disorders of Gastrointestinal Function, p. 1179.

Question 2: (see full question)

Crohn's disease has a distinguishing pattern in the gastrointestinal (GI) tract. The surface has granulomatous lesions surrounded by normal-appearing mucosal tissue. A complication of the pattern includes which of the following?

You selected:

Fistula formation

Correct Explanation:

In Crohn's disease all layers of the bowel are involved. Complications of Crohn's disease include fistula formation, abdominal abscess formation, and intestinal obstruction. Fistul ... (more)

Reference:

Grossman, S, Porth, CM (2014). Porth’s Pathophysiology Concepts of Altered Health States. 9thed. Philadelphia. Lippincott Williams & Wilkins, Chapter 45, Disorders of Gastrointestinal Function, p. 1186.

Question 3:

The body uses common physiologic responses that are common to gastrointestinal (GI) disorders. A nurse would expect to see which of the following physiologic responses known to be protective by removing noxious agents from the body?

(see full question)

You selected:

Nausea

Incorrect Correct response:

Vomiting


Explanation:

Vomiting is the only protective response that removes noxious agents form the GI tract, although anorexia, nausea, and diarrhea are common physiologic responses to many GI disorder ... (more)

Reference:

Grossman, S, Porth, CM (2014). Porth’s Pathophysiology Concepts of Altered Health States. 9thed. Philadelphia. Lippincott Williams & Wilkins, Chapter 45, Disorders of Gastrointestinal Function, p. 1171.

Question 4:

The physician suspects a client has developed diverticular disease. The most appropriate test to confirm the diagnosis would be:

(see full question)

You selected:

CT scan

Correct Explanation:

Diverticular disease may be determined by CT scan or ultrasound. CT scans are the safest and most cost-effective. Due to risk of peritonitis, barium enema studies should be avoided ... (more)

Reference:

Grossman S, Porth CM (2014). Porth’s Pathophysiology: Concepts of Altered Health States. 9th ed. Philadelphia: Lippincott Williams & Wilkins, Chapter 45, Disorders of Gastrointestinal Function, p. 1192.

Question 5:

A patient presents with a perforated peptic ulcer. Which of the following complications would be a priority for the nurse?

(see full question)

You selected:

Assessment of a rigid, boardlike abdomen

Correct Explanation:

Perforation occurs when an ulcer erodes through all the layers of the stomach or duodenum wall. With perforation, gastrointestinal contents enter the peritoneum and cause peritonit ... (more)

Reference:

Grossman, S, Porth, CM (2014). Porth’s Pathophysiology Concepts of Altered Health States. 9thed. Philadelphia. Lippincott Williams & Wilkins, Chapter 45, Disorders of Gastrointestinal Function, p. 1181.


Question 6: (see full question)

You selected:

An elderly patient presents with loose mucousy stools. The nurse suspects the patient has Clostridium difficile. What is a priority assessment for the nurse? Ask the patient about his or her antibiotic use.

Correct Explanation:

The diagnosis of C. difficile-associated diarrhea requires a careful history, with particular emphasis on antibiotic use.

Reference:

Grossman, S, Porth, CM (2014). Porth’s Pathophysiology Concepts of Altered Health States. 9thed. Philadelphia. Lippincott Williams & Wilkins, Chapter 45, Disorders of Gastrointestinal Function, p. 1191.

Question 7:

In the treatment of Crohn's disease, many medications are used to mediate the inflammatory process. One specific medication that is used is infliximab, a monoclonal antibody. The nurse is aware that infliximab functions in which of the following ways?

(see full question)

You selected:

Targets destruction of tumor necrosis factor (TNF)

Correct Explanation:

Infliximab, a monoclonal antibody, targets the destruction of TNF, a mediator of the inflammatory response, whose expression is increased in inflammatory processes such as Crohn's ... (more)

Reference:

Grossman, S, Porth, CM (2014). Porth’s Pathophysiology Concepts of Altered Health States. 9thed. Philadelphia. Lippincott Williams & Wilkins, Chapter 45, Disorders of Gastrointestinal Function, p. 1188.

Question 8: (see full question)

An ultrasound has confirmed appendicitis as the cause of a 20-year-old man's sudden abdominal pain. Which of the following etiologic processes is implicated in the development of appendicitis?

You selected:

Obstruction of the intestinal lumen

Correct


Explanation:

Appendicitis is thought to be related to intraluminal obstruction with a fecalith or to twisting. Osmotic and bacterial changes are not thought to induce appendicitis, and the inte ... (more)

Reference:

Grossman S, Porth CM (2014). Porth’s Pathophysiology: Concepts of Altered Health States. 9th ed. Philadelphia: Lippincott Williams & Wilkins, Chapter 45, Disorders of Gastrointestinal Function, p. 1193.

Question 9:

A patient is admitted with an abrupt onset of referred pain to the epigastric area, with an episode of nausea. On the nurse's initial assessment, the patient is lying still and taking shallow breaths, with a rigid abdomen. Which of the following problems is the patient experiencing?

(see full question)

You selected:

Peptic ulcer

Incorrect Correct response:

Peritonitis

Explanation:

The onset of peritonitis may be acute, as with a ruptured appendix, or it may have a more gradual onset, as occurs in pelvic inflammatory disease. The pain usually is more intense ... (more)

Reference:

Grossman, S, Porth, CM (2014). Porth’s Pathophysiology Concepts of Altered Health States. 9thed. Philadelphia. Lippincott Williams & Wilkins, Chapter 45, Disorders of Gastrointestinal Function, p. 1198.

Question 10: (see full question)

An elderly patient has been placed on a broad-spectrum antibiotic for a recurrent urinary tract infection. Which of the following potential problems would the nurse anticipate in this patient?

You selected:

Colonization of Clostridium difficile

Correct Explanation:

Clostridium difficile colitis is associated with antibiotic therapy. Almost any antibiotic may cause C. difficile colitis, but broad-spectrum antibiotics with activity against Gram ... (more)


Question 1: (see full question)

You selected:

A rare condition caused by gastrin-secreting tumors most commonly found in the small intestine or pancreas is called which of the following? Zollinger-Ellison syndrome

Correct Explanation:

Zollinger-Ellison syndrome is a rare condition caused by a gastrin-secreting tumor (gastrinoma).

Reference:

Grossman, S, Porth, CM (2014). Porth’s Pathophysiology Concepts of Altered Health States. 9thed. Philadelphia. Lippincott Williams & Wilkins, Chapter 45, Disorders of Gastrointestinal Function, p. 1182.

Question 2: (see full question)

Following the analysis of a recent barium enema and colonoscopy with biopsy, a client has been diagnosed with colorectal cancer. Which of the following treatment modalities will be the mainstay of this client's treatment?

You selected:

Surgery

Correct Explanation:

The only recognized treatment for cancer of the colon and rectum is surgical removal. Postoperative radiation therapy may be used and has in some cases demonstrated increased 5-yea ... (more)

Reference:

Grossman S, Porth CM (2014). Porth’s Pathophysiology: Concepts of Altered Health States. 9th ed. Philadelphia: Lippincott Williams & Wilkins, Chapter 45, Disorders of Gastrointestinal Function, p. 1203.

Question 3: (see full question)

Celiac disease commonly presents in infancy as failure to thrive. It is an inappropriate T-cell–mediated immune response, and there is no cure for it. What is the treatment of choice for celiac disease?

You selected:

Removal of gluten from the diet

Correct Explanation:

The primary treatment of celiac disease consists of removal of gluten and related proteins from the diet. No other answer is correct.


Reference:

Grossman S, Porth CM (2014). Porth's Pathophysiology: Concepts of Altered Health States. 9th ed. Philadelphia: Lippincott Williams & Wilkins, Chapter 45: Disorders of Gastrointestinal Function, p. 1199.

Question 4:

The nurse caring for a client with diverticulitis should assess for which of the following clinical manifestations?

(see full question)

You selected:

Increased abdominal distention

Incorrect Correct response:

Lower left quadrant pain

Explanation:

Diverticulitis is a complication of diverticulosis in which there is inflammation and gross or microscopic perforation of the diverticulum with possible abscess formation and witho ... (more)

Reference:

Grossman S, Porth CM (2014). Porth’s Pathophysiology: Concepts of Altered Health States. 9th ed. Philadelphia: Lippincott Williams & Wilkins, Chapter 45, Disorders of Gastrointestinal Function, p. 1192.

Question 5:

Constipation is the infrequent or difficult passage of stools.

(see full question)

You selected:

True

Correct Explanation:

Constipation can be defined as the infrequent, incomplete, or difficult passage of stools.

Reference:

Grossman S, Porth CM (2014). Porth’s Pathophysiology: Concepts of Altered Health States. 9th ed. Philadelphia: Lippincott Williams & Wilkins, Chapter 45, Disorders of Gastrointestinal Function, p. 1196.

Question 6: (see full question)

An elderly patient with a history of severe constipation tells the nurse that he has been experiencing watery stool and fecal incontinence the last 2 days. The nurse suspects which of the following?

You selected:

Fecal impaction


Correct Explanation:

Fecal impaction should be suspected in elderly and/or immobilized patients who develop watery stools with fecal and/or urinary incontinence.

Reference:

Grossman S, Porth CM (2014). Porth’s Pathophysiology: Concepts of Altered Health States. 9th ed. Philadelphia: Lippincott Williams & Wilkins, Chapter45, Disorder of Gastrointestinal Function, p. 1196.

Question 7:

While assessing a client diagnosed with inflammatory bowel diseases, the nurse should assess for systemic manifestations that may include: Select all that apply.

(see full question)

You selected:

• Autoimmune anemia • Lactose intolerance • Mouth ulcerations

Incorrect Correct response:

• Autoimmune anemia • Lactose intolerance

Explanation:

Inflammatory bowel diseases produce inflammation of the bowel, with a lack of confirming evidence of a proven causative agent, have a pattern of familial occurrence, and can be acc ... (more)

Reference:

Grossman S, Porth CM (2014). Porth’s Pathophysiology: Concepts of Altered Health States. 9th ed. Philadelphia: Lippincott Williams & Wilkins, Chapter 45, Disorders of Gastrointestinal Function, p. 1186.

Question 8:

The patient who has experienced third-degree burns is susceptible to which of the following specific types of gastrointestinal (GI) ulceration?

(see full question)

You selected:

Gastric

Incorrect Correct response:

Stress

Explanation:

Stress ulcer refer to GI ulcerations that develop in people with large-surfacearea burns.


Reference:

Grossman, S, Porth, CM (2014). Porth’s Pathophysiology Concepts of Altered Health States. 9thed. Philadelphia. Lippincott Williams & Wilkins, Chapter 45, Disorders of Gastrointestinal Function, p. 1185.

Question 9: (see full question)

A patient diagnosed with ulcerative colitis has been experiencing more than six bloody stools daily with evidence of toxicity. The nurse should question which of the following orders from the physician?

You selected:

Physical examination

Incorrect Correct response:

Colonoscopy

Explanation:

Diagnosis of ulcerative colitis is based on history and physical examination. The diagnosis usually is confirmed by sigmoidoscopy, colonoscopy, biopsy, and by negative stool examin ... (more)

Reference:

Grossman, S, Porth, CM (2014). Porth’s Pathophysiology Concepts of Altered Health States. 9thed. Philadelphia. Lippincott Williams & Wilkins, Chapter 45, Disorders of Gastrointestinal Function, p. 1189.

Question 10:

A nurse caring for a newborn infant with a tracheoesophageal fistula (TEF) includes which of the following?

(see full question)

You selected:

Avoiding feedings

Correct Explanation:

The newborn infant with TEF has frothing and bubbling at the mouth and nose and episodes of coughing, vomiting, cyanosis, and respiratory distress, which can be exacerbated by oral ... (more)

Question 1:

Crohn's type of inflammatory bowel disease is characterized by which of the following?

(see full question)

You selected:

Granulomatous lesions

Correct Explanation:

Crohn's disease is a recurrent, granulomatous type of inflammatory response


with formation of multiple sharply demarcated, granulomatous lesions that are surrounded by normal-appea ... (more)

Reference:

Grossman S, Porth CM (2014). Porth’s Pathophysiology: Concepts of Altered Health States. 9th ed. Philadelphia: Lippincott Williams & Wilkins, Chapter 45, Disorders of Gastrointestinal Function, p. 1186.

Question 2:

A patient is diagnosed with Crohn's disease. The nurse instructs the patient on which of the following types of dietary needs?

(see full question)

You selected:

High-calorie, vitamin, and protein diet

Correct Explanation:

Nutritional deficiencies are common in Crohn's disease because of diarrhea, steatorrhea, and other malabsorption problems. A nutritious diet that is high in calories, vitamins, and ... (more)

Reference:

Grossman, S, Porth, CM (2014). Porth’s Pathophysiology Concepts of Altered Health States. 9thed. Philadelphia. Lippincott Williams & Wilkins, Chapter 45, Disorders of Gastrointestinal Function, p. 1188.

Question 3:

The parietal cells of the stomach produce hydrochloric acid to protect the mucosal covering from gastric erosion. The nurse understands that Helicobacter pylori (H. pylori) produces an enzyme that enables its survival in this acidic environment. Which of the following enzymes does H. pylori produce?

(see full question)

You selected:

Urease

Correct Explanation:

As the most common cause of chronic gastritis, H. pylori is able to buffer the acidity of their immediate environment by producing sufficient ammonia, which is created from secreti ... (more)

Reference:

Grossman, S. Porth, CM (2014). Porth’s Pathophysiology Concepts of Altered Health States. 9thed. Philadelphia. Lippincott Williams & Wilkins, Chapter 45,


Disorders of Gastrointestinal Function, p. 1179.

Question 4: (see full question)

You selected:

A client has been admitted to the hospital with an exacerbation of peptic ulcer disease. The nurse is aware the client is at risk for: Select all that apply. • Perforation • Hemorrhage • Obstruction

Correct Explanation:

The most common complications of peptic ulcer are hemorrhage, perforation, and gastric outlet obstruction. Weight gain and increased urinary output would not occur as the client ma ... (more)

Reference:

Grossman, S. & Porth, C.M. Porth's Pathophysiology: Concepts of Altered Health States, 9th ed., Philadelphia: Lippincott Williams & Wilkins, 2014, Chapter 45, Disorders of Gastrointestinal Function, p. 1181.

Question 5:

A client who is diagnosed with Zollinger-Ellison syndrome will exhibit which symptoms?

(see full question)

You selected:

Diarrhea with fat deposits

Correct Explanation:

Zollinger-Ellison syndrome causes clients to have diarrhea from hypersecretion or from the inactivation of intestinal lipase and impaired fat digestion. Diseases associated with ch ... (more)

Reference:

Grossman S, Porth CM (2014). Porth’s Pathophysiology: Concepts of Altered Health States. 9th ed. Philadelphia: Lippincott Williams & Wilkins, Chapter 45, Disorders of Gastrointestinal Function, p. 1182.

Question 6:

The nurse knows that her student understands the major causes of mechanical bowel obstruction when the student tells her that they include which of the following?

(see full question)


You selected:

Abdominal distention

Incorrect Correct response:

Postoperative adhesions

Explanation:

Major triggering causes of mechanical bowel obstruction include external hernia (i.e., inguinal, femoral, or umbilical) and postoperative adhesions. Paralytic (adynamic) obstructio ... (more)

Reference:

Grossman S, Porth CM (2014). Porth’s Pathophysiology: Concepts of Altered Health States. 9th ed. Philadelphia: Lippincott Williams & Wilkins, Chapter 45, Disorders of Gastrointestinal Function, p. 1196.

Question 7: (see full question)

A patient is admitted with an intestinal obstruction with abdominal bloating. Which of the following types of treatment does the nurse know will help decompress the bowel?

You selected:

Rectal tube to suction

Incorrect Correct response:

Nasogastric tube to suction

Explanation:

The cardinal symptoms of intestinal obstruction are pain, absolute constipation, abdominal distention, sign of fluid volume deficit, and vomiting. Treatment depends on the cause an ... (more)

Reference:

Grossman, S, Porth, CM (2014). Porth’s Pathophysiology Concepts of Altered Health States. 9thed. Philadelphia. Lippincott Williams & Wilkins, Chapter 45, Disorders of Gastrointestinal Function, p. 1198.

Question 8:

The physician suspects a client has developed diverticular disease. The most appropriate test to confirm the diagnosis would be:

(see full question)

You selected:

Flat-plate x-ray

Incorrect Correct response:

CT scan

Explanation:

Diverticular disease may be determined by CT scan or ultrasound. CT scans


are the safest and most cost-effective. Due to risk of peritonitis, barium enema studies should be avoided ... (more)

Reference:

Grossman S, Porth CM (2014). Porth’s Pathophysiology: Concepts of Altered Health States. 9th ed. Philadelphia: Lippincott Williams & Wilkins, Chapter 45, Disorders of Gastrointestinal Function, p. 1192.

Question 9: (see full question)

Infants and children commonly have gastroesophageal reflux. Many times it is asymptomatic and resolves on its own. What are the signs and symptoms of gastroesophageal reflux in infants with severe disease?

You selected:

Tilting of the head to one side and arching of the back

Correct Explanation:

Tilting of the head to one side and arching of the back may be noted in children with severe reflux. Early satiety is another indication of gastroesophageal reflux, but not c ... (more)

Reference:

Grossman S, Porth CM (2014). Porth's Pathophysiology: Concepts of Altered Health States. 9th ed. Philadelphia: Lippincott Williams & Wilkins, Chapter 45: Disorders of Gastrointestinal Function, p. 1176.

Question 10: (see full question)

A patient complains of having bouts of diarrhea. The patient wants to know which antidiarrheal would be best to use. Which of the following antidiarrheals does the nurse know will help stimulate water and electrolyte absorption?

You selected:

Diphenoxylate (Lomotil)

Correct Explanation:

Drugs in the treatment of diarrhea include diphenoxylate (Lomotil) and loperamide (Imodium), which are opiumlike drugs. These drugs decrease gastrointestinal motility and stimulate ... (more)

Question 1:

A patient is admitted with an exacerbation of Crohn's disease and has been experiencing a weight loss of 20 pounds in 2 weeks. The patient is unable to

(see full question)


absorb nutrients from the intestine. The nurse would expect which of the following types of diet to be ordered for this patient? You selected:

High-calorie, low-fat diet

Incorrect Correct response:

Total parenteral nutrition

Explanation:

Total parenteral nutrition consists of intravenous administration of hypertonic glucose solutions to which amino acids and fats may be added. This form of nutritional therapy may b ... (more)

Reference:

Grossman, S, Porth, CM (2014). Porth’s Pathophysiology Concepts of Altered Health States. 9thed. Philadelphia. Lippincott Williams & Wilkins, Chapter 45, Disorders of Gastrointestinal Function, p. 1188.

Question 2:

The physician thinks a teenager is having clinical manifestations of irritable bowel syndrome. Which complaints would support this diagnosis? Select all that apply.

(see full question)

You selected:

• Pain is most severe at night. • Pain is relieved by defecation. • Pain is worse after and between meals.

Incorrect Correct response:

• Pain is relieved by defecation. • Pain is described as “cramping” in the lower abdomen.

Explanation:

A hallmark of irritable bowel syndrome is abdominal pain that is relieved by defecation and associated with a change in consistency or frequency of stools. Irritable bowel disease ... (more)

Reference:

rossman S, Porth CM (2014). Porth’s Pathophysiology: Concepts of Altered Health States. 9th ed. Philadelphia: Lippincott Williams & Wilkins, Chapter 45, Disorders of Gastrointestinal Function, p. 1184.

Question 3:

Inflammatory bowel disease (IBD) is used to designate two related inflammatory intestinal disorders: Crohn's disease and ulcerative colitis. The nurse recognizes the difference between the distribution pattern between

(see full question)


Crohn's disease and ulcerative colitis. Which of the following patterns describes Crohn's disease? You selected:

Skip lesions

Correct Explanation:

Distribution patterns of disease with skip lesions in Crohn's disease, and continuous involvement of the colon starting at the rectum in ulcerative colitis. Crohn's disease primari ... (more)

Reference:

Grossman, S, Porth, CM (2014). Porth’s Pathophysiology Concepts of Altered Health States. 9thed. Philadelphia. Lippincott Williams & Wilkins, Chapter 45, Disorders of Gastrointestinal Function, p. 1185.

Question 4: (see full question)

A patient diagnosed with ulcerative colitis has been experiencing more than six bloody stools daily with evidence of toxicity. The nurse should question which of the following orders from the physician?

You selected:

Physical examination

Incorrect Correct response:

Colonoscopy

Explanation:

Diagnosis of ulcerative colitis is based on history and physical examination. The diagnosis usually is confirmed by sigmoidoscopy, colonoscopy, biopsy, and by negative stool examin ... (more)

Reference:

Grossman, S, Porth, CM (2014). Porth’s Pathophysiology Concepts of Altered Health States. 9thed. Philadelphia. Lippincott Williams & Wilkins, Chapter 45, Disorders of Gastrointestinal Function, p. 1189.

Question 5: (see full question)

A patient is admitted with chronic gastritis. The nurse expects which of the following invasive tests to be performed to establish the presence of Helicobacter pylori (H. pylori)?

You selected:

Endoscopic biopsy

Correct Explanation:

Methods for establishing the presence of H. pylori infection include the carbon


urea breath test, the stool antigen test, and an endoscopic biopsy for urease testing. The invasive ... (more)

Reference:

Grossman, S. Porth, CM (2014). Porth’s Pathophysiology Concepts of Altered Health States. 9thed. Philadelphia. Lippincott Williams & Wilkins, Chapter 45, Disorders of Gastrointestinal Function, p. 1179.

Question 6:

A teenager has been diagnosed with failure to thrive. The health care providers suspect a malabsorption syndrome. In addition to having diarrhea and bloating, the client more than likely has what hallmark manifestation of malabsorption?

(see full question)

You selected:

Fatty, yellow-gray, foul-smelling stools

Correct Explanation:

General symptoms of malabsorption syndrome include diarrhea, flatulence, bloating, cramping, and weight loss. A hallmark of malabsorption is steatorrhea, characterized by fatty, ye ... (more)

Reference:

Grossman, S. & Porth, C.M. Porth's Pathophysiology: Concepts of Altered Health States, 9th ed., Philadelphia: Lippincott Williams & Wilkins, 2014, Chapter 45, Disorders of Gastrointestinal Function, p. 1199.

Question 7:

A nurse caring for a newborn infant with a tracheoesophageal fistula (TEF) includes which of the following?

(see full question)

You selected:

Keeping infant supine

Incorrect Correct response:

Avoiding feedings

Explanation:

The newborn infant with TEF has frothing and bubbling at the mouth and nose and episodes of coughing, vomiting, cyanosis, and respiratory distress, which can be exacerbated by oral ... (more)

Reference:

Grossman S, Porth CM (2014). Porth’s Pathophysiology: Concepts of Altered Health States. 9th ed. Philadelphia: Lippincott Williams & Wilkins, Chapter 45,


Disorders of Gastrointestinal Function, p. 1173.

Question 8: (see full question)

You selected:

Which of the following clients should the nurse observe most closely for the signs and symptoms of paralytic ileus? A client who is first day postoperative following gallbladder surgery

Correct Explanation:

Paralytic ileus is a significant complication of abdominal surgery. The problem is not associated with the use of antidiarrheal medications, obesity, or irritable bowel syndrome.

Reference:

Grossman S, Porth CM (2014). Porth’s Pathophysiology: Concepts of Altered Health States. 9th ed. Philadelphia: Lippincott Williams & Wilkins, Chapter 45, Disorders of Gastrointestinal Function, p. 1197.

Question 9:

While assessing a client diagnosed with inflammatory bowel diseases, the nurse should assess for systemic manifestations that may include: Select all that apply.

(see full question)

You selected:

• Rheumatoid arthritis • Mouth ulcerations • Autoimmune anemia • Lactose intolerance

Incorrect Correct response:

• Autoimmune anemia • Lactose intolerance

Explanation:

Inflammatory bowel diseases produce inflammation of the bowel, with a lack of confirming evidence of a proven causative agent, have a pattern of familial occurrence, and can be acc ... (more)

Reference:

Grossman S, Porth CM (2014). Porth’s Pathophysiology: Concepts of Altered Health States. 9th ed. Philadelphia: Lippincott Williams & Wilkins, Chapter 45, Disorders of Gastrointestinal Function, p. 1186.


(see full question)

An elderly patient with a history of severe constipation tells the nurse that he has been experiencing watery stool and fecal incontinence the last 2 days. The nurse suspects which of the following?

You selected:

Clostridium difficile

Question 10:

Incorrect Correct response:

Fecal impaction

Explanation:

Fecal impaction should be suspected in elderly and/or immobilized patients who develop watery stools with fecal and/or urinary incontinence.

Question 1:

The incidence of stomach cancer has significantly decreased in the United States, yet it remains the leading cause of death worldwide. The nurse understands the reason for the high mortality rate in stomach cancer is because of which of the following reasons?

(see full question)

You selected:

Patients have few early symptoms of the disease.

Correct Explanation:

Although the incidence of cancer of the stomach has declined over the past 50 years in the United States, it remains the leading cause of death worldwide. Because there are few ear ... (more)

Reference:

Grossman, S, Porth, CM (2014). Porth’s Pathophysiology Concepts of Altered Health States. 9thed. Philadelphia. Lippincott Williams & Wilkins, Chapter 45, Disorders of Gastrointestinal Function, p. 1183.

Question 2:

In the treatment of Crohn's disease, many medications are used to mediate the inflammatory process. One specific medication that is used is infliximab, a monoclonal antibody. The nurse is aware that infliximab functions in which of the following ways?

(see full question)

You selected:

Targets destruction of tumor necrosis factor (TNF)

Correct Explanation:

Infliximab, a monoclonal antibody, targets the destruction of TNF, a mediator of the inflammatory response, whose expression is increased in inflammatory processes such as Crohn's ... (more)


Reference:

Grossman, S, Porth, CM (2014). Porth’s Pathophysiology Concepts of Altered Health States. 9thed. Philadelphia. Lippincott Williams & Wilkins, Chapter 45, Disorders of Gastrointestinal Function, p. 1188.

Question 3: (see full question)

A patient is admitted to an acute care facility with a Clostridium difficile infection. The nurse anticipates administering which of the following medications?

You selected:

Metronidazole

Correct Explanation:

Metronidazole is the medication of choice for treatment of C. difficile infections. It may be given intravenously or orally. Vancomycin is reserved for people who cannot tolerate m ... (more)

Reference:

Grossman S, Porth CM (2014). Porth’s Pathophysiology: Concepts of Altered Health States. 9th ed. Philadelphia: Lippincott Williams & Wilkins, Chapter 45, Disorders of Gastrointestinal Function, p. 1191.

Question 4:

A client has been admitted to the hospital with an exacerbation of peptic ulcer disease. The nurse is aware the client is at risk for: Select all that apply.

(see full question)

You selected:

• Hemorrhage • Obstruction • Weight gain • Perforation

Incorrect Correct response:

• Perforation • Hemorrhage • Obstruction

Explanation:

The most common complications of peptic ulcer are hemorrhage, perforation, and gastric outlet obstruction. Weight gain and increased urinary output would not occur as the client ma ... (more)


Reference:

Grossman, S. & Porth, C.M. Porth's Pathophysiology: Concepts of Altered Health States, 9th ed., Philadelphia: Lippincott Williams & Wilkins, 2014, Chapter 45, Disorders of Gastrointestinal Function, p. 1181.

Question 5:

A patient presents with a perforated peptic ulcer. Which of the following complications would be a priority for the nurse?

(see full question)

You selected:

Assessment of a rigid, boardlike abdomen

Correct Explanation:

Perforation occurs when an ulcer erodes through all the layers of the stomach or duodenum wall. With perforation, gastrointestinal contents enter the peritoneum and cause peritonit ... (more)

Reference:

Grossman, S, Porth, CM (2014). Porth’s Pathophysiology Concepts of Altered Health States. 9thed. Philadelphia. Lippincott Williams & Wilkins, Chapter 45, Disorders of Gastrointestinal Function, p. 1181.

Question 6: (see full question)

A patient is admitted with chronic gastritis. The nurse expects which of the following invasive tests to be performed to establish the presence of Helicobacter pylori (H. pylori)?

You selected:

Endoscopic biopsy

Correct Explanation:

Methods for establishing the presence of H. pylori infection include the carbon urea breath test, the stool antigen test, and endoscopic biopsy for urease testing. The invasive tes ... (more)

Reference:

Grossman, S. Porth, CM (2014). Porth’s Pathophysiology Concepts of Altered Health States. 9thed. Philadelphia. Lippincott Williams & Wilkins, Chapter 45, Disorders of Gastrointestinal Function, p. 1179.

Question 7:

The body uses common physiologic responses that are common to gastrointestinal (GI) disorders. A nurse would expect to see which of the

(see full question)


following physiologic responses known to be protective by removing noxious agents from the body? You selected:

Vomiting

Correct Explanation:

Vomiting is the only protective response that removes noxious agents form the GI tract, although anorexia, nausea, and diarrhea are common physiologic responses to many GI disorder ... (more)

Reference:

Grossman, S, Porth, CM (2014). Porth’s Pathophysiology Concepts of Altered Health States. 9thed. Philadelphia. Lippincott Williams & Wilkins, Chapter 45, Disorders of Gastrointestinal Function, p. 1171.

Question 8:

Which symptom is often observed in cases of peritonitis?

(see full question)

You selected:

Abdominal rigidity

Correct Explanation:

The abdomen is rigid and becomes boardlike because of reflex muscle guarding. The patient typically becomes tachycardiac, has increased WBC count, and breathes in a shallow way to ... (more)

Reference:

Grossman S, Porth CM (2014). Porth’s Pathophysiology: Concepts of Altered Health States. 9th ed. Philadelphia: Lippincott Williams & Wilkins, Chapter 45, Disorders of Gastrointestinal Function, p. 1198.

Question 9:

A patient is admitted to the medical surgical unit with a history of inflammatory bowel disease. The nurse knows that the clinical manifestations of both Crohn's disease and ulcerative colitis are the result of activation of which of the following cells?

(see full question)

You selected:

Inflammatory

Correct Explanation:

The term "inflammatory bowel disease" is used to designate two related inflammatory intestinal disorders: Crohn's disease and ulcerative colitis. Both diseases produce inflammation ... (more)


Reference:

Grossman, S, Porth, CM (2014). Porth’s Pathophysiology Concepts of Altered Health States. 9thed. Philadelphia. Lippincott Williams & Wilkins, Chapter 45, Disorders of Gastrointestinal Function, pp. 1182, 1186.

Question 10:

An older patient presents with a perforation of a peptic ulcer. The nurse will monitor for signs and symptoms of which of the following problems?

(see full question)

You selected:

Peritonitis

Correct Explanation:

Perforation occurs when an ulcer erodes through all the layers of the stomach or duodenum wall. With perforation, gastrointestinal contents enter the peritoneum and cause peritonit ... (more)

Question 1: (see full question)

A client newly diagnosed with pancreatic cancer is admitted to begin treatment. Which pain discriptors can be associated with adenocarcinomas of the pancreas?

You selected:

Abdominal pain following eating a large meal heavy with fat and protein.

Incorrect Correct response:

Dull epigastric pain accompanied by back pain, worse when lying flat and relieved by sitting forward.

Explanation:

The most common pain [with pancreatic cancer] is a dull epigastric pain often accompanied by back pain, often worse in the supine position, and relieved by sitting forward. Sharp, ... (more)

Reference:

Grossman S, Porth CM (2014). Porth’s Pathophysiology: Concepts of Altered Health States. 9th ed. Philadelphia: Lippincott Williams & Wilkins, Chapter 46, Disorders of the Hepatobiliary and Exocrine Pancreas Function, p. 1239.

Question 2:

The community health nurse is teaching about prevention of hepatitis A. Which of these groups does the nurse suggest will benefit from this vaccine?

(see full question)

You selected:

Those traveling to third world countries


Correct Explanation:

International hepatitis A vaccine is suggested for travelers to regions where sanitation is poor and endemic HAV infections are high, children living in communities with high rates ... (more)

Reference:

Grossman S, Porth CM (2014). Porth’s Pathophysiology: Concepts of Altered Health States. 9th ed. Philadelphia: Lippincott Williams & Wilkins, Chapter 46, Disorders of Hepatobiliary and Exocrine Pancreas Function, p. 1221.

Question 3:

A major factor in the development of hepatic encephalopathy is:

(see full question)

You selected:

Neurotoxin accumulation

Correct Explanation:

Although the cause of hepatic encephalopathy is unknown, the accumulation of neurotoxins, which appear in the blood because the liver has lost its detoxifying capacity, is believed ... (more)

Reference:

Grossman S, Porth CM (2014). Porth’s Pathophysiology: Concepts of Altered Health States. 9th ed. Philadelphia: Lippincott Williams & Wilkins, Chapter 46, Disorders of Hepatobilary and Exocrine Pancreas Function, p. 1232.

Question 4:

Which of these assessments does the nurse recognize is consistent with signs and symptoms of acute pancreatitis?

(see full question)

You selected:

Periumbilical pain

Correct Explanation:

Abdominal pain is a cardinal manifestation of acute pancreatitis. The pain is usually located in the epigastric or periumbilical region and may radiate to the back, chest, or flank ... (more)

Reference:

Grossman S, Porth CM (2014). Porth’s Pathophysiology: Concepts of Altered Health States. 9th ed. Philadelphia: Lippincott Williams & Wilkins, Chapter 46, Disorders of Hepatobiliary and Exocrine Pancreas Function, p. 1238.


Question 5: (see full question)

You selected:

When assessing the client during the icteric phase of viral hepatitis, which of these findings does the nurse anticipate observing? Yellow-tinged sclera

Correct Explanation:

The icterus phase is reflected by development of jaundice of skin and sclera, liver tenderness and worsened prodromal symptoms.

Reference:

Grossman S, Porth CM (2014). Porth’s Pathophysiology: Concepts of Altered Health States. 9th ed. Philadelphia: Lippincott Williams & Wilkins, Chapter 46, Disorders of Hepatobiliary and Exocrine Pancreas Function, p. 1220.

Question 6:

When assessing the client with acute pancreatitis, which of these diagnostic tests, consistent with the disease, does the nurse anticipate will be altered?

(see full question)

You selected:

Amylase and lipase

Correct Explanation:

Serum amylase and lipase are the laboratory markers most commonly used to establish a diagnosis of acute pancreatitis.

Reference:

Grossman S, Porth CM (2014). Porth’s Pathophysiology: Concepts of Altered Health States. 9th ed. Philadelphia: Lippincott Williams & Wilkins, Chapter 46, Disorders of Hepatobiliary and Exocrine Pancreas Function, p. 1238.

Question 7:

An ultrasound (US) of a client with intermittent pain reveals gallbladder sludge. Which of the following client history items are likely factors in the US result? Select all that apply.

(see full question)

You selected:

Correct

• Current prescription for a medicine to lower cholesterol. • Recent pregnancy with a 6-month-old child at home • Had lap band surgery 2 years ago and lost 100 pounds


Explanation:

Three factors contribute to the formation of gallstones: abnormalities in the composition of bile, stasis of bile (rather than rapid elimination), and inflammation of the gallbladd ... (more)

Reference:

Grossman, S. & Porth, C.M. Porth's Pathophysiology: Concepts of Altered Health States, 9th ed., Philadelphia: Lippincott Williams & Wilkins, 2014, Chapter 46, Disorders of Hepatobiliary and Exocrine Pancreas Function, p. 1235.

Question 8: (see full question)

A client tells the nurse that he is concerned about developing hepatitis after being exposed to contaminated feces, saliva, and food. The nurse is aware that the client is at risk for:

You selected:

Hepatitis A

Correct Explanation:

Hepatitis A is normally transmitted through the fecal–oral route by drinking contaminated milk or water and eating shellfish from infected waters. Hepatitis B is transmitted thro ... (more)

Reference:

Grossman S, Porth CM (2014). Porth’s Pathophysiology: Concepts of Altered Health States. 9th ed. Philadelphia: Lippincott Williams & Wilkins, Chapter 46, Disorders of Hepatobiliary and Exocrine Pancreas Function, p. 1221.

Question 9:

A nurse is reviewing laboratory results of a client that were ordered to assess liver function .The results indicating injury to liver cells would include:

(see full question)

You selected:

Liver enzymes

Correct Explanation:

Serum liver enzymes, especially alanine aminotransferase (ALT) and aspartate aminotransferase (AST), are used to assess injury to liver cells; plasma proteins (e.g., serum albumin) ... (more)

Reference:

Grossman, S. & Porth, C.M. Porth's Pathophysiology: Concepts of Altered Health States, 9th ed., Philadelphia: Lippincott Williams & Wilkins, 2014,


Chapter 46, Disorders of Hepatobiliary and Exocrine Function, p. 1217.

Question 10: (see full question)

You selected:

A client with a history of cancer that metastasized to the liver has arrived at the outpatient clinic to have a paracentesis performed. The physician anticipates that the client will have more than 5 L of fluid removed. The physician has prescribed intravenous albumin following the procedure. The client asks why she needs, "more fluids in my vein?" The nurse responds: "Albumin is a volume expander. Since a lot of fluid was removed, you have a decrease in your vascular volume, so without this albumin, your kidneys will try to reabsorb and hold onto water."

Correct Explanation:

Large-volume paracentesis (removal of 5 L or more of ascitic fluid) may be done in persons with massive ascites and pulmonary compromise. Because the removal of fluid produces a de ... (more)

Question 1:

The nurse is caring for a female client with cholelithiasis. When teaching the client about the disease, the nurse includes which of these points?

(see full question)

You selected:

"Gallstones have developed, which are typically composed of cholesterol."

Correct Explanation:

Cholelithiasis or gallstones is caused by precipitation of substances contained in bile, mainly cholesterol and bilirubin. It is most common in women, multiple pregnancies, those t ... (more)

Reference:

Grossman S, Porth CM (2014). Porth’s Pathophysiology: Concepts of Altered Health States. 9th ed. Philadelphia: Lippincott Williams & Wilkins, Chapter 46, Disorders of Hepatobiliary and Exocrine Pancreas Function, p. 1236.

Question 2:

A client with multiple pain-related injuries to the back, knees, and hips is admitted with acute liver failure. Upon procuring a medication list, the nurse notes that the client is taking several over-the-counter medications that contain a preparation known to be the drug that most commonly causes liver failure. Which drug is in the client's medications?

(see full question)

You selected:

Acetaminophen


Correct Explanation:

The drug most commonly involved is acetaminophen, with half the cases reported to be unintentional overdoses. Unintentional overdoses may occur when people unknowingly take several ... (more)

Reference:

Grossman S, Porth CM (2014). Porth’s Pathophysiology: Concepts of Altered Health States. 9th ed. Philadelphia: Lippincott Williams & Wilkins, Chapter 46, Disorders of Hepatobiliary and Exocrine Pancreas Function, p. 1219.

Question 3: (see full question)

The nurse is caring for a client with right upper quadrant pain secondary to acute choledocholithiasis. The nurse anticipates if the common bile duct becomes obstructed, manifestations will also include which of these?

You selected:

Vomiting

Incorrect Correct response:

Hyperbilirubinemia

Explanation:

Choledocholithiasis, stones in the common duct, usually originate in the gallbladder, but can form spontaneously in the common duct. Bilirubinuria and an elevated serum bilirubin ( ... (more)

Question 4: (see full question)

A 16-year-old girl has been admitted to the emergency department after ingesting 20 g of acetaminophen (Tylenol) in a suicide attempt. The care team would recognize that this patient faces risk for which of these complications?

You selected:

Toxic hepatitis

Correct Explanation:

Among the manifestations of the direct, predictable liver injuries that accompany overdoses of acetaminophen is toxic hepatitis with tissue necrosis. HDV infection and secondary bi ... (more)

Reference:

Grossman S, Porth CM (2014). Porth’s Pathophysiology: Concepts of Altered Health States. 9th ed. Philadelphia: Lippincott Williams & Wilkins, Chapter 46, Disorders of Hepatobilary and Exocrine Pancreas Function, p. 1219.


(see full question)

A client is suspected of having the onset of alcoholic liver disease. The nurse should be assessing for which of the following manifestations related to the necrosis of liver cells?

You selected:

Tremors of the hands

Question 5:

Incorrect Correct response:

Rapid onset of jaundice

Explanation:

Alcoholic hepatitis is the intermediate stage between fatty changes and cirrhosis and is characterized by inflammation and necrosis of liver cells. The condition is always serious ... (more)

Reference:

Grossman, S. & Porth, C.M. Porth's Pathophysiology: Concepts of Altered Health States, 9th ed., Philadelphia: Lippincott Williams & Wilkins, 2014, Chapter 46, Disorders of Hepatobiliary and Exocrine Pancreas Function, p. 1227.

Question 6: (see full question)

When teaching a group of nursing students about the liver, the nurse relates that Kupffer cells function to remove harmful substances or cells from the portal blood and venous sinusoids through which of these processes?

You selected:

Phagocytosis

Correct Explanation:

Kupffer cells, which line the venous sinusoids are reticuloendothelial cells that are capable of removing, engulfing, and phagocytizing old and defective blood cells, bacteria, and ... (more)

Reference:

Grossman S, Porth CM (2014). Porth’s Pathophysiology: Concepts of Altered Health States. 9th ed. Philadelphia: Lippincott Williams & Wilkins, Chapter 46, Disorders of Hepatobilary and Exocrine Pancreas Function, p. 1210.

Question 7:

When caring for the client with acute pancreatitis, which of these alterations does the nurse recognize is consistent with the disease?

(see full question)

You selected: Incorrect

Leukopenia


Correct response:

Hyperglycemia

Explanation:

Serum amylase and lipase are the laboratory markers most commonly used to establish a diagnosis of acute pancreatitis. The white blood cell count may be increased, and hyperglycemi ... (more)

Reference:

Porth CM (2011). Essentials of Pathophysiology: Concepts of Altered Health States. 3rd ed. Philadelphia: Lippincott Williams & Wilkins, Chapter 30, Disorders of Hepatobiliary and Exocrine Pancreas Function, p. 1238.

Question 8: (see full question)

A client has begun to display manifestations of hepatic encephalopathy. The family is concerned and asks the nurse what caused this condition to develop. Which is the best response by the nurse?

You selected:

Accumulation of ammonia in the blood

Correct Explanation:

Hepatic encephalopathy occurs in liver failure and results from the liver being unable to convert ammonia to urea. Ammonia moves directly into the general circulation and from ther ... (more)

Reference:

Grossman S, Porth CM (2014). Porth’s Pathophysiology: Concepts of Altered Health States. 9th ed. Philadelphia: Lippincott Williams & Wilkins, Chapter 46, Disorders of Hepatobiliary and Exocrine Pancreas Function, p. 1232.

Question 9: (see full question)

The liver has many jobs. One of the most important functions of the liver is to cleanse the portal blood of old and defective blood cells, bacteria in the bloodstream, and any foreign material. Which cells in the liver are capable of removing bacteria and foreign material from the portal blood?

You selected:

Kupffer cells

Correct

Question 10:

When caring for the client with hepatic failure, the nurse recognizes which of


(see full question)

these problems places the client at increased risk for bleeding?

You selected:

Increased prothrombin time

Correct

(see full question)

The physician suspects a client may have developed pancreatitis, and the physician has ordered laboratory blood work. Diagnosis-confirming results would identify:

You selected:

Altered alkaline phosphatase and red blood cell count

Question 1:

Incorrect Correct response:

High serum amylase and lipase

Explanation:

Laboratory criteria for the diagnosis of pancreatitis are serum amylase or lipase greater than three times the upper limit of normal. Altered alkaline phosphatase and prothrombin l ... (more)

Reference:

Grossman S, Porth CM (2014). Porth’s Pathophysiology: Concepts of Altered Health States. 9th ed. Philadelphia: Lippincott Williams & Wilkins, Chapter 46, Disorders of Hepatobiliary and Exocrine Pancreas Function, p. 1238.

Question 2: (see full question)

Ascites is an accumulation of fluid in the peritoneal cavity and usually occurs in advanced cirrhosis. What is the treatment of choice for ascites?

You selected:

Paracentesis

Incorrect Correct response:

Diuretics

Explanation:

Because of the many limitations in sodium restriction, the use of diuretics has become the mainstay of treatment for ascites. A paracentesis may be done if the diuretics do n ... (more)

Reference:

Grossman S, Porth CM (2014). Porth's Pathophysiology: Concepts of Altered Health States. 9th ed. Philadelphia: Lippincott Williams & Wilkins, Chapter 46: Disorders of Hepatobilary and Exocrine Pancreas Function, p. 1230.


Question 3: (see full question)

You selected:

A newborn is admitted to the hospital with a high bilirubin level of 13 mg/dL. The assessment data related to this lab value includes: Select all that apply. • Colicky, intermittent pain associated with formula feeding • Yellowish discoloration of the skin • Yellowing of the sclera of the eye

Incorrect Correct response:

• Yellowish discoloration of the skin • Yellowing of the sclera of the eye • Dark-colored urine

Explanation:

Jaundice (i.e., icterus) results from an abnormally high accumulation of bilirubin in the blood, as a result of which there is a yellowish discoloration to the skin and deep tissue ... (more)

Question 4: (see full question)

What laboratory markers are most commonly used to diagnose acute pancreatitis?

You selected:

Lipase and amylase

Correct Explanation:

Serum amylase and lipase are the laboratory markers most commonly used to establish a diagnosis of acute pancreatitis. Cholesterol and triglycerides are not used as laborator ... (more)

Reference:

Grossman S, Porth CM (2014). Porth's Pathophysiology: Concepts of Altered Health States. 9th ed. Philadelphia: Lippincott Williams & Wilkins, Chapter 46: Disorders of Hepatobilary and Exocrine Pancreas Function, p. 1238.

Question 5:

The nurse is assessing a client who has just been admitted to the unit with a diagnosis of cholelithiasis. The nurse is aware that the client may manifest:

(see full question)

You selected: Correct

Right upper quadrant pain


Explanation:

The pain is usually located in the upper right quadrant or epigastric area and may be referred to the upper back, right shoulder, or midscapular region. Typically, the pain is abru ... (more)

Reference:

Grossman S, Porth CM (2014). Porth’s Pathophysiology: Concepts of Altered Health States. 9th ed. Philadelphia: Lippincott Williams & Wilkins, Chapter 46, Disorders of Hepatobiliary and Exocrine Pancreas Function, p. 1236.

Question 6: (see full question)

All diseases have risk factors. What is the most significant environmental risk factor for pancreatic cancer?

You selected:

Cigarette smoking

Correct Explanation:

In pancreatic cancer, the most significant and reproducible environmental risk factor is cigarette smoking. The other answers are incorrect.

Question 7: (see full question)

The nurse is reviewing lab results of a client who has liver failure. The nurse determines that the client is at an increased risk for bleeding when the results include:

You selected:

Increased prothrombin time

Correct Explanation:

Clients with liver failure have malabsorption of vitamin K (decrease), which impairs the synthesis of clotting factors. An increased prothrombin time places the client at risk for ... (more)

Reference:

Grossman S, Porth CM (2014). Porth’s Pathophysiology: Concepts of Altered Health States. 9th ed. Philadelphia: Lippincott Williams & Wilkins, Chapter 46, , Disorders of Hepatobiliary and Exocrine Pancreas Function, p. 1231.

Question 8:

What should the nurse teach a client who is diagnosed with chronic hepatitis B infection about treatment?

(see full question)


You selected:

“Steroids are often used to control hepatitis B infection.”

Incorrect Correct response:

“The treatment is effective if your liver enzymes return to normal.”

Explanation:

Chronic hepatitis B infection is treated with diet to avoid stressing the liver (no alcohol; moderate fat intake) and medications. Drugs include interferons and the nucleotide and ... (more)

Reference:

Grossman S, Porth CM (2014). Porth’s Pathophysiology: Concepts of Altered Health States. 9th ed. Philadelphia: Lippincott Williams & Wilkins, Chapter 46, Disorders of Hepatobiliary and Exocrine Pancreas Function, p. 1224.

Question 9: (see full question)

When caring for the client with portal hypertension and ascites, which of these dietary interventions does the nurse suggest to prevent the progression of fluid accumulation?

You selected:

Limit intake of sodium.

Correct Explanation:

Treatment of ascites usually focuses on dietary restriction of sodium, administration of diuretics, and possible fluid restriction.

Question 1:

A client who has been diagnosed with acute symptomatic viral hepatitis is now in the icteric period. The nurse would expect the client to manifest:

(see full question)

You selected:

Severe pruritus and liver tenderness

Correct Explanation:

Severe pruritus and liver tenderness are common during the icterus period. Chills, fever, and severe anorexia occur during the prodromal period. The disappearance of jaundice occur ... (more)

Reference:

Grossman, S. & Porth, C.M. Porth's Pathophysiology: Concepts of Altered Health States, 9th ed., Philadelphia: Lippincott Williams & Wilkins, 2014, Chapter 46, Disorders of Hepatobiliary and Exocrine Pancreas Function, p. 1220.


(see full question)

The nurse is caring for a client with right upper quadrant pain secondary to acute choledocholithiasis. The nurse anticipates if the common bile duct becomes obstructed, manifestations will also include which of these?

You selected:

Hyperbilirubinemia

Question 2:

Correct Explanation:

Choledocholithiasis, stones in the common duct, usually originate in the gallbladder, but can form spontaneously in the common duct. Bilirubinuria and an elevated serum bilirubin ( ... (more)

Reference:

Grossman S, Porth CM (2014). Porth’s Pathophysiology: Concepts of Altered Health States. 9th ed. Philadelphia: Lippincott Williams & Wilkins, Chapter 46, Disorders of Hepatobilary and Exocrine Pancreas Function, p. 1237.

Question 3:

The community health nurse is offering education about hepatitis. Which of these groups does the nurse relate are at risk for hepatitis C? Select all that apply.

(see full question)

You selected:

• Individuals diagnosed with pancreatitis • Injection drug users

Incorrect Correct response:

• Sexually active homosexual men • Injection drug users

Explanation:

Recreational injection drug use is the most common mode of HCV transmission in the United States and Canada. High-risk sexual behavior, having sex with multiple partners or sex wit ... (more)

Question 4:

A client newly diagnosed with pancreatic cancer is admitted to begin treatment. Which pain discriptors can be associated with adenocarcinomas of the pancreas?

(see full question)

You selected:

Dull epigastric pain accompanied by back pain, worse when lying flat and relieved by sitting forward.

Correct Explanation:

The most common pain [with pancreatic cancer] is a dull epigastric pain often


accompanied by back pain, often worse in the supine position, and relieved by sitting forward. Sharp, ... (more)

Reference:

Grossman S, Porth CM (2014). Porth’s Pathophysiology: Concepts of Altered Health States. 9th ed. Philadelphia: Lippincott Williams & Wilkins, Chapter 46, Disorders of the Hepatobiliary and Exocrine Pancreas Function, p. 1239.

Question 5:

The nurse teaches the client that which of these contributed to his developing acute cholelithiasis?

(see full question)

You selected:

Stasis of bile

Correct Explanation:

Three factors contribute to the formation of gallstones: abnormalities in the composition of bile, stasis of bile (rather than rapid elimination), and inflammation of the gallbladd ... (more)

Reference:

Grossman S, Porth CM (2014). Porth’s Pathophysiology: Concepts of Altered Health States. 9th ed. Philadelphia: Lippincott Williams & Wilkins, Chapter 46, Disorders of Hepatobilary and Exocrine Pancreas Function, p. 1236.

Question 6: (see full question)

Many drugs are metabolized and detoxified in the liver. Most drug metabolizing occurs in the central zones of the liver. What condition is caused by these drug-metabolizing actions?

You selected:

Central cirrhosis

Incorrect Correct response:

Centrilobular necrosis

Explanation:

Because of the greater activity of the drug-metabolizing enzymes in the central zones of the liver, these agents typically cause centrilobular necrosis. The other answers are ... (more)

Question 7:

A client is undergoing diagnostic testing for possible liver failure. While educating a group of nursing students, the nurse asks them what the clinical manifestations of liver failure include. Which answers are accurate? Select all

(see full question)


that apply. You selected:

• Palpable masses located in liver region of the abdomin. • Dull epigastric pain accompanied by back pain. • Sweet, musty breath.

Incorrect Correct response:

• Sweet, musty breath. • Multiple bruising noted on body.

Explanation:

Fetor hepaticus refers to a characteristic musty, sweetish odor of the breath in the person in advanced liver failure, resulting from the metabolic by-products of the intestinal ba ... (more)

Reference:

Grossman S, Porth CM (2014). Porth’s Pathophysiology: Concepts of Altered Health States. 9th ed. Philadelphia: Lippincott Williams & Wilkins, Chapter 46, Disorders of Hepatobiliary and Exocrine Pancreas Function, p. 1231-1232.

Question 8: (see full question)

The nurse is caring for a client with hepatitis and jaundice. The nurse recognizes that without sufficient circulating bile salts the client will have intolerance to which of these ingested substances?

You selected:

Fats

Correct Explanation:

Lack of production of bile salts causes malabsorption of fat and fat-soluble vitamins.

Reference:

Grossman S, Porth CM (2014). Porth’s Pathophysiology: Concepts of Altered Health States. 9th ed. Philadelphia: Lippincott Williams & Wilkins, Chapter 46, Disorders of Hepatobiliary and Exocrine Pancreas Function, p. 1212.

Question 9:

Which of the following signs and symptoms is most suggestive of acute cholecystitis?

(see full question)

You selected:

Fever and sudden abdominal distention

Incorrect Correct response:

Upper right quadrant or epigastric pain

Explanation:

Persons with acute cholecystitis usually experience an acute onset of upper


right quadrant or epigastric pain. Nausea and vomiting are also common, although these are not specific ... (more)

(see full question)

A client is suspected of having the onset of alcoholic liver disease. The nurse should be assessing for which of the following manifestations related to the necrosis of liver cells?

You selected:

Rapid onset of jaundice

Question 10:

Correct Explanation:

Alcoholic hepatitis is the intermediate stage between fatty changes and cirrhosis and is characterized by inflammation and necrosis of liver cells. The condition is always serious ... (more)

Question 1:

When explaining acute pancreatitis to a newly diagnosed client, the nurse will emphasize the pathogenesis begins with an inflammatory process whereby:

(see full question)

You selected:

Activated pancreatic enzymes escape into surrounding tissues, causing autodigestion of pancreatic tissue.

Correct Explanation:

Acute pancreatitis is associated with the escape of activated pancreatic enzymes into the pancreas and surrounding tissues. These enzymes cause fat necrosis, or autodigestion, of t ... (more)

Reference:

Grossman, S. & Porth, C.M. Porth's Pathophysiology: Concepts of Altered Health States, 9th ed., Philadelphia: Lippincott Williams & Wilkins, 2014, Chapter 46, Disorders of Hepatobiliary and Exocrine Pancreas Function, p. 1238.

Question 2:

A client with a history of cancer that metastasized to the liver has arrived at the outpatient clinic to have a paracentesis performed. The physician anticipates that the client will have more than 5 L of fluid removed. The physician has prescribed intravenous albumin following the procedure. The client asks why she needs, "more fluids in my vein?" The nurse responds:

(see full question)

You selected:

"Albumin is a volume expander. Since a lot of fluid was removed, you have a decrease in your vascular volume, so without this albumin, your kidneys will try to reabsorb and hold onto water."


Correct Explanation:

Large-volume paracentesis (removal of 5 L or more of ascitic fluid) may be done in persons with massive ascites and pulmonary compromise. Because the removal of fluid produces a de ... (more)

Reference:

Grossman S, Porth CM (2014). Porth’s Pathophysiology: Concepts of Altered Health States. 9th ed. Philadelphia: Lippincott Williams & Wilkins, Chapter 46, Disorders of Hepatobiliary and Exocrine Pancreas Function, p. 1230.

Question 3:

A client has been admitted with hepatocellular jaundice as a result of taking medications. From the following list of medications in the client's home, the nurse knows which medications may place the client at high risk for developing hepatocellular jaundice? Select all that apply.

(see full question)

You selected:

• Non-steroidal anti-inflammatory drugs (NSAIDs) for joint pain • Anabolic steroids illegally taken for body building • Sulfamethoxazole-trimethoprim for urinary tract infection

Incorrect Correct response:

• Oral contraceptives for birth control• Anabolic steroids illegally taken for body building • Sulfamethoxazole-trimethoprim for urinary tract infection

Question 4:

What should the nurse teach a client who is diagnosed with chronic hepatitis B infection about treatment?

(see full question)

You selected:

“The treatment is effective if your liver enzymes return to normal.”

Correct Explanation:

Chronic hepatitis B infection is treated with diet to avoid stressing the liver (no alcohol; moderate fat intake) and medications. Drugs include interferons and the nucleotide and ... (more)

Reference:

Grossman S, Porth CM (2014). Porth’s Pathophysiology: Concepts of Altered Health States. 9th ed. Philadelphia: Lippincott Williams & Wilkins, Chapter 46, Disorders of Hepatobiliary and Exocrine Pancreas Function, p. 1224.


Question 5: (see full question)

You selected:

When assessing the client during the icteric phase of viral hepatitis, which of these findings does the nurse anticipate observing? Yellow-tinged sclera

Correct Explanation:

The icterus phase is reflected by development of jaundice of skin and sclera, liver tenderness and worsened prodromal symptoms.

Reference:

Grossman S, Porth CM (2014). Porth’s Pathophysiology: Concepts of Altered Health States. 9th ed. Philadelphia: Lippincott Williams & Wilkins, Chapter 46, Disorders of Hepatobiliary and Exocrine Pancreas Function, p. 1220.

Question 6: (see full question)

Ascites is an accumulation of fluid in the peritoneal cavity and usually occurs in advanced cirrhosis. What is the treatment of choice for ascites?

You selected:

Diuretics

Correct Explanation:

Because of the many limitations in sodium restriction, the use of diuretics has become the mainstay of treatment for ascites. A paracentesis may be done if the diuretics do n ... (more)

Question 7: (see full question)

The health care provider has completed the assessment of a client who presented to the emergency department with jaundice. The provider determines that the jaundice is a result of:

You selected:

Impaired uptake of bilirubin by the liver

Correct Explanation:

The four major causes of jaundice are excessive destruction of RBCs, impaired uptake of bilirubin by the liver cells, decreased conjugation of bilirubin, and obstruction of the bow ... (more)

Reference:

Grossman S, Porth CM (2014). Porth’s Pathophysiology: Concepts of Altered Health States. 9th ed. Philadelphia: Lippincott Williams & Wilkins, Chapter 46, Disorders of Hepatobiliary and Exocrine Pancreas Function, p. 1216.


(see full question)

The physician suspects a client may have developed pancreatitis, and the physician has ordered laboratory blood work. Diagnosis-confirming results would identify:

You selected:

High serum amylase and lipase

Question 8:

Correct Explanation:

Laboratory criteria for the diagnosis of pancreatitis are serum amylase or lipase greater than three times the upper limit of normal. Altered alkaline phosphatase and prothrombin l ... (more)

Reference:

Grossman S, Porth CM (2014). Porth’s Pathophysiology: Concepts of Altered Health States. 9th ed. Philadelphia: Lippincott Williams & Wilkins, Chapter 46, Disorders of Hepatobiliary and Exocrine Pancreas Function, p. 1238.

Question 9:

When assessing the client with acute pancreatitis, which of these diagnostic tests, consistent with the disease, does the nurse anticipate will be altered?

(see full question)

You selected:

Amylase and lipase

Correct Explanation:

Serum amylase and lipase are the laboratory markers most commonly used to establish a diagnosis of acute pancreatitis

Question 10:

A major factor in the development of hepatic encephalopathy is:

(see full question)

You selected:

Neurotoxin accumulation

Correct Explanation:

Although the cause of hepatic encephalopathy is unknown, the accumulation of neurotoxins, which appear in the blood because the liver has lost its detoxifying capacity, is believed ... (more)

Question 1:

The nurse is caring for the client with pancreatic cancer. The nurse monitors the client for which of these complications?

(see full question)


You selected:

Hyperglycemia due to inability to synthesize insulin

Correct Explanation:

The endocrine pancreas supplies the insulin needed to lower glucose levels in the blood; damage to the pancreas may alter this function causing hyperglycemia.

Reference:

Grossman S, Porth CM (2014). Porth’s Pathophysiology: Concepts of Altered Health States. 9th ed. Philadelphia: Lippincott Williams & Wilkins, Chapter 46, Disorders of Hepatobiliary and Exocrine Pancreas Function, p. 1209.

Question 2:

A client with multiple pain-related injuries to the back, knees, and hips is admitted with acute liver failure. Upon procuring a medication list, the nurse notes that the client is taking several over-the-counter medications that contain a preparation known to be the drug that most commonly causes liver failure. Which drug is in the client's medications?

(see full question)

You selected:

Acetaminophen

Correct Explanation:

The drug most commonly involved is acetaminophen, with half the cases reported to be unintentional overdoses. Unintentional overdoses may occur when people unknowingly take several ... (more)

Reference:

Grossman S, Porth CM (2014). Porth’s Pathophysiology: Concepts of Altered Health States. 9th ed. Philadelphia: Lippincott Williams & Wilkins, Chapter 46, Disorders of Hepatobiliary and Exocrine Pancreas Function, p. 1219.

Question 3: (see full question)

A client tells the nurse that he is concerned about developing hepatitis after being exposed to contaminated feces, saliva, and food. The nurse is aware that the client is at risk for:

You selected:

Hepatitis A

Correct Explanation:

Hepatitis A is normally transmitted through the fecal–oral route by drinking contaminated milk or water and eating shellfish from infected waters. Hepatitis B is transmitted thro ... (more)


Reference:

Grossman S, Porth CM (2014). Porth’s Pathophysiology: Concepts of Altered Health States. 9th ed. Philadelphia: Lippincott Williams & Wilkins, Chapter 46, Disorders of Hepatobiliary and Exocrine Pancreas Function, p. 1221.

Question 4:

The nurse is assessing a client exposed to viral hepatitis who is thought to be in the prodromal phase. When assessing the client, which of these symptoms does the nurse recognize are consistent with this phase? Select all that apply.

(see full question)

You selected:

• Lack of energy • Myalgia • Lack of appetite

Correct

Question 5: (see full question)

You selected:

When caring for the client with Laennec’s cirrhosis, the nurse recognizes which of these is an expected etiology of jaundice? Increased conjugation of billirubin

Incorrect Correct response:

Impaired uptake of billirubin

Explanation:

The four major causes of jaundice are excessive destruction of red blood cells, impaired uptake of bilirubin by the liver cells, decreased conjugation of bilirubin and obstruction ... (more)

Reference:

Grossman S, Porth CM (2014). Porth’s Pathophysiology: Concepts of Altered Health States. 9th ed. Philadelphia: Lippincott Williams & Wilkins, Chapter 46, Disorders of Hepatobiliary and Exocrine Pancreas Function, p. 1216.

Question 6:

A client is admitted to a nursing unit with severe edema. The nursing student caring for this client overhears the physician and a medical student talking about the client's albumin level. When discussing the flow of fluids into and out of cells, albumin plays a significant role in which pressure gradient?

(see full question)


You selected:

Colloidal osmotic

Correct Explanation:

One of the most important of these secretory proteins is albumin. Albumin contributes significantly to the plasma colloidal osmotic pressure and to the binding and transport of num ... (more)

Reference:

Grossman S, Porth CM (2014). Porth’s Pathophysiology: Concepts of Altered Health States. 9th ed. Philadelphia: Lippincott Williams & Wilkins, Chapter 39, Disorders of Fluid and Electrolyte Balance, p.1025.

Question 7:

When caring for the client with hepatic failure, the nurse recognizes which of these problems places the client at increased risk for bleeding?

(see full question)

You selected:

Increased prothrombin time

Correct Explanation:

An increased prothrombin time would increase the risk for bleeding. Other factors that contribute to increased bleeding risk in patients with liver failure is malabsorption of vita ... (more)

Reference:

Grossman S, Porth CM (2014). Porth’s Pathophysiology: Concepts of Altered Health States. 9th ed. Philadelphia: Lippincott Williams & Wilkins, Chapter 46, Disorders of Hepatobiliary and Exocrine Pancreas Function, p. 1231.

Question 8: (see full question)

A client has begun to display manifestations of hepatic encephalopathy. The family is concerned and asks the nurse what caused this condition to develop. Which is the best response by the nurse?

You selected:

Accumulation of ammonia in the blood

Correct

Question 9: (see full question)

The client has right upper quadrant pain caused by acute choledocholithiasis. The health care provider suspects the common bile duct is obstructed, based


on which of the flowing lab values? You selected:

Bilirubin 15 mg/dL (high)

Correct Explanation:

Choledocholithiasis, stones in the common duct, usually originate in the gallbladder but can form spontaneously in the common duct. Bilirubinuria and an elevated serum bilirubin ar ... (more)

Reference:

Grossman, S. & Porth, C.M. Porth's Pathophysiology: Concepts of Altered Health States, 9th ed., Philadelphia: Lippincott Williams & Wilkins, 2014, Chapter 46, Disorders of Hepatobiliary and Exocrine Pancreas Function, p. 1237.

Question 10:

When teaching a client at the GI clinic about chronic pancreatitis, which of these does the nurse relate is the long term result of this condition?

(see full question)

You selected:

Progressive destruction of the organ

Correct Explanation:

The chief distinction between acute and chronic pancreatitis is the irreversibility of pancreatic function with chronic pancreatitis. While the most common cause of chronic pancrea ... (more)

Question 1:

A client is admitted with late manifestations of cirrhosis. Which of the following clinical manifestations confirm this diagnosis? Select all that apply.

(see full question)

You selected:

• Hepatomegaly felt on deep palpation • Diffuse liver fibrosis with large, palpable lumps

Incorrect Correct response:

• Hepatomegaly felt on deep palpation • GI bleeding related to esophageal varices • Splenomegaly with bleeding tendencies

Explanation:

The manifestations of cirrhosis are variable, ranging from asymptomatic hepatomegaly to hepatic failure. Often there are no symptoms until the disease is far advanced. The late man ... (more)


Reference:

Grossman S, Porth CM (2014). Porth’s Pathophysiology: Concepts of Altered Health States. 9th ed. Philadelphia: Lippincott Williams & Wilkins, Chapter 46, Disorders of Hepatobilary and Exocrine Pancreas Function, p. 1229.

Question 2:

The client in the GI clinic tells the nurse she is concerned she has something wrong with her gallbladder like what her friend with gallstones and inflammation had. Which of these complaints does the nurse recognize that supports the client's concern?

(see full question)

You selected:

Intolerance to greasy food; burping

Correct Explanation:

The manifestations of chronic cholecystitis are more vague than those of acute cholecystitis. There may be intolerance to fatty foods, belching, and other indications of discomfort ... (more)

Reference:

Grossman S, Porth CM (2014). Porth’s Pathophysiology: Concepts of Altered Health States. 9th ed. Philadelphia: Lippincott Williams & Wilkins, Chapter 46, Disorders of Hepatobiliary and Exocrine Pancreas Function, p. 1237.

Question 3:

Which of the following signs and symptoms is most suggestive of acute cholecystitis?

(see full question)

You selected:

Upper right quadrant or epigastric pain

Correct Explanation:

Persons with acute cholecystitis usually experience an acute onset of upper right quadrant or epigastric pain. Nausea and vomiting are also common, although these are not specific ... (more)

Question 4:

A client presents to the emergency department vomiting large amounts of bright red blood. The client has a history of alcohol abuse, and the physician suspects esophageal varices. The drug that is used to reduce splanchnic and hepatic blood flow and portal pressures is:

(see full question)

You selected:

Lisinopril, an ACE inhibitor


Incorrect Correct response:

Ocetrotide, a long-acting synthetic analog of somatostatin

Explanation:

Several methods are used to control acute hemorrhage, including pharmacologic therapy, balloon tamponade, and emergent endoscopic therapy. Pharmacologic methods include administrat ... (more)

Reference:

Grossman, S. & Porth, C.M. Porth's Pathophysiology: Concepts of Altered Health States, 9th ed., Philadelphia: Lippincott Williams & Wilkins, 2014, Chapter 46, Disorders of Hepatobiliary and Exocrine Pancreas Function, p. 1230.

Question 5: (see full question)

The nurse is reviewing lab results of a client who has liver failure. The nurse determines that the client is at an increased risk for bleeding when the results include:

You selected:

Increased prothrombin time

Correct Explanation:

Clients with liver failure have malabsorption of vitamin K (decrease), which impairs the synthesis of clotting factors. An increased prothrombin time places the client at risk for ... (more)

Reference:

Grossman S, Porth CM (2014). Porth’s Pathophysiology: Concepts of Altered Health States. 9th ed. Philadelphia: Lippincott Williams & Wilkins, Chapter 46, , Disorders of Hepatobiliary and Exocrine Pancreas Function, p. 1231.

Question 6: (see full question)

When assessing the nutritional intake of a client with generalized pancreatic cancer, which of these statements by the client is consistent with the disease and food intake?

You selected:

Client reports pain is worse two hours after eating.

Incorrect Correct response:

Client states pain becomes worse with food intake.

Explanation:

Cancer of the body of the pancreas usually impinges on the celiac ganglion, causing pain. The pain usually worsens with ingestion of food or assumption


of the supine position.

Question 7: (see full question)

You selected:

When assessing a client with acute cholecystitis, the nurse anticipates the client's report of pain will be consistent with which of these descriptions? Pain in the right upper quadrant referred to the same shoulder

Correct Explanation:

The pain of biliary colic begins abruptly and increases in intensity. It is usually located in the upper right quadrant or epigastric area and may be referred to the upper back, th ... (more)

Reference:

Grossman S, Porth CM (2014). Porth’s Pathophysiology: Concepts of Altered Health States. 9th ed. Philadelphia: Lippincott Williams & Wilkins, Chapter 46, Disorders of Hepatobiliary and Exocrine Pancreas Function, p. 1236.

Question 8:

When caring for the client with pancreatic cancer, for which of these signs and symptoms does the nurse assess?

(see full question)

You selected:

Fever, chills and weight loss

Incorrect Correct response:

Central abdominal pain, jaundice and weight loss

Explanation:

The clinical manifestations of pancreatic cancer depend on the size and location of the tumor as well as its metastasis. Pain, jaundice, and weight loss constitute the classic pres ... (more)

Reference:

Grossman S, Porth CM (2014). Porth’s Pathophysiology: Concepts of Altered Health States. 9th ed. Philadelphia: Lippincott Williams & Wilkins, Chapter 46, Disorders of Hepatobiliary and Exocrine Pancreas Function, p. 1239.

Question 9:

A client is prescribed erythromycin for an infection. What manifestations will the nurse recognize that indicate the onset of drug-induced cholestasis?

(see full question)

You selected: Incorrect

Ascites and immunosuppression


Correct response:

Jaundice and pruritus

Explanation:

Jaundice and itching are the early manifestations of drug-induced cholestasis. Clients generally report no change in feelings of well-being. The symptoms subside when the drug is w ... (more)

Question 10: (see full question)

When teaching a group of nursing students about the liver, the nurse relates that Kupffer cells function to remove harmful substances or cells from the portal blood and venous sinusoids through which of these processes?

You selected:

Phagocytosis

Correct Explanation:

Kupffer cells, which line the venous sinusoids are reticuloendothelial cells that are capable of removing, engulfing, and phagocytizing old and defective blood cells, bacteria, and ... (more)

Question 1: (see full question)

Given the fact that acute pancreatitis can result in severe, life-threatening complications, the nurse should be assessing the client for which of the following complications?

You selected:

Acute tubular necrosis

Correct Explanation:

Complications of acute pancreatitis include the systemic inflammatory response, acute respiratory distress syndrome, acute tubular necrosis, and organ failure. Cerebral hemorrhage, ... (more)

Reference:

Grossman, S. & Porth, C.M. Porth's Pathophysiology: Concepts of Altered Health States, 9th ed., Philadelphia: Lippincott Williams & Wilkins, 2014, Chapter 46, Disorders of Hepatobiliary and Exocrine Pancreas Function, p. 1238-1239.

Question 2:

The nurse is assessing a client with acute pancreatitis who denies use of alcohol. When teaching the client about additional causes of acute pancreatitis, the nurse includes which of these in the discussion? Select all that apply.

(see full question)

You selected:

• Abdominal trauma • Increased cholesterol


Incorrect Correct response:

• Gallstones • Increased cholesterol • Abdominal trauma

Explanation:

Acute pancreatitis is associated with alcohol abuse, gallstones, hyperlipidemia, hypercalcemia, infections (particularly viral), abdominal and surgical trauma, and drugs such as t ... (more)

Reference:

Grossman S, Porth CM (2014). Porth’s Pathophysiology: Concepts of Altered Health States. 9th ed. Philadelphia: Lippincott Williams & Wilkins, Chapter 46, Disorders of Hepatobiliary and Exocrine Pancreas Function, p. 1238.

Question 3:

Which of these substances should the nurse teach the client with pancreatitis is essential to avoid?

(see full question)

You selected:

Fatty foods

Incorrect Correct response:

Alcohol

Explanation:

Clients with chronic pancreatitis must be told alcohol is forbidden as it frequently precipitates attacks.

Question 4: (see full question)

The nurse is caring for a client with liver disease who has edema throughout the body. When reviewing the medical record, the nurse recognizes which of these altered diagnostic tests is consistent with development of edema?

You selected:

Decreased albumin

Correct Explanation:

Altered function of the liver causes decreased levels of plasma proteins, particularly albumin, which contributes to edema formation.

Reference:

Grossman S, Porth CM (2014). Porth’s Pathophysiology: Concepts of Altered Health States. 9th ed. Philadelphia: Lippincott Williams & Wilkins, Chapter 46, Disorders of Hepatobiliary and Exocrine Pancreas Function, p. 1212.


Question 5: (see full question)

You selected:

A client who has been diagnosed with acute symptomatic viral hepatitis is now in the icteric period. The nurse would expect the client to manifest: Severe pruritus and liver tenderness

Correct Explanation:

Severe pruritus and liver tenderness are common during the icterus period. Chills, fever, and severe anorexia occur during the prodromal period. The disappearance of jaundice occur ... (more)

Reference:

Grossman, S. & Porth, C.M. Porth's Pathophysiology: Concepts of Altered Health States, 9th ed., Philadelphia: Lippincott Williams & Wilkins, 2014, Chapter 46, Disorders of Hepatobiliary and Exocrine Pancreas Function, p. 1220.

Question 6: (see full question)

The nurse is caring for a client with right upper quadrant pain secondary to acute choledocholithiasis. The nurse anticipates if the common bile duct becomes obstructed, manifestations will also include which of these?

You selected:

Hyperbilirubinemia

Correct Explanation:

Choledocholithiasis, stones in the common duct, usually originate in the gallbladder, but can form spontaneously in the common duct. Bilirubinuria and an elevated serum bilirubin ( ... (more)

Question 7: (see full question)

A client's long-standing diagnosis of congenital hemolytic anemia often manifests itself with jaundice. What type of jaundice does this client most likely experience?

You selected:

Intrahepatic

Incorrect Correct response:

Prehepatic

Explanation:

The major cause of prehepatic jaundice is excessive hemolysis of red blood cells. Hemolytic jaundice occurs when red blood cells are destroyed at a rate in excess of the liver's ab ... (more)


Reference:

1Grossman S, Porth CM (2014). Porth’s Pathophysiology: Concepts of Altered Health States. 9th ed. Philadelphia: Lippincott Williams & Wilkins, Chapter 46, , Disorders of Hepatobiliary and Exocrine Pancreas Function, p. 1216.

Question 8: (see full question)

The obstetric nurse is caring for a woman in the delivery room who has tested positive for the HBsAg. Which of these interventions does the nurse recognize is indicated?

You selected:

Vaccination of the newborn with immunoglobulin

Correct Explanation:

In individuals with hepatitis B, the HBsAg is the viral antigen measured most routinely in blood. It appears before onset of symptoms, peaks during overt disease, and then declines ... (more)

Reference:

Grossman S, Porth CM (2014). Porth’s Pathophysiology: Concepts of Altered Health States. 9th ed. Philadelphia: Lippincott Williams & Wilkins, Chapter 46, Disorders of Hepatobiliary and Exocrine Pancreas Function, p. 1223.

Question 9:

When caring for the client with Laennec’s cirrhosis, the nurse recognizes which of these is an expected etiology of jaundice?

(see full question)

You selected:

Impaired uptake of billirubin

Correct Explanation:

The four major causes of jaundice are excessive destruction of red blood cells, impaired uptake of bilirubin by the liver cells, decreased conjugation of bilirubin and obstruction ... (more)

Question 10: (see full question)

A client who has been taking acetaminophen 1000 mg every 4 hours presents to the Urgent Care Center with increased abdominal pain, elevated ALT, AST, and bilirubin levels. The nurse suspects the client is experiencing:

You selected:

Direct hepatotoxic reaction

Correct


Explanation:

Direct hepatotoxic reactions usually are a recognized characteristic of certain drugs. They usually result from drug metabolism and the generation of toxic metabolites from drugs l ... (more)

Question 1: (see full question)

Antibody testing has confirmed that a man is positive for hepatitis A virus (HAV). Which of the client's statements suggests that he understands his new diagnosis?

You selected:

"I don't know why I didn't bother to get vaccinated against this."

Correct Explanation:

A vaccine is available for HAV. The disease is normally self-limiting and does not result in carrier status. Transmission is usually by the fecal–oral route, rather than sexual t ... (more)

Reference:

Grossman S, Porth CM (2014). Porth’s Pathophysiology: Concepts of Altered Health States. 9th ed. Philadelphia: Lippincott Williams & Wilkins, Chapter 46, Disorders of Hepatobilary and Exocrine Pancreas Function, p. 1221.

Question 2:

A client has been diagnosed with cholelithiasis. The nurse is aware the client is at risk for: Select all that apply.

(see full question)

You selected:

• Obstruction of bile flow • Acute cholecystitis • Biliary colic

Incorrect Correct response:

• Obstruction of bile flow • Biliary colic • Acute cholecystitis • Chronic cholecystitis

Explanation:

Cholelithiasis predisposes to obstruction of bile flow, causing biliary colic, and acute or chronic cholecystitis. Absence of vitamin B12 results from lack of gastric intrinsic fac ... (more)

Reference:

Grossman, S. & Porth, C.M. Porth's Pathophysiology: Concepts of Altered Health States, 9th ed., Philadelphia: Lippincott Williams & Wilkins, 2014, Chapter 46, Disorders of Hepatobiliary and Exocrine Pancreas Function, p. 1240.


(see full question)

The nurse is caring for a college student with infectious mononucleosis. For which of these reasons does the nurse teach the client to abstain from alcohol during the acute phase of the disease?

You selected:

Alcohol may worsen a mild hepatitis which occurs in the acute phase.

Question 3:

Correct Explanation:

Viruses causing systemic disease that can involve the liver include EpsteinBarr virus (infectious mononucleosis), which may cause a mild hepatitis during the acute phase.

Question 4: (see full question)

The nurse is caring for a client who is concerned he may have contracted hepatitis C from a sexual partner. Which of these is it correct for the nurse to teach the client about this disease?

You selected:

Antibodies may be detected in the blood in 6 to 8 weeks.

Correct Explanation:

The incubation period for HCV infection ranges from 2 to 26 weeks (average, 6 to 12 weeks). With newer antibody testing methods, infection often can be detected as early as 6 to 8 ... (more)

Reference:

Grossman S, Porth CM (2014). Porth’s Pathophysiology: Concepts of Altered Health States. 9th ed. Philadelphia: Lippincott Williams & Wilkins, Chapter 46, Disorders of Hepatobiliary and Exocrine Pancreas Function, p. 1223.

Question 5:

When teaching the client about the cause of acute pancreatitis, which of these points should be included in the teaching session?

(see full question)

You selected:

An infection of the pancreas has occurred.

Incorrect Correct response:

Pancreatic enzymes are activated before leaving the pancreas causing injury.

Explanation:

The pathogenesis of acute pancreatitis involves the autodigestion of pancreatic tissue by prematurely activated pancreatic enzymes. Trypsin in activated, then can activate a variet ... (more)


Reference:

Grossman S, Porth CM (2014). Porth’s Pathophysiology: Concepts of Altered Health States. 9th ed. Philadelphia: Lippincott Williams & Wilkins, Chapter 46, Disorders of Hepatobiliary and Exocrine Pancreas Function, p. 1238.

Question 6:

When caring for the client with pancreatic cancer, for which of these signs and symptoms does the nurse assess?

(see full question)

You selected:

Central abdominal pain, jaundice and weight loss

Correct Explanation:

The clinical manifestations of pancreatic cancer depend on the size and location of the tumor as well as its metastasis. Pain, jaundice, and weight loss constitute the classic pres ... (more)

Question 7: (see full question)

The nurse is caring for a client with hepatitis and jaundice. The nurse recognizes that without sufficient circulating bile salts the client will have intolerance to which of these ingested substances?

You selected:

Fats

Correct Explanation:

Lack of production of bile salts causes malabsorption of fat and fat-soluble vitamins.

Reference:

Grossman S, Porth CM (2014). Porth’s Pathophysiology: Concepts of Altered Health States. 9th ed. Philadelphia: Lippincott Williams & Wilkins, Chapter 46, Disorders of Hepatobiliary and Exocrine Pancreas Function, p. 1212.

Question 8: (see full question)

The nurse is assessing a client with a history of long-term alcohol use for signs and symptoms of early cirrhosis. Which of these questions by the nurse will help elicit these early symptoms?

You selected:

"Do you find your appetite is poor lately?"

Correct Explanation:

Often there are no symptoms until the disease is far advanced. The most common signs and symptoms of cirrhosis are weight loss (sometimes masked by ascites), weakness, and anorexia ... (more)

Reference:

Grossman S, Porth CM (2014). Porth’s Pathophysiology: Concepts of Altered


Health States. 9th ed. Philadelphia: Lippincott Williams & Wilkins, Chapter 46, Disorders of Hepatobiliary and Exocrine Pancreas Function, p. 1228.

Question 9: (see full question)

You selected:

A client is prescribed erythromycin for an infection. What manifestations will the nurse recognize that indicate the onset of drug-induced cholestasis? Jaundice and pruritus

Correct Explanation:

Jaundice and itching are the early manifestations of drug-induced cholestasis. Clients generally report no change in feelings of well-being. The symptoms subside when the drug is w ... (more)

Question 10:

When assessing the client with acute pancreatitis, which of these diagnostic tests, consistent with the disease, does the nurse anticipate will be altered?

(see full question)

You selected:

Amylase and lipase

Correct Explanation:

Serum amylase and lipase are the laboratory markers most commonly used to establish a diagnosis of acute pancreatitis.

Question 1:

The nurse is providing nutritional support education to the family of an obese 10-year-old. The most important information to provide would be:

(see full question)

You selected:

The weight loss intervention should include all family members.

Correct Explanation:

The goal should be healthy eating and activity, not achievement of ideal body weight. The child’s activities should be more active than watching television or using the computer ..... (more)

Reference:

Grossman S, Porth CM (2014). Porth’s Pathophysiology: Concepts of Altered Health States. 9th ed. Philadelphia: Lippincott Williams & Wilkins, Ch. 47, Alterations in Nutritional Status, pp. 1252-1253.

Question 2:

In the phases of metabolism, which of the following requires energy rather than providing it for use by the body?

(see full question)


You selected:

Catabolism

Incorrect Correct response:

Anabolism

Explanation:

Anabolism uses energy rather than producing it. Catabolism, digestion, and breakdown of complex molecules produce energy for the body to use.

Reference:

Grossman S, Porth CM (2014). Porth’s Pathophysiology: Concepts of Altered Health States. 9th ed. Philadelphia: Lippincott Williams & Wilkins, Ch. 47, Alterations in Nutritional Status, p. 1245.

Question 3:

A child has been diagnosed with marasmus. Which clinical manifestations should the school nurse assess to confirm the diagnosis? Select all that apply.

(see full question)

You selected:

• Discolored hair • Bradycardia • Stunted growth pattern

Incorrect Correct response:

• Bradycardia

Question 4: (see full question)

The nurse is assessing a patient who is in the clinic for a routine physical. The patient is female and has upper body obesity. Which of the following would the nurse describe as being the body type of this patient?

You selected:

Apple shaped

Correct Explanation:

Patients with upper body obesity are often referred to as being shaped like an "apple" compared with lower body obesity, which is more "pear" shaped.

Reference:

Grossman S, Porth CM (2014). Porth’s Pathophysiology: Concepts of Altered Health States. 9th ed. Philadelphia: Lippincott Williams & Wilkins, Chapter 47, Alteration in Nutritional Status, p. 1253.


Question 5: (see full question)

You selected:

A client developed malnutrition while hospitalized for an illness. The client does not understand how that could have happened. The best response would be: “Malnutrition is also common during illness, recovery from trauma, and hospitalization.”

Correct Explanation:

Protein–energy malnutrition most often occurs secondary to trauma or illness. Kwashiorkor-like secondary protein–energy malnutrition occurs most commonly in associati ... (more)

Reference:

Grossman, S. & Porth, C.M. Porth's Pathophysiology: Concepts of Altered Health States, 9th ed., Philadelphia: Lippincott Williams & Wilkins, 2014, Chapter 47, Alterations in Nutritional Status, p. 1257.

Question 6:

The health care provider reviews the waist–hip ratio of a male. The ratio is 1.0. The provider would interpret his result as:

(see full question)

You selected:

Upper body obesity

Correct Explanation:

A waist–hip ratio greater than 1.0 in men and 0.8 in women indicates upper body obesity. Research suggests that fat distribution may be a more important factor for morbidity and ... (more)

Question 7:

A nurse assessing a female patient obtains a waist circumference of 92 cm from the patient. Which of the following does this finding indicate to the nurse?

(see full question)

You selected:

The patient may have increased health risks associated with this finding.

Correct Explanation:

A waist circumference of 88 cm or greater in women and 102 cm or greater in men has been associated with increased health risk.

Reference:

Grossman S, Porth CM (2014). Porth’s Pathophysiology: Concepts of Altered Health States. 9th ed. Philadelphia: Lippincott Williams & Wilkins, Chapter 47, Alteration in Nutritional Status, p. 1253.


Question 8: (see full question)

You selected:

A patient is having difficulty with a weight loss plan and asks the nurse, "What is wrong with me? Why can't I lose this weight?" Which of the following rationales might the nurse offer to this patient? "Obesity ultimately results from a metabolic disorder causing a decrease in the metabolic rate."

Incorrect Correct response:

"Obesity ultimately results from an energy imbalance of eating too many calories and not getting enough exercise."

Explanation:

The epidemic of obesity results from many causative factors, and research is adding to the body of knowledge almost daily. Although overweight and obesity ultimately result from an ... (more)

Reference:

Grossman S, Porth CM (2014). Porth’s Pathophysiology: Concepts of Altered Health States. 9th ed. Philadelphia: Lippincott Williams & Wilkins, Chapter 47, Alteration in Nutritional Status, p. 1252.

Question 9: (see full question)

A nurse is caring for a patient who has been admitted to an inpatient facility for the treatment of anorexia nervosa. When planning the care of this patient, which of the following personality traits does the nurse anticipate observing?

You selected:

Introverted and shy

Incorrect Correct response:

Perfectionistic and compulsive

Explanation:

The causes of anorexia appear to be multifactorial, with determinants that include genetic influence; personality traits of perfectionism and compulsiveness; anxiety disorders; fam ... (more)

Question 1: (see full question)

A nurse is determining the type of obesity in a patient prior to initiating a weight loss program. Which of the following results would indicate that the patient has central obesity?

You selected:

Waist-hip ratio greater than 1.0

Incorrect


Correct response:

Waist-hip ratio greater than 0.8

Explanation:

Obesity type is determined by dividing the waist by the circumference of the hip. Comparison of the waist measurement and hip measurement can identify the type of obesity. A waist ... (more)

Reference:

Grossman S, Porth CM (2014). Porth’s Pathophysiology: Concepts of Altered Health States. 9th ed. Philadelphia: Lippincott Williams & Wilkins, Chapter 47, Alteration in Nutritional Status, p. 1253.

Question 2:

The nurse is teaching a basic nutrition course for the community. Which statement should the nurse include about fat-soluble vitamins?

(see full question)

You selected:

Excess amounts of these vitamins are excreted in the urine.

Incorrect Correct response:

The fat-soluble vitamins are stored in adipose tissue and may reach toxic levels.

Explanation:

The four fat-soluble vitamins are vitamins A, D, E, and K. They require dietary fat for absorption and transport. The nine water-soluble vitamins are vitamin C and the B vitamins, ... (more)

Reference:

Grossman S, Porth CM (2014). Porth’s Pathophysiology: Concepts of Altered Health States. 9th ed. Philadelphia: Lippincott Williams & Wilkins, Chapter 47, Alterations in Nutritional Status, p. 1249.

Question 3: (see full question)

The nurse is providing dietary instruction to a client whose lab values indicate a high level of blood cholesterol. The client asks if there are any food contents that need to be avoided. The best response would be:

You selected:

Saturated fatty acids

Correct Explanation:

The saturated fatty acids elevate blood cholesterol, whereas the monounsaturated and polyunsaturated fats lower blood cholesterol.


Reference:

Grossman S, Porth CM (2014). Porth’s Pathophysiology: Concepts of Altered Health States. 9th ed. Philadelphia: Lippincott Williams & Wilkins, Ch. 47, Alterations in Nutritional Status, p. 1248.

Question 4: (see full question)

The nurse is providing education to a client who has been instructed to increase the amount of protein in her diet. Which foods should the nurse recommend?

You selected:

Milk and eggs

Correct

Question 5: (see full question)

You selected:

A nurse is caring for a group of patients. Which of the following patients has the greatest caloric needs? Breast-feeding mother who delivered 1 month ago

Correct Explanation:

Energy requirements are greater during growth periods. A person requires approximately 115 kcal/kg of body weight at birth, 105 kcal/kg at 1 year of age, and 80 kcal/kg from 1 to 10 y ... (more)

Reference:

Grossman S, Porth CM (2014). Porth’s Pathophysiology: Concepts of Altered Health States. 9th ed. Philadelphia: Lippincott Williams & Wilkins, Chapter 47, Alterations in Nutritional Status, p. 1248.

Question 6:

The nurse in the obstetrics clinic is advising a pregnant woman about nutritional needs during pregnancy. Which of these should the nurse include in the teaching plan?

(see full question)

You selected:

A pregnant woman needs an extra 300 kcal/day above the usual caloric intake.

Correct Explanation:

During pregnancy, a woman needs an extra 300 kcal/day above her usual requirement. Saturated fats usually are from animal sources and remain solid at room temperature, with the exce ... (more)


Reference:

Grossman S, Porth CM (2014). Porth’s Pathophysiology: Concepts of Altered Health States. 9th ed. Philadelphia: Lippincott Williams & Wilkins, Chapter 47, Alterations in Nutritional Status, pp. 1248-1249.

Question 7:

A patient informs the nurse that he or she feels as though he or she have a kidney stone again. The patient is on a diet for weight loss and feels as though this may be a contributing factor as it never occurred before going on the diet. Which of the following diets does the nurse suspect the patient may be on?

(see full question)

You selected:

Low-carbohydrate/high protein

Correct Explanation:

Carbohydrate diets are effective for weight loss, especially in the initial stages but can contribute to health risks. Higher protein diets can increase the risk of kidney stones, ... (more)

Reference:

Grossman S, Porth CM (2014). Porth’s Pathophysiology: Concepts of Altered Health States. 9th ed. Philadelphia: Lippincott Williams & Wilkins, Chapter 47, Alteration in Nutritional Status, p. 1255.

Question 8:

A client is undergoing a bioelectrical impedance test to estimate body fat. The nurse will explain to the client that this test involves:

(see full question)

You selected:

Using electrodes to send harmless current through the body to measure resistance to estimate body fat

Correct Explanation:

Bioelectrical impedance involves the use of electrodes attached to the wrists a

Question 9:

Following yearly routine physical examination by the health care provider, a client has been diagnosed with upper body obesity along with central fat distribution. The client is at greater risk for developing which of the following disease processes?

(see full question)

You selected:

Cardiometabolic disorders

Correct Explanation:

Upper body obesity, more than other types of obesity, carries a high


cardiometabolic risk. Obese people tend to develop joint problems and arthritis, but there is no direct associa ... (more)

Reference:

Grossman S, Porth CM (2014). Porth’s Pathophysiology: Concepts of Altered Health States. 9th ed. Philadelphia: Lippincott Williams & Wilkins, Ch. 47, Alterations in Nutritional Status, p. 1253.

Question 10:

When a patient asks why some body fat is good, the nurse responds based on what fact?

(see full question)

You selected:

The body stores energy in adipose tissue

Correct Explanation:

More than 90% of body energy is stored in adipose tissues of the body. The other options do not represent factual data about fat or adipose tissue.

Question 1:

When discussing adolescent health with a group of high school teachers, the school nurse shares that each year their student population is becoming increasingly obese. Therefore, the school nurse took the opportunity to educate the teachers about signs and symptoms of which of the following high-risk disease processes?

(see full question)

You selected:

Type 2 diabetes mellitus

Correct Explanation:

Childhood obesity is directly related to the increased incidence of type 2 diabetes. Until recently, type 2 diabetes rarely developed in children. Rheumatoid arthritis, attention d ... (more)

Reference:

Grossman S, Porth CM (2014). Porth’s Pathophysiology: Concepts of Altered Health States. 9th ed. Philadelphia: Lippincott Williams & Wilkins, Ch. 47, Alterations in Nutritional Status, p. 1254.

Question 2:

A client asks the nurse the purpose of white fat. The best response would be:

(see full question)

You selected:

Gluconeogenesis


Incorrect Correct response:

Energy storage

Explanation:

There are two types of adipose tissue: white (unilocular) and brown (multilocular). White fat is the predominant form of adipose tissue in adults. The functions of white fat includ ... (more)

Reference:

Grossman S, Porth CM (2014). Porth’s Pathophysiology: Concepts of Altered Health States. 9th ed. Philadelphia: Lippincott Williams & Wilkins, Ch. 47, Alterations in Nutritional Status, p. 1246.

Question 3:

A frail, 87-year-old female client has been admitted to a hospital after a fall and has been diagnosed with failure to thrive. Which of the following laboratory values would suggest that the client may be experiencing malnutrition?

(see full question)

You selected:

Low prealbumin

Correct Explanation:

Prealbumin levels are a reliable indicator of calorie–protein malnutrition. Low blood sugar does not necessarily indicate malnutrition, and neither C-reactive protein nor bilirub ... (more)

Question 4:

In addition to facilitating bowel movements, a diet that is high in fiber confers which of the following benefits?

(see full question)

You selected:

Increasing intestinal absorption of vitamins and minerals

Incorrect Correct response:

Lowering cholesterol and blood glucose

Explanation:

Fiber binds with cholesterol and prevents it from being absorbed by the body as well as lowers blood glucose. It does not directly detoxify the body or increase intestinal absorpti ... (more)

Reference:

Grossman S, Porth CM (2014). Porth’s Pathophysiology: Concepts of Altered Health States. 9th ed. Philadelphia: Lippincott Williams & Wilkins, Ch. 47, Alterations in Nutritional Status, p. 1249.


Question 5: (see full question)

You selected:

The nurse is caring for a client diagnosed with marasmus. The nurse would expect the client to manifest: Loss of subcutaneous tissue, stunted growth, protuberant abdomen

Correct Explanation:

Loss of subcutaneous tissue, stunted growth, and a protuberant abdomen are present in marasmus. There is inadequate food intake that leads to deficiency in calories and protein. Un ... (more)

Reference:

Grossman S, Porth CM (2014). Porth’s Pathophysiology: Concepts of Altered Health States. 9th ed. Philadelphia: Lippincott Williams & Wilkins, Ch. 47, Alterations in Nutritional Status, p. 1257.

Question 6:

The nurse is considering targeting a risk group for preventative education. Which of the following groups would benefit most from this education?

(see full question)

You selected:

Patients with anorexia nervosa

Incorrect Correct response:

Obese patients age 20–40

Explanation:

The excess body fat of obesity often significantly impairs health. As a result, obesity is the second leading cause of preventable death in the United States in adults under age 70 ... (more)

Question 7:

An 18-year-old client is seen in the emergency department after fainting at school. The client reports severe indigestion, and the nurse notes tooth erosion and an irregular heartbeat. The nurse suspects the client may have developed:

(see full question)

You selected:

Bulimia nervosa

Correct Explanation:

The client is exhibiting the complications of bulimia nervosa that result from overeating, self-induced vomiting, and cathartic and diuretic abuse. Dental abnormalities due to the ... (more)


Reference:

Grossman S, Porth CM (2014). Porth’s Pathophysiology: Concepts of Altered Health States. 9th ed. Philadelphia: Lippincott Williams & Wilkins, Ch. 47, Alterations in Nutritional Status, p. 1260.

Question 8: (see full question)

A child has been diagnosed with marasmus. Which clinical manifestations should the school nurse assess to confirm the diagnosis? Select all that apply.

You selected:

• Bradycardia

Incorrect Correct response:

• Bradycardia • Stunted growth pattern

Explanation:

Inadequate food intake, with equal deficiencies of calories and protein, is the cause of marasmus, which is characterized by low heart rate, blood pressure, and body temperature; d ... (more)

Reference:

Grossman S, Porth CM (2014). Porth’s Pathophysiology: Concepts of Altered Health States. 9th ed. Philadelphia: Lippincott Williams & Wilkins, Ch. 47, Alterations in Nutritional Status, p. 1257.

Question 9:

Which of these reflects a positive outcome to interventions provided for a 20year-old woman undergoing treatment for anorexia nervosa?

(see full question)

You selected:

Client has monthly menses.

Correct Explanation:

The individual with anorexia nervosa is expected to maintain a minimally normal body weight (e.g., at least 85% of minimal expected weight). The individual has an excessive concern ... (more)

Question 10:

When explaining to a group of parents of school-aged children the importance of protein intake, the nurse emphasizes which food choices that contain all

(see full question)


essential amino acids in adequate amounts per serving? Select all that apply. You selected:

• Milk • Chicken • Eggs

Correct Explanation:

Proteins are composed of amino acids, nine of which leucine, isoleucine, methionine, phenylalanine, threonine, tryptophan, valine, lysine, and histidine. The foods that provide the ... (more)

Question 1:

For a patient with a vitamin A deficiency, which of the following items on the menu should the nurse encourage the patient to choose for supper?

(see full question)

You selected:

• Grilled beef liver with carrots and canteloupe

Correct Explanation:

Major food sources for vitamin A are liver, butter, whole milk, cheese, egg yolk; for provitamin A: carrots, green leafy vegetables, sweet potatoes, pumpkin, winter squash, apricot ... (more)

Reference:

Grossman S, Porth CM (2014). Porth’s Pathophysiology: Concepts of Altered Health States. 9th ed. Philadelphia: Lippincott Williams & Wilkins, Chapter 47, Alteration in Nutritional Status, p. 1250.

Question 2:

The poison control nurse is helping calm a hysterical mother who called because her child ingested 10 tablets of B-complex vitamins. The mother keeps repeating over and over again, "Is my child is going to die?" Which of these should the nurse convey to the mother?

(see full question)

You selected:

Water-soluble vitamins are excreted into the urine, making toxicity less likely.

Correct Explanation:

Fat-soluble vitamins are stored in the body and may reach toxic levels if ingested in amounts greater than what is required by the body. Because the water-soluble vitamins are excr ... (more)

Reference:

Grossman S, Porth CM (2014). Porth’s Pathophysiology: Concepts of Altered Health States. 9th ed. Philadelphia: Lippincott Williams & Wilkins, Chapter 47,


Alterations in Nutritional Status, p. 1249.

Question 3: (see full question)

You selected:

In the phases of metabolism, which of the following requires energy rather than providing it for use by the body? Anabolism

Correct Explanation:

Anabolism uses energy rather than producing it. Catabolism, digestion, and breakdown of complex molecules produce energy for the body to use.

Reference:

Grossman S, Porth CM (2014). Porth’s Pathophysiology: Concepts of Altered Health States. 9th ed. Philadelphia: Lippincott Williams & Wilkins, Ch. 47, Alterations in Nutritional Status, p. 1245.

Question 4: (see full question)

When educating students about the differences between brown fat versus white fat, the instructor will share that brown fat has iron in its mitochondria that will facilitate:

You selected:

Insulation and cushioning of the body’s vital organs

Incorrect Correct response:

Production of a protein that releases the energy generated from metabolism as heat

Question 5:

When trying to explain to parents of school-aged children how to find the "good" fats that should used for cooking, which explanation is most accurate?

(see full question)

You selected:

Look for plant-based oils like "canola" on the label.

Correct Explanation:

The saturated fatty acids elevate blood cholesterol, whereas the monounsaturated and polyunsaturated fats lower blood cholesterol. Saturated fats are usually derived from animal so ... (more)

Reference:

Grossman S, Porth CM (2014). Porth’s Pathophysiology: Concepts of Altered Health States. 9th ed. Philadelphia: Lippincott Williams & Wilkins, Chapter 47,


Alterations in Nutritional Status, pp. 1248-1249.

Question 6: (see full question)

You selected:

When caring for a client with hypomagnesemia, the nurse prioritizes assessment of which of these systems? Cardiac

Correct Explanation:

Assessment of the cardiac system is essential as magnesium is needed for proper nervous system function and to maintain the tone of the blood vessels. Low magnesium levels produce ... (more)

Reference:

Grossman S, Porth CM (2014). Porth’s Pathophysiology: Concepts of Altered Health States. 9th ed. Philadelphia: Lippincott Williams & Wilkins, Chapter 47, Alterations in Nutritional Status, p. 1250.

Question 7:

Which statement is true concerning energy requirements across the lifespan?

(see full question)

You selected:

Regardless of gender, adolescents require 50 kcal/kg of body weight.

Incorrect Correct response:

A newborn requires more kcal/kg of body weight than a 10 year old.

Explanation:

Energy requirements are greater during growth periods. A person requires approximately 115 kcal/kg of body weight at birth, 105 kcal/kg at 1 year of age, and 80 kcal/kg from 1 to 10 y ... (more)

Question 8: (see full question)A nurse is caring for a patient who has been admitted to an inpatient facility for the treatment of anorexia nervosa. When planning the care of this patient, which of the following personality traits does the nurse anticipate observing?You selected:Introverted and shyIncorrectCorrect response:Perfectionistic and compulsiveExplanation: The causes of anorexia appear to be multifactorial, with determinants that include genetic influence; personality traits of perfectionism and compulsiveness; anxiety disorders; fam ... (more)

Reference:


Grossman S, Porth CM (2014). Porth’s Pathophysiology: Concepts of Altered Health States. 9th ed. Philadelphia: Lippincott Williams & Wilkins, Chapter 47, Alteration in Nutritional Status, p. 1259. Question 9: (see full question)When caring for a client in the medical clinic who has tried to lose weight multiple times, the client asks the nurse if she should try a high-protein, very low-calorie restricted diet. The nurse encourages her to seek guidance from the health care provider as these diets may cause which complication?You selected:Cardiac dysrhythmiasCorrectExplanation: VLCDs restrict calories to approximately 450 kcal/day, primarily made up of protein. This diet has higher risks, including abnormal heart rhythms and cholelithiasis. Anyone on this ... (more)

Reference: Grossman S, Porth CM (2014). Porth’s Pathophysiology: Concepts of Altered Health States. 9th ed. Philadelphia: Lippincott Williams & Wilkins, Chapter 47, Alterations in Nutritional Status, p. 1255. Question 10: (see full question)Digested dietary carbohydrates are primarily converted to:You selected:GlucoseCorrectExplanation: Dietary carbohydrates are largely converted to glucose, which is stored as glycogen in the liver and skeletal muscle cells.

Reference: Grossman S, Porth CM (2014). Porth’s Pathophysiology: Concepts of Altered Health States. 9th ed. Philadelphia: Lippincott Williams & Wilkins, Ch. 47, Alterations in Nutritional Status, p. 1249.

(see full question)

The nurse is caring for a client with chronic renal failure who must begin restricting potassium intake. Which of these foods does the nurse emphasize should be avoided?

You selected:

Potatoes

Question 1:

Correct Explanation:

Foods containing potassium include meats, poultry, fish, milk and cheese, cereals, legumes, and nuts.

Reference:

Grossman S, Porth CM (2014). Porth’s Pathophysiology: Concepts of Altered Health States. 9th ed. Philadelphia: Lippincott Williams & Wilkins, Chapter 47, Alterations in Nutritional Status, p. 1250.


Question 2: (see full question)

You selected:

A child has been diagnosed with marasmus. Which clinical manifestations should the school nurse assess to confirm the diagnosis? Select all that apply. • Stunted growth pattern • Bradycardia

Correct Explanation:

Inadequate food intake, with equal deficiencies of calories and protein, is the cause of marasmus, which is characterized by low heart rate, blood pressure, and body temperature; d ... (more)

Reference:

Grossman S, Porth CM (2014). Porth’s Pathophysiology: Concepts of Altered Health States. 9th ed. Philadelphia: Lippincott Williams & Wilkins, Ch. 47, Alterations in Nutritional Status, p. 1257.

Question 3:

Parents are concerned that their child may be at risk for the development of childhood obesity. Select the factors that would place a child at risk. Select all that apply.

(see full question)

You selected:

• Heredity • Mother had gestational diabetes during pregnancy • Ethnicity

Correct Explanation:

Childhood obesity is determined by a combination of hereditary and environmental factors. It is associated with obese parents, gestational diabetes and excessive weight gain during ... (more)

Question 7:

In addition to facilitating bowel movements, a diet that is high in fiber confers which of the following benefits?

(see full question)

You selected:

Lowering cholesterol and blood glucose

Correct Explanation:

Fiber binds with cholesterol and prevents it from being absorbed by the body as well as lowers blood glucose. It does not directly detoxify the body or increase intestinal absorpti ... (more)


Reference:

Grossman S, Porth CM (2014). Porth’s Pathophysiology: Concepts of Altered Health States. 9th ed. Philadelphia: Lippincott Williams & Wilkins, Ch. 47, Alterations in Nutritional Status, p. 1249.

Question 8: (see full question)

A nurse is instructing a patient about proper nutrition. When the MyPlate model is used for healthy eating, which of the following food groups should the nurse encourage the patient to eat the most of?

You selected:

Vegetables

Correct Explanation:

According to Figure 47.6, the MyPlate model demonstrates that the largest portion of food should come from the vegetable food group. The smallest amount of food should come from t ... (more)

Reference:

Grossman S, Porth CM (2014). Porth’s Pathophysiology: Concepts of Altered Health States. 9th ed. Philadelphia: Lippincott Williams & Wilkins, Chapter 47, Alteration in Nutritional Status, p. 1255.

Question 9:

A patient who is in the early first trimester of pregnancy has been encouraged to take a folic acid supplement. In addition, the nurse encourages the patient to eat food rich in folic acid. Which of the following foods should the nurse suggest the patient eat?

(see full question)

You selected:

Green leafy vegetables

Correct Explanation:

A major food source of folacin (folic acid) is green leafy vegetables, liver, and legumes.

Reference:

Grossman S, Porth CM (2014). Porth’s Pathophysiology: Concepts of Altered Health States. 9th ed. Philadelphia: Lippincott Williams & Wilkins, Chapter 47, Alteration in Nutritional Status, p. 1250.

Question 10:

A client asks the nurse what the body's primary source of immediate energy is.


(see full question)

The best response would be:

You selected:

Carbohydrates

Correct Explanation:

Carbohydrates are the body’s primary source of immediate energy. They supply 4 kcal/g, are stored in limited quantities as glycogen, and can be converted to fatty acids. Fats ... (more)

Question 1: (see full question)

A patient arrives at the clinic and informs the nurse that there is an "itchy rash" on his or her face. Which of the following types of deficiency should the nurse suspect that may have caused dermatitis in this patient?

You selected:

Alpha-linolenic acid

Incorrect Correct response:

Linoleic acid

Explanation:

Deficiency of linoleic acid results in dermatitis and deficiency of alpha-linolenic acid can result in neurological abnormalities and poor growth. Folic acid deficiency does not ca ... (more)

Reference:

Grossman S, Porth CM (2014). Porth’s Pathophysiology: Concepts of Altered Health States. 9th ed. Philadelphia: Lippincott Williams & Wilkins, Chapter 47, Alteration in Nutritional Status, p. 1249.

Question 2:

The nurse in the obstetrics clinic is advising a pregnant woman about nutritional needs during pregnancy. Which of these should the nurse include in the teaching plan?

(see full question)

You selected:

A pregnant woman needs an extra 300 kcal/day above the usual caloric intake.

Correct Explanation:

During pregnancy, a woman needs an extra 300 kcal/day above her usual requirement. Saturated fats usually are from animal sources and remain solid at room temperature, with the exce ... (more)

Reference:

Grossman S, Porth CM (2014). Porth’s Pathophysiology: Concepts of Altered


Health States. 9th ed. Philadelphia: Lippincott Williams & Wilkins, Chapter 47, Alterations in Nutritional Status, pp. 1248-1249.

(see full question)

A patient tells the nurse that he or she is going on a low carbohydrate diet in order to lose weight. Which of the following should the nurse inform the patient what the carbohydrate intake should be to prevent tissue wasting and ketosis?

You selected:

50–100 g/day

Question 3:

Correct Explanation:

The amount of carbohydrates needed to prevent tissue wasting and ketosis is 50–100 g/day.

Reference:

Grossman S, Porth CM (2014). Porth’s Pathophysiology: Concepts of Altered Health States. 9th ed. Philadelphia: Lippincott Williams & Wilkins, Chapter 47, Alteration in Nutritional Status, p. 1249.

Question 4: (see full question)

The nurse is caring for a client who has obesity. The nurse recognizes that psychological factors may contribute to obesity. It would be most important for the nurse to assess the client for:

You selected:

Using food as a reward

Correct

Question 5: (see full question)

You selected:

The health care provider reviews the waist–hip ratio of a male. The ratio is 1.0. The provider would interpret his result as: Upper body obesity

Correct Explanation:

A waist–hip ratio greater than 1.0 in men and 0.8 in women indicates upper body obesity. Research suggests that fat distribution may be a more important factor for morbidity and ... (more)

Reference:

Grossman S, Porth CM (2014). Porth’s Pathophysiology: Concepts of Altered Health States. 9th ed. Philadelphia: Lippincott Williams & Wilkins, Ch. 47, Alterations in Nutritional Status, p. 1253.


Question 6: (see full question)

The body mass index (BMI) is the measurement used to determine a person’s healthy weight. A BMI between 18.5 and 24.9 is considered the lowest health risk in relation to the weight of a person. How is the BMI calculated?

You selected:

BMI = weight [pounds]/height [meter2]

Incorrect Correct response:

BMI = weight [kg]/height [meter2]

Explanation:

The body mass index (BMI) uses height and weight to determine healthy weight. It is calculated by dividing the weight in kilograms by the height in meters squared (BMI = weig ... (more)

Reference:

Grossman S, Porth CM (2014). Porth's Pathophysiology: Concepts of Altered Health States. 9th ed. Philadelphia: Lippincott Williams & Wilkins, Chapter 47: Alterations in Nutritional Status, p. 1252.

Question 7:

A client diagnosed with a binge-eating disorder has made a goal to establish a regular, healthy eating pattern. Select the interventions the client should implement. Select all that apply.

(see full question)

You selected:

• Keeping a daily food intake report • Eating six small meals a day • Making a meal plan • Exercising regularly

Incorrect Correct response:

• Exercising regularly • Making a meal plan • Keeping a daily food intake report • Avoiding alcohol

Explanation:

The primary goal of therapy for binge-eating disorders is to establish a regular, healthy eating pattern. Persons with binge-eating disorders who have been successfully treated for ... (more)

Question 8:

When educating students about the differences between brown fat versus white fat, the instructor will share that brown fat has iron in its mitochondria that will facilitate:

(see full question)


You selected:

Production of a protein that releases the energy generated from metabolism as heat

Correct Explanation:

Brown fat differs from white fat in terms of its thermogenic capacity (its ability to produce heat). The color of brown fat reflects the presence of iron in its abundant mitochondr ... (more)

Reference:

Grossman, S. & Porth, C.M. Porth's Pathophysiology: Concepts of Altered Health States, 9th ed., Philadelphia: Lippincott Williams & Wilkins, 2014, Chapter 47, Alterations in Nutritional Status, p. 1246.

Question 9: (see full question)

Knowing that excess adipose tissue can result in chronic inflammation, the high school nurse should be assessing obese students for which of the following health problems?

You selected:

Type 2 diabetes

Correct Explanation:

The inflammatory response that is initiated by adipose tissue is implicated in the pathogenesis of insulin resistance and type 2 diabetes. This inflammatory process is not known to ... (more)

Reference:

Grossman S, Porth CM (2014). Porth’s Pathophysiology: Concepts of Altered Health States. 9th ed. Philadelphia: Lippincott Williams & Wilkins, Ch. 47, Alterations in Nutritional Status, p. 1254.

Question 10:

A nurse observes a patient moving restlessly in the hospital bed. Which of the following types of energy expenditure can be affected by this activity?

(see full question)

You selected:

Resting energy equivalent

Incorrect Correct response:

Nonexercise activity thermogenesis (NEAT)

Question 1:

A patient informs the nurse that he or she would like to begin a healthier dietary intake and would like to limit carbohydrate consumption.

(see full question)


Understanding that the patient must have carbohydrates to avoid tissue wasting and ketosis, Which of the following forms of carbohydrate should the nurse suggest? Select all that apply. You selected:

• Whole grains • Fruits • Vegetables

Correct Explanation:

Carbohydrates should be in the form of whole grains, vegetables, and fruits, which have higher fiber content compared to refined flour and sugar products. Ground beef and eggs are ... (more)

Reference:

Grossman S, Porth CM (2014). Porth’s Pathophysiology: Concepts of Altered Health States. 9th ed. Philadelphia: Lippincott Williams & Wilkins, Chapter 47, Alteration in Nutritional Status, p. 1249.

Question 2:

A nurse is caring for a group of patients. Which of the following patients has the greatest caloric needs?

(see full question)

You selected:

Breast-feeding mother who delivered 1 month ago

Correct Explanation:

Energy requirements are greater during growth periods. A person requires approximately 115 kcal/kg of body weight at birth, 105 kcal/kg at 1 year of age, and 80 kcal/kg from 1 to 10 y ... (more)

Reference:

Grossman S, Porth CM (2014). Porth’s Pathophysiology: Concepts of Altered Health States. 9th ed. Philadelphia: Lippincott Williams & Wilkins, Chapter 47, Alterations in Nutritional Status, p. 1248.

Question 3:

A nurse is caring for a patient with bulimia nervosa who has been admitted to the hospital with weakness, hypotension, and abdominal discomfort. Which of the following types of acid-base imbalance should the nurse assess the patient for?

(see full question)

You selected: Incorrect

Metabolic alkalosis


Correct response:

Metabolic acidosis

Explanation:

Potassium, chloride, and hydrogen are lost in the vomitus of those with bulimia nervosa, and frequent vomiting predisposes the patient to metabolic acidosis with hypokalemia.

Question 4:

In order to prevent muscle wasting, what is the minimal range of carbohydrates that must be consumed per day?

(see full question)

You selected:

50 to 100 grams

Correct Explanation:

50 to 100 grams per day must be ingested to prevent ketosis and tissue wasting.

Reference:

Grossman S, Porth CM (2014). Porth’s Pathophysiology: Concepts of Altered Health States. 9th ed. Philadelphia: Lippincott Williams & Wilkins, Ch. 47, Alterations in Nutritional Status, p. 1249.

Question 5: (see full question)

A nurse is educating a group of patients about adequate nutrition. Which of the following should the nurse include when discussing diets that are adequate in calories but deficient in protein?

You selected:

Patients may be at risk for developing anemia.

Incorrect Correct response:

Patients may be at risk for developing kwashiorkor.

Explanation:

Diets that are adequate in calories, but inadequate in protein can result in kwashiorkor. If both calories and protein are inadequate, protein-calorie malnutrition occurs. Patients ... (more)

Reference:

Grossman S, Porth CM (2014). Porth’s Pathophysiology: Concepts of Altered Health States. 9th ed. Philadelphia: Lippincott Williams & Wilkins, Chapter 47, Alteration in Nutritional Status, p. 1248.

Question 6:

A frail, 87-year-old female client has been admitted to a hospital after a fall and has been diagnosed with failure to thrive. Which of the following laboratory values would suggest that the client may be experiencing malnutrition?

(see full question)


You selected:

Low prealbumin

Correct Explanation:

Prealbumin levels are a reliable indicator of calorie–protein malnutrition. Low blood sugar does not necessarily indicate malnutrition, and neither C-reactive protein nor bilirub ... (more)

Reference:

Grossman S, Porth CM (2014). Porth’s Pathophysiology: Concepts of Altered Health States. 9th ed. Philadelphia: Lippincott Williams & Wilkins, Ch. 47, Alterations in Nutritional Status, p. 1258.

Question 7:

Which of the following statements best conveys the endocrine function of adipose tissue? Adipose tissue:

(see full question)

You selected:

Antagonizes the effects of insulin on cell membranes.

Incorrect Correct response:

Produces leptin, which mediates body weight.

Question 8:

A patient informs the nurse that he or she feels as though he or she have a kidney stone again. The patient is on a diet for weight loss and feels as though this may be a contributing factor as it never occurred before going on the diet. Which of the following diets does the nurse suspect the patient may be on?

(see full question)

You selected:

Low-carbohydrate/high protein

Correct Explanation:

Carbohydrate diets are effective for weight loss, especially in the initial stages but can contribute to health risks. Higher protein diets can increase the risk of kidney stones, ... (more)

Reference:

Grossman S, Porth CM (2014). Porth’s Pathophysiology: Concepts of Altered Health States. 9th ed. Philadelphia: Lippincott Williams & Wilkins, Chapter 47, Alteration in Nutritional Status, p. 1255.


Question 9: (see full question)

You selected:

A nurse is assigned to care for a group of patients. Which of the following patients is most likely to be diagnosed with malabsorption syndrome? A 5-year-old taking pancreatic enzymes for cystic fibrosis

Incorrect Correct response:

A 40-year-old with severe inflammatory bowel disease

Explanation:

Some types of malnutrition are caused by acute and chronic illnesses, such as occurs in people with Crohn's disease who are unable to absorb nutrients from their food. Lactose int ... (more)

Reference:

Grossman S, Porth CM (2014). Porth’s Pathophysiology: Concepts of Altered Health States. 9th ed. Philadelphia: Lippincott Williams & Wilkins, Chapter 47, Alterations of Nutritional Status, p. 1257.

Question 10: (see full question)

A nurse is assisting a group of patients with a weight loss program and is to determine the obesity type that each patient has. Which of the following would help the nurse determine the type of obesity?

You selected:

Dividing the waist measurement by the hip circumference

Correct Explanation:

Obesity type is determined by dividing the waist by the hip circumference. Comparing the waist measurements can identify the type of obesity.

Question 1:

When caring for a client with hypomagnesemia, the nurse prioritizes assessment of which of these systems?

(see full question)

You selected:

Cardiac

Correct Explanation:

Assessment of the cardiac system is essential as magnesium is needed for proper nervous system function and to maintain the tone of the blood vessels. Low magnesium levels produce ... (more)

Reference:

Grossman S, Porth CM (2014). Porth’s Pathophysiology: Concepts of Altered Health States. 9th ed. Philadelphia: Lippincott Williams & Wilkins, Chapter 47,


Alterations in Nutritional Status, p. 1250.

Question 2: (see full question)

You selected:

In the phases of metabolism, which of the following requires energy rather than providing it for use by the body? Anabolism

Correct Explanation:

Anabolism uses energy rather than producing it. Catabolism, digestion, and breakdown of complex molecules produce energy for the body to use.

Reference:

Grossman S, Porth CM (2014). Porth’s Pathophysiology: Concepts of Altered Health States. 9th ed. Philadelphia: Lippincott Williams & Wilkins, Ch. 47, Alterations in Nutritional Status, p. 1245.

Question 3:

A nurse observes a patient moving restlessly in the hospital bed. Which of the following types of energy expenditure can be affected by this activity?

(see full question)

You selected:

Nonexercise activity thermogenesis (NEAT)

Correct Explanation:

NEAT includes the energy expended in maintaining posture and in activities such as fidgeting. Diet-induced thermogenesis describes the energy used by the body for the digestion, ab ... (more)

Question 1:

When caring for a client with hypomagnesemia, the nurse prioritizes assessment of which of these systems?

(see full question)

You selected:

Cardiac

Correct Explanation:

Assessment of the cardiac system is essential as magnesium is needed for proper nervous system function and to maintain the tone of the blood vessels. Low magnesium levels produce ... (more)

Reference:

Grossman S, Porth CM (2014). Porth’s Pathophysiology: Concepts of Altered


Health States. 9th ed. Philadelphia: Lippincott Williams & Wilkins, Chapter 47, Alterations in Nutritional Status, p. 1250.

Question 2: (see full question)

You selected:

In the phases of metabolism, which of the following requires energy rather than providing it for use by the body? Anabolism

Correct Explanation:

Anabolism uses energy rather than producing it. Catabolism, digestion, and breakdown of complex molecules produce energy for the body to use.

Reference:

Grossman S, Porth CM (2014). Porth’s Pathophysiology: Concepts of Altered Health States. 9th ed. Philadelphia: Lippincott Williams & Wilkins, Ch. 47, Alterations in Nutritional Status, p. 1245.

Question 3:

A nurse observes a patient moving restlessly in the hospital bed. Which of the following types of energy expenditure can be affected by this activity?

(see full question)

You selected:

Nonexercise activity thermogenesis (NEAT)

Correct Explanation:

NEAT includes the energy expended in maintaining posture and in activities such as fidgeting. Diet-induced thermogenesis describes the energy used by the body for the digestion, ab ... (more)

Question 4:

A patient presents to the clinic with severe edema. Which of the following types of deficiency should be suspected in this patient?

(see full question)

You selected:

Protein

Correct Explanation:

Severe protein deficiency is associated with extensive loss of the visceral protein compartment with a resultant hypoalbuminemia that gives rise to generalized or dependent edema.


Reference:

Grossman S, Porth CM (2014). Porth’s Pathophysiology: Concepts of Altered Health States. 9th ed. Philadelphia: Lippincott Williams & Wilkins, Chapter 47, Alterations in Nutritional Status, p. 1257.

Question 5:

Which of the following factors does the nurse recognize as being independent predictors of risk factors and mortality for obese patients?

(see full question)

You selected:

Presence of excess fat in the abdomen out of proportion to total body fat

Correct Explanation:

The presence of excess fat in the abdomen out of proportion to total body fat is an independent predictor of risk factors and mortality. Both BMI and waist circumference are positi ... (more)

Reference:

Grossman S, Porth CM (2014). Porth’s Pathophysiology: Concepts of Altered Health States. 9th ed. Philadelphia: Lippincott Williams & Wilkins, Chapter 47, Alteration in Nutritional Status, p. 1253.

Question 6:

A nurse is explaining how the majority of body energy is stored in the body. The best explanation would be through:

(see full question)

You selected:

Triglycerides

Correct Explanation:

More than 90% of body energy is stored as triglycerides in the fat cells of the body. The body has a limited ability to store dietary carbohydrates and proteins as energy sources.

Reference:

Grossman S, Porth CM (2014). Porth’s Pathophysiology: Concepts of Altered Health States. 9th ed. Philadelphia: Lippincott Williams & Wilkins, Ch. 47, Alterations in Nutritional Status, p. 1246.

Question 7:

The nurse is considering targeting a risk group for preventative education. Which of the following groups would benefit most from this education?

(see full question)

You selected:

Obese patients age 20–40


Correct

Question 8: (see full question)

You selected:

An obese patient is being seen in the clinic for treatment of a weight problem. The patient states that he or she has been on multiple dietary regimens without success. In which of the following situations is when treatment is indicated? Select all that apply. • There is a large waist circumference with a BMI of 25–29 • There is a body mass index (BMI) of 30 or higher • There are two or more risk factors • The patient requests help

Incorrect Correct response:

• There is a body mass index (BMI) of 30 or higher • There is a large waist circumference with a BMI of 25–29 • There are two or more risk factors

Explanation:

The current recommendation is that treatment is indicated in all people who have a BMI of 30 or higher and in those who have a BMI of 25–29.9 or a large waist circumference a ... (more)

Reference:

Grossman S, Porth CM (2014). Porth’s Pathophysiology: Concepts of Altered Health States. 9th ed. Philadelphia: Lippincott Williams & Wilkins, Chapter 47, Alteration in Nutritional Status, p. 1255.

Question 9:

Which of the following clients coming to a small free clinic are at high risk for malnutrition? Select all that apply.

(see full question)

You selected:

• A 4-year-old child who lives with a single mom in a rooming house • A 60-year-old homeless Vietnam veteran complaining of pain • An 88-year-old senior citizen on a fixed budget

Correct Explanation:

Among the many causes of malnutrition are poverty and lack of knowledge, acute and chronic illness, and self-imposed dietary restrictions. Homeless people, the elderly, and the chi ... (more)

Reference:

Grossman, S. & Porth, C.M. Porth's Pathophysiology: Concepts of Altered


Health States, 9th ed., Philadelphia: Lippincott Williams & Wilkins, 2014, Chapter 47, Alterations in Nutritional Status, p. 1257.

Question 10: (see full question)

The body mass index (BMI) is the measurement used to determine a person’s healthy weight. A BMI between 18.5 and 24.9 is considered the lowest health risk in relation to the weight of a person. How is the BMI calculated?

You selected:

BMI = weight [kg]/height [meter2]

Correct

Question 5:

Which of the following body mechanisms is necessary for food intake control?

(see full question)

You selected:

Leptin receptor stimulation

Correct Explanation:

Leptin receptor stimulation of the hypothalamus causes decreased appetite and increased metabolic rate/energy consumption. Natural appetite suppression is a response to cholecystoki ... (more)

Reference:

Grossman S, Porth CM (2014). Porth’s Pathophysiology: Concepts of Altered Health States. 9th ed. Philadelphia: Lippincott Williams & Wilkins, Ch. 47, Alterations in Nutritional Status, p. 1246.

Question 6:

A client asks the nurse what the body's primary source of immediate energy is. The best response would be:

(see full question)

You selected:

Carbohydrates

Correct Explanation:

Carbohydrates are the body’s primary source of immediate energy. They supply 4 kcal/g, are stored in limited quantities as glycogen, and can be converted to fatty acids. Fats ... (more)


Reference:

Grossman, S. & Porth, C.M. Porth's Pathophysiology: Concepts of Altered Health States, 9th ed., Philadelphia: Lippincott Williams & Wilkins, 2014, Chapter 47, Alterations in Nutritional Status, p. 1249.

Question 7:

The parish nurse is serving in a third-world country where protein calorie malnutrition is common. When assessing the children, which of these does the nurse anticipate will be present in the children with marasmus? Select all that apply.

(see full question)

You selected:

• Infections • Small stature

Incorrect Correct response:

• Diarrhea • Infections • Small stature

Explanation:

The child with marasmus has a wasted appearance, with loss of muscle mass, stunted growth, loss of subcutaneous fat, a protuberant abdomen, wrinkled skin, sparse, dry, and dull hai ... (more)

Question 8: (see full question)

A nurse is educating a group of patients about adequate nutrition. Which of the following should the nurse include when discussing diets that are adequate in calories but deficient in protein?

You selected:

Patients may be at risk for developing kwashiorkor.

Correct Explanation:

Diets that are adequate in calories, but inadequate in protein can result in kwashiorkor. If both calories and protein are inadequate, protein-calorie malnutrition occurs. Patients ... (more)

Reference:

Grossman S, Porth CM (2014). Porth’s Pathophysiology: Concepts of Altered Health States. 9th ed. Philadelphia: Lippincott Williams & Wilkins, Chapter 47, Alteration in Nutritional Status, p. 1248.

Question 9:

The poison control nurse is helping calm a hysterical mother who called because her child ingested 10 tablets of B-complex vitamins. The mother keeps repeating over and over again, "Is my child is going to die?" Which of

(see full question)


these should the nurse convey to the mother? You selected:

Water-soluble vitamins are excreted into the urine, making toxicity less likely.

Correct Explanation:

Fat-soluble vitamins are stored in the body and may reach toxic levels if ingested in amounts greater than what is required by the body. Because the water-soluble vitamins are excr ... (more)

Reference:

Grossman S, Porth CM (2014). Porth’s Pathophysiology: Concepts of Altered Health States. 9th ed. Philadelphia: Lippincott Williams & Wilkins, Chapter 47, Alterations in Nutritional Status, p. 1249.

Question 10: (see full question)

The nurse is providing dietary instruction to a client whose lab values indicate a high level of blood cholesterol. The client asks if there are any food contents that need to be avoided. The best response would be:

You selected:

Saturated fatty acids

Correct Explanation:

The saturated fatty acids elevate blood cholesterol, whereas the monounsaturated and polyunsaturated fats lower blood cholesterol.


Turn static files into dynamic content formats.

Create a flipbook
Issuu converts static files into: digital portfolios, online yearbooks, online catalogs, digital photo albums and more. Sign up and create your flipbook.